EM EOR
infection of uterus during miscarriage :( cervical os open with purulent cervical discharge + uterine tenderness tx: COMPLETE evacuation of uterine contents
Describe septic spontaneous abortion
Colonoscopy is ordered 4-8 weeks after resolution of symptoms and initial episode to exclude colorectal cancer.
If a patient is presenting with their first diverticulitis, what should be ordered thereafter outpatient?
drown it with liquid 2-4% lidocaine or mineral oil then refer to ENT not a true emergency
If a patient presents with a live insect in their ear, how should this be treated?
1 year isoniazid prophylaxis for household members stop therapy with 2 negative AFB smears + cultures in a row
What is the prophylaxis for household members living with someone with an active TB infection? When can a active TB patient stop therapy?
blowout fracture patho: blunt trauma resulting in inferior orbital wall fracture presentation: -eyelid swelling, ecchymosis -decreased VA -enophthlamos (sunken eye) -pain with EOM -epistaxis -damage to infraorbital nerve: paresthesias -entrapped inferior rectus muscle, cannot look up gum/lips CT scan to dx treatment: PROMPT opthalmic referral; surgical tx
16-year-old who was hit in the right eye by a baseball. The area is ecchymotic and swollen. He complains of pain, rated 6 out of 10. On physical exam, the patient has eyelid swelling, decreased visual acuity, enophthalmos (sunken eye), and anesthesia/paresthesia in the gums and upper lips. What is the likely diagnosis?
Clavicular fracture patho: direct fall on lateral aspect of shoulder MCC. In newborns birth trauma presentation: - Middle 1/3 most common site - swelling / erythema / deformity of clavicle - TTP - tenting of skin may occur diagnostics: AP and clavicle view x-rays treatment: - simple arm sling 4-6 weeks - IF proximal 1/3 fractured: consult ortho - begin PT after 4 weeks
23-year-old woman arrives at the emergency room after a biking accident. She reports acute pain after falling on her shoulder. On physical exam, there is swelling, erythema, and tenderness on the anterior aspect of her right shoulder. No tenting of the skin is noted. Distal pulses are intact, and there is no motor or sensory deficits. X-ray of the shoulder shows the following. What is the likely diagnosis?
Crohn's disease patho: transmural thickening, mouth to anus, skip lesions presentation: -RLQ pain -apthous ulcers -non-bloody diarrhea -weight loss -fistulas* labs: -ASCA + -barium enema: cobblestoning treatment: -prenidone, mesalamine -surgery not curative
25-year-old man with an 18-month history of chronic abdominal pain. Associated with this abdominal pain for the past 3 months have been nonbloody diarrhea, anorexia, and a weight loss of 20 pounds. He has developed a painful area around the anus. On examination, the patient has diffuse abdominal tenderness. He looks thin and unwell. He has a tender, erythematous area in the right perirectal area. What is the likely diagnosis? -patho -presentation -treatment
patellofemoral syndrome patho: overuse injury presentation: - anterior knee pain around patella worse with knee hyperflexion - compression of patella during knee extension will prduce symptoms or apprehensiveness treatment: - NSAIDS / rest - strengthening of quadriceps - elastic knee sleeve
28 year-old female presents with knee pain. Her pain worsens when she is running downhill or climbing up the stairs. She describes the pain as "achy" and being behind the knee. When sitting for long periods of time, she reports her knees feeling stiff. She denies any recent history of trauma and has never had surgery. She has tried multiple over-the-counter analgesics but were not effective. She is a registered dietician and an avid runner who is preparing for a marathon. What is the likely diagnosis? -patho -presentation -treatment
epididymitis patho: retrograde spread of organisms via vas deferens presentation: -dysuria -unilateral dull aching scrotal pain radiates to the ipsilateral flank -swollen, tender epididmyis -POSITIVE prehn sign (relief with scrotal elevation ) diagnostics: UA pyuria and bacteriuria treatment: <35: ceftriaxone + doxycycline >35: levofloxacin or bactrim if pt practices insertive anal intercourse: Ceftriaxone + fluroquinolone
28-year-old male with a dull, achy scrotal pain that has been gradually increasing over the last several days. He also reports pain with urination. Physical exam reveals a swollen right testicle with substantial induration. Urinalysis reveals positive leukocyte esterase and 20 WBC/HPF. What is the likely diagnosis? -patho -presentation -diagnostics -treatment
premature rupture of membranes >37 weeks gestation prior to start of uterine contractions (preterm PROM = <37 weeks) presentation: sudden gush of clear or pale yellow fluid diagnostics: confirm it's amniotic fluid -speculum: fluid pooling at posterior fornix -nitrazine test: turns blue (pH amniotic fluid >7.1) -ferning: fluid specimen on slide will crystallize into a fern pattern treatment: - >34 weeks: induce labor - 32-34 weeks: check lung maturity then induce - <32 weeks: stop contractions and start 2 doses of steroid injection then delivery baby, give Mom ABx complications w/ PROM: infection + cord prolapse
31-year-old female G3P1 at 37 weeks gestation presents to the ED reporting a sudden gush of fluid from the vagina. She reports that the fluid is mostly clear and denies the presence of blood or mucus. External fetal monitoring demonstrates reactive fetal heart tracing and no uterine contractions. On speculum exam, you note a closed cervical os and a pool of clear/yellow fluid in the vaginal vault. The fluid is fern and nitrite positive. What is the likely diagnosis? -definition -presentation -diagnostics -treatment
Histoplasmosis bird or bat droppings (cave, zoo); Missipisspi Ohio river x-ray: mediastinal or hilar lymphadenopathy , BB sized calcifications treatment: itraconazole or Amphotericin B
31-year-old male presents with a cough and mild shortness of breath for three days. The cough is occasionally productive of yellowish mucus. He reports a low-grade fever with this episode but says that he has otherwise been healthy. He has spent the last month working in bat caves. He denies tobacco or alcohol use. What is the likely diagnosis? -causes -diagnostics -treatment
placenta previa patho: placenta lies very low in the uterus and covers all or part of cervix presentation: >28 weeks painless vaginal bleeding diagnostic: ultrasound (transvaginal) DO NOT DO A PELVIC EXAM! CONTRAINDICATED! b/c can cause further seperation of placental attachment treatment: -strict bed rest: no intercourse, no vigorous exercise -type + screen (blood transfusion may be prequired) -C section preferred delivery method
33-year-old woman, G2P1, at 35 weeks' gestation with a complaint of painless vaginal bleeding that began two hours ago and has delivered a substantial amount of blood with clots. She has had no evident pain or cramping. Upon physical examination, the fetal heart rate is noted to be normal. Her last pregnancy was delivered by emergency cesarean at 37 weeks due to a breech presentation during labor. What is the likely diagnosis? -patho -presentation -diagnostics -treatment
gastritis (due to NSAID + alcohol use) patho: diminishing local prostaglandin production in stomach + duodenum presentation: -belching -bloating -distension -<3 burn -abdominal pain tx: STOP NSAIDS -4-8 weeks of PPI (acid suppression)
37-year-old male with a history of daily NSAID use complaining of epigastric pain, nausea, and vomiting, all worsened by eating. On physical examination, he is tender to palpation in the epigastrium. He admits to drinking approximately two beers per day. What is the likely diagnosis?
gastritis due to H. pylori infection patho: H. pylori is a gram negative spiral shaped bacteria labs: -Urea breath test -fecal antigen -IgG antibodies -endoscopy w/ biopsy treatment: -bismuth + PPI + tetracycline + metronidazole
38-year-old male presents with epigastric pain, nausea, and vomiting, all worsened by eating. On physical examination, he is tender to palpation in the epigastrium. Urea breath test is positive. What is the likely diagnosis? -patho -labs -treatment
Thoracic outlet syndrome patho: pressure on the blood vessels / nerves in the ribs, collarbones, or neck muscles. Usually d/t trauma, injuries, pregnancy, or extra rib presentation: - pain in shoulders + neck - numbness, weakness, and coldness in the finger - Adson's maneuver: decreased radial pulse diagnostics: CT angio treatment: PT + pain relief
40 year-old man presents with difficulty grasping his golf club. The patient reports difficulty with maintaining a strong grip in various situations. This symptom is accompanied by numbness and tingling. On physical exam, there is atrophy of his intrinsic hand muscles. What is the likely diagnosis?
stress incontinence causes: weakness of pelvic floor → loss of bladder support → increased pressure on bladder **multiple vaginal deliveries** presentation: incontinence during activities that increase abdominal pressure such as sneezing, laugh, walking upstairs treatment: -Kegal exercises -vaginal estrogens -pessary -mid urethral sling (surgery)
45 year-old female G5P5 complains of small quantities of urine leaking when she coughs, sneezes, or laughs. Her genitourinary examination is unremarkable and her urinalysis is normal. What is the likely diagnosis? -causes -presentation -treatment
candidiasis diagnosis: KOH prep branching hyphae, spores, pseudohyphae tx: fluconazole, clotrimazole (topical)
A 20 year old presents with white curd like vaginal discharge and a diffusely itchy vulva. She was recently prescribed Augmentin for a sinus infection. KOH prep shows pseudohypae. What is the likely diagnosis & treatment? -risk factors -diagnosis
Bed rest <2 days + NSAIDS + muscle relaxants resume activity as tolerated; if not improved in 4 weeks return to clinic
What is the treatment for back strain associated low back pain
plantar fasciitis patho: chronic overuse results in microtears of the plantar fascia presentation: inferior heel pain that is worse AFTER period of rest (i.e. first steps in the morning out of bed) -pain increases w/ dorisflexion of toes risk factors: -pes planus -pes cavus -obesity -sedentary lifestyle -excessive running treatment: shoe inserts, NSAIDS
47-year-old man presents with a history of worsening burning left heel pain for the last few weeks. He states that it is worse in the morning after getting out of bed and beginning to walk; it improves with exercise. He denies any foot numbness, tingling, or known injury. He states that he recently began a jogging regimen. What is the likely diagnosis? -patho -presentation -risk factors
Acute Lymphocytic Leukemia (ALL) presentation: CHILDREN -LAD -bone pain -fever -bleeding
5 y/o child with bone pain, bleeding, and fever. His mom notes that the patient has been less active and has had recurrent upper respiratory infections over the past several months. Clinical examination reveals diffuse lymphadenopathy. His CBC demonstrates anemia and leukopenia, and the blood smear shows peripheral blasts. What is the likely diagnosis?
influenza everyone >6 months should get an annual flu vaccine presentation: -fever -coryza -cough -headache -malaise diagnostics: rapid antigen test -primary influenza pneumonia: bilateral diffuse infiltrates treatment: -antivirals <48 hours (Zanamivir + Osteltamivir tx influenza A and B)
5-year-old with sudden onset of fever, chills, malaise, sore throat, headache, and coryza. The child is also complaining of myalgia, especially in her back and legs. On physical exam, the patient appears lethargic, has a temperature of 102.5 F, and palpable cervical lymph nodes. Breath sounds are distant with faint end-expiratory wheezes. Chest xray is negative for pneumonia. What is the likely diagnosis? -presentation -diagnostics -treatment
Elbow (Olecranon) bursitis must aspirate to r/o septic arthritis + gout
50 y/o computer programmer with right elbow swelling for the past few months. The swelling has worsened and became more painful over the last week. He works 12 hours a day and spends a lot of time at his desk keyboarding and working with his mouse. On examination, he has no fever. There is a red and tender swelling of the left posterior elbow. What is the likely diagnosis?
Osteomyelitis patho: can occur as a result of hematgenous seeding, continuous spread of infection, or direct inoculation into intact bone presentation: -fever - restricted ROM with involved extremity - refusal to bear weight diagnostics: - x-ray (although MRI will show changes before x-ray) - elevated CRP - definitive: blood culture or bone bx treatment: 4-6 weeks of ABx - Cefazolin + Naficillin (S. aureu coverage) - puncture wound: pseudomonas coverage w/ FQ
54-year-old male with fever, chills, and pain in the left foot. His symptoms progressively worsened over the course of a week. Medical history is significant for poorly controlled type II diabetes mellitus. On physical exam, there is tenderness to palpation of the left foot. Laboratory testing is significant for an elevated erythrocyte sedimentation rate and C-reactive protein; as well as, leukocytosis. Plain radiograph demonstrates periosteal thickening and soft tissue swelling. What is the most likely diagnosis? -patho -presentation -diagnostics -treatment
Hammer Toe / Mallet Toe patho: deformity of the 2nd, 3rd, or 4th toe presentation: toe is bent at middle join (flexion of PIP) looks like hammer; hyper-extension with MTP + DIP joitn treatment: - change footwear (avoid narrow / high heeled shpes) - exercises: toe curls - surgery last line
55 year-old female presents with a painful deformity of the second toe. The toe is painful, especially when she is running, walking long distances, or wearing shoes. On physical exam, there is swelling and redness of the PIP joint, an inability to straighten the second toe, and a callus on the top of the PIP joint. What is the likely diagnosis?
pulmonary embolism S1Q3T3
What is this EKG finding associated with?
cirrhosis presentation: -ascites -pulmonary edema -esophageal varices -Terry Nails (white nail beds) -spider angioma -palmer erythema -jaundice labs: -AST >ALT (alcohol induced) -hypoalbuminemia -low platelets (portal HTN, platelets stuck in spleen) -U/S: nodular liver treatment: tx focuses on specific symptoms -avoid alcohol, restrict salt intake
60-year old man presents to you on account of right upper quadrant pain, anorexia, and fatigue. On history taking, patient revealed that he has been taking excessive alcohol for over 20 years. Physical examination reveals icterus, gynecomastia, spider angioma, hepatomegaly, shifting dullness, visibly distended abdominal wall veins. Serum AST, AST, and ALP are elevated. What is the likely diagnosis? -presentation -labs -treatment
Chronic Lymphocytic Leukemia (CLL) most common type of leukemia in adults presentation: fatigue, LAD, splenomegaly diagnostics: -mature lymphocytes -smudge cells on PBS
60-year-old male complains of fatigue. On physical exam he has painless cervical lymphadenopathy and hepatosplenomegaly. His blood tests demonstrate severe anemia, decreased neutrophil count. PBS shows smudge cells. What is the likely diagnosis? -presentation -diagnostics -treatment
large bowel obstruction MCC: cancer, volvulus presentation: elderly -increasing abdominal pain -abdominal distension -obstipation -less vomiting than SBO KUB: dilated loops of bowel with air fluid levels with little or no gas in colon
61-year-old male with abdominal distension and colicky pain. Physical exam reveals high-pitched bowel sounds and diffuse abdominal tenderness. The abdominal radiograph demonstrates a distended proximal colon with haustra, air -fluid-levels and no gas in the rectum. What is the likely diagnosis? -MCC -presentation
mitral regurgitation murmur: holosystolic high pitched blowing murmur at the apex and radiates to axilla with a split S2
64-year-old obese man with a history of hyperlipidemia presents to the ED with shortness of breath. His lung exam is notable for bibasilar crackles. On cardiac exam, you note a hyperactive precordium with a new III/VI blowing holosystolic murmur at the apex with a split S2 that radiates to the axilla. What is the likely diagnosis? describe the murmur
rotator cuff tear MC injury = supraspinatous presentation: -shoulder pain with overhead activity or at night while lying on arm -weakness -acute injury: immobility diagnostics: -xray: initial imaging (loss of subacromial space due to upward migration of humeral head) -MRI: most accurate treatment: -NSAIDS, steroid injection, PT -surgical repair IF fail 3-6 months of conservative tx
69-year-old male with right shoulder pain for the past several months. He reports that he cannot reach above his head without severe pain. As a retired carpenter, he reports that this has significantly impacted his quality of life. Additionally, he is unable to lie on his left side at night due to shoulder pain. On physical exam, there is focal tenderness over the left anterolateral shoulder. Radiography reveals reduced space between the acromion and humeral head. What is the likely diagnosis? -presentation -diagnostics -treatment
prostatitis presentation: -fever -lower back pain -urinary sx: frequency, urgency, dysuria -DRE: large, tender, boddy prostate diagnostics: -UA: pyuria + hematuria -prostatic fluid: leukocytosis **DO NOT MASSAGE PROSTATE as this can lead to sepsis! treatment: - <35: Ceftriaxone + Doxy -> 35: E. Coli + pseudomonas MCC → fluoroquinolones or bactrim for 4-6 weeks (if chronic, 6-12 weeks tx)
70 y/o man with a history of benign prostatic hyperplasia reports 3-days of fever, chills, and pain with urination. He was recently catheterized during admission to the hospital. Physical exam reveals a tender and enlarged prostate on digital rectal exam. Urinalysis reveals pyuria and hematuria. What is the likely diagnosis? -presentation -diagnostics -treatment
mitral stenosis murmur: diastolic rumbling murmur with opening snap heard best at apex
72-year-old female who presents to your office for a routine check-up. While she otherwise feels well, it has been a long time since she last received medical care. On exam her you note an apical, rumbling diastolic murmur with a split s1 that occurs following an opening snap. The rumbling is loudest at the start of diastole and is heard best at the left sternal border and apex. She has no other physical exam findings and has no other past medical history. What is the likely diagnosis? -describe
macular degeneration (wet) patho: gradual painless loss of central vision (b/c macula = central vision acuity) glaucoma presents with peripheral vision loss initially wet: advanced form of dry age related macular degeneration (responsible for most cases of blindness d/t MD) -neovascularization occurs leading to retinal hemorrhages and scarring -amsler gride: straight lines appear bent tx: -intravitrol VEGF inhibitors (bevacizumab) to reduce neovascularization -zinc and antioxidant vitamins
82 year-old male who arrives for his follow-up visit for worsening central visual loss. He describes a phenomenon of wavy or distorted vision that has deteriorated rather quickly. He is "having difficulty seeing words when he reads." When looking at a specific region of the Amsler grid, he reports a dark "spot" in the center, with bent lines. On the fundoscopic exam there is neovascularization and small retinal hemorrhages. What is the likely diagnosis? -patho -presentation on fundascopic exam -treatment
foreign body aspiration patho: FB lodges in supraglottic airway, triggering protective reflexes that result in laryngospasm (R main bronchus site) presentation: 6 mo-3 year highest risk -sudden onset of coughing, stridor, wheezing, choking, respiratory distress -difficulty speaking -unexplained nose bleed or vomiting of blood -cyanosis -asymmetrical decreased breath sounds or localized wheezing diagnostics: -nose to anus x-ray r/o FB ingestion -asymmetrical radiograph findings (air does not exit lung; more hypodense) -ABG treatment: EMERGENT ACTION -rigid bronchoscopy removes foreign body (flexible or rigid can diagnose)
A 1 year old child comes into the ER because their mom is concerned about an abrupt onset of cough. The sputum associated with the cough is foul smelling, the baby has a fever and audible wheeze. The patient has a 5 year old brother who has lots of Legos. What is the likely diagnosis? -patho -presentation -diagnostics -treatment
(c) A. An indirect inguinal hernia is caused by a patent processus vaginalis and the hernial contents may be felt in the ipsilateral scrotum.
A 20 year-old male presents with a mass in the groin. On examination with the patient standing, a mass is noted that extends into the scrotum. The patient denies any trauma. The most likely diagnosis is A. an indirect inguinal hernia. B. a direct inguinal hernia. C. an obturator hernia. D. a femoral hernia.
-maternal weight -BP -fundal height -fetal size -urine dipstick
What labs are done at every prenatal visit
acute glomerulonephritis (caused by Steph. pharyngitis) causes: -post-infectious s/p strep infection -goodpastures, Wegner's presentation: common findings despite different causes* -hematuria -HTN -peripheral & periorbital edea UA: dysmorphic RBCs FeNa >1% (abnormal) R U/S no hydronephrosis treatment: steroids control inflammatory response
A 10 year old female is presenting with an 3 day swelling around her eyelids. She has also had scant urine that is coca-cola colored. She was diagnosed with Strep throat 2 weeks ago and just finished her Amoxicillin. Blood work shows elevated SCR. UA with microscopy shows dysmorphic RBC and proteinuria. What is the likely diagnosis? (be specific!) -causes -presentation -diagnostics -treatment
acute osteomyelitis of the tibia patho: inoculation + spread via blood steam (in this case dental work), continuous spread of infection, or direct inoculation of bone MCC = S. aureus presentation: -pain -decreased ROM, refusal to bear weight -soft tissue swelling -fever can occur diagnostics: -xray will be normal in early infections, MRI is first line in early detection -can also utilize bone scan and blood culture treatment: 4-6 weeks of IV antibiotics (depending on pathogen) but usually requires: cefazolin, nafcillin or vanco (MRSA) **want to track CRP to ensure it is going down with treatment
A 12 year old is brought into the ER because she has developed a 103 degree fever. She is unable to weight bear on her right leg and her lower leg is swollen, and tender to palpation. Her parents recently took her to get a root canal a month ago for a dead tooth. X-ray is normal but MRI shows the above photo. Bloodwork shows elevated WBC, elevated ESR and CRP. What is the likely diagnosis? -patho -presentation -diagnostics -treatment
acute asthma exacerbation: severe changes from patient baseline -supplemental O2 (hypoxemia) -hydration: 1-2 L NS bolus over 1 hour -nebulized albuterol -predinisone -plus or minus: ipratropium, MgSO4
A 12-year-old boy presents to the ED with sudden onset of dyspnea with wheezing. The patient had a similar appearance a month ago. An arterial blood gas shows hypoxemia, hypercapnia, and acidosis. The chest x-ray shows clear lung fields. He is afebrile. How should this patient be treated?
threatened -viable pregnancy -bleeding - +/- cramping -cervical os CLOSED give pt progesterone
A 15 week G1P0 pregnant female presents to the clinical with vaginal bleeding and cramping abdominal pain. She is quite worried something is wrong with the baby. On transvaginal ultrasound, there is cardiac activity of the fetus and the cervical os is closed. What type of abortion classification is this?
infectious mononucleosis pathogen: EBV diagnosis: -EBV titer -Monospot test: heterophile antibody -atypical lymphocytes treatment: supportive -need to ensure splenomegaly resolves prior to resuming sports (3 weeks non-contact, 4 weeks contact)
A 15-year-old boy presents to the clinic with fever, posterior cervical lymphadenopathy, and sore throat. On PE tonsils present with exudates and splenomegaly is present. What is the likely diagnosis? -pathogen -diagnosis -treatment
encephalitis caused by HSV (MCC) presentation: sx begin with flu like illness -focal neurologic effects like hemiparesis, CN palsies, and sensory defects, altered mental status, personality changes, encephalitis different from meningitis by altered brain functioning diagnostics: -LP + CSF: normal glucose, normal or mildly elevated protein and pressure, elevated WBC with lymphocyte dominant -MRI: temporal edema -Kernig + Brudnznki usually absent tx: IV acyclovir for HSV encephalitis
A 15-year-old girl presents with loss of consciousness. She is accompanied by her mother, who states that the patient initially fell ill several days ago with a headache, muscle aches, and fever. The patient developed a severe headache today, accompanied by double vision, difficulty speaking, confusion, and eventual loss of consciousness. She has not taken any medications aside from acetaminophen (Tylenol) for her fever. Her mother states that her daughter is usually active and had been playing soccer regularly until she became ill; the patient has been fairly healthy aside from occasional cold sores. Past medical history is significant for frequent ear infections as a toddler that were treated with tympanostomy tube placement at age 2. Brain imaging reveals edema of the temporal lobe. What is the likely diagnosis? -patho -presentation -diagnostics -treatment
Smith fracture MOI: FOOSH + flexed wrist presentation: gross deformity of wrist (palmer/down displacement of wrist compared to hand) -swollen, painful -reduced ROM xray: volar displacement and angulation of the distal radius (garden spade abnormality)
A 16-year-old boy is brought to the Emergency Department by his mother following a fall from a skateboard on an outstretched left hand with his wrist flexed. He is in severe pain and is holding his left arm against his body for support. There is a palmer deformity of the distal radius compared to the hand with edema and ecchymosis of the wrist. What is the likely diagnosis? -MOI -presentation -xray
Colles Fracture MOI: FOOSH on extended wrist presentation: gross deformity of wrist (dorsal displacement of wrist compared to hand) -swollen, painful -reduced ROM xrays: 3 views of wrist AP, lateral + olbique -dorsal displacement + angulation of distal radius (up) tx: reduction, long arm or sugar tong splint
A 16-year-old boy is brought to the Emergency Department by his mother following a fall from a skateboard onto his left outstretched hand (non-dominant) 30 minutes ago. He screamed on the fall and continues to hold his left forearm near his wrist with his right hand. He holds the left arm against his body for support. There is a "dinner fork" deformity, with edema and ecchymosis at the wrist. He is neurovascularly intact, and the skin is closed. What is the likely diagnosis? -MOI -presentation -xrays -treatment
acute appendicitis patho: Caused by fecalith (obstruction) presentation: -pain starts vague, central, poorly localized → pain at McBurney's point in RLQ -malaise -anorexia, nausea -bowel irregularity (diarrhea or constipation) -progression → flank pain, dysuria, RLQ pain diagnostics: -UA + Pregnancy test b/c dysuria -CBC: increases -CRP: increases -Compression U/S: children + pregnant females -CT of pelvis + abdomen: thickened appendix, appendicolith, + inflammation treatment: NPO, IVF, antiemetics, analgesia -appendectomy
A 16-year-old boy is gripped with sudden abdominal pain at school and starts throwing up. The pain is initially central around the umbilicus without any radiation then becomes more vaguely centered in the right lower abdomen. He is brought to the ER where his vitals are: temperature 101.1 F, BP 110/72 mm Hg, pulse 106/min, and SPO2 92%. Chest is clear and abdominal exam reveals tenderness in the right lower quadrant with rebound and hypoactive bowel sounds. Labs show HB of 14g/dl, WBC 15500, and platelets 420,000/uL. Obstructive series shows non specific bowel dilatation. CT scan of the abdomen is ordered. What is the likely diagnosis? -patho -presentation -diagnostics -treatment
meningococcal meningitis presentation: -acute onset (1-2 days) developing confusion, delirium, seizures and coma -petechial rash (hallmark of N. meningitidis) including mucus membranes -high fever, N/V -headache -abdominal and extremity pain lumbar puncture: -cloudy/purulent -elevated pressure, protein, WBC (neutrophils) -decreased glucose treatment: Penicillin G
A 16-year-old boy presents with acute onset of stiff neck, fever, headache, and vomiting. On exam, he appears lethargic, has limited range of motion of his neck, and a petechial rash is noted. Fever is 103°F. Lumbar puncture shows CSF with elevated opening pressure, low glucose, high protein and high WBC. What is the likely diagnosis? -patho -presentation -diagnostics -treatment
Cranial CT Scan In a patient with neurological symptoms after a head injury, head imaging is advised. Cranial CT is considered the imaging modality of choice for traumatic brain injury after a thorough history, physical, and neurologic examination.Compared to MRI scans, CT scans are more sensitive for hyperacute and acute intracranial hemorrhage and are more quickly, easily, and cheaply performed. In the absence of available CT scanning (diffuse axonal injury)
A 16-year-old boy sustained a head injury during wrestling practice. During a practice match, the boy's head struck the concrete floor, and he then received a secondary blow to the head from a nearby wrestler. He briefly lost consciousness and then was dazed, confused, and slow to respond to questions for a few minutes. His vision became briefly blurry and double. His balance was initially unstable but improved with time; however, he developed lethargy and a headache. He did not have any nausea or vomiting. His mother was instructed to have him checked for a head injury. On exam about an hour later, he appears tired but alert and oriented. He's able to recount the events leading to his injury, but he's somewhat unsure of what happened right after. His vitals are stable, and visual acuity is 20/40 in both eyes; head and neurologic exam are normal, including equal and reactive pupils and cranial nerves. His very anxious mother says he suffered a concussion 2 seasons ago while playing football and had to sit out of practice for 2 weeks. What is the preferred management of this patient?
hypokalemia presentation: -muscle weakness* -polyuria and polydipsia* -tachyarrythmias -palpations -decreased DTR -constipation -dyspnea EKG: -tachyarrhythmias -prominent U waves -prolonged QT interval -ST depression treatment -PO KCL -IV KCL if need it to work quickly
A 17 year old female is presenting to the ED due to extreme fatigue, malaise and muscle cramping. She has had excessive thirst and urination today. She admits to have bulimia and has thrown up 5 times this week. EKG shows prolonged QT interval, tachycardia and U waves. What electrolyte abnormality does she like have? -presentation -EKG -treatment
concussion patho: acceleration-deceleration movement of head that causes stretching + shearing of axons presentation: -dizziness -amnesia (retrograde or antegrade) -headache -N/V -visual disturbances -imbalance diagnostics: -CT without contrast (check for bleeding) treatment: head and neck should be immobilized until imaging. Patient should rest until symptoms resolve. and observation (in or outpt) for 24 hours is recommended
A 17 year old football player comes into the ER after a helmet to helmet injury occurred two hours ago. He reports that he felt dizzy afterwards, now has nausea, trouble walking, double vision, confusion, and headache. What is the likely diagnosis? -patho -presentaiton -diagnostics -treatment
otitis externa patho: cerumen hydrophobic protective barrier to underlying skin + acidic pH inhibits bacterial + fungal growth -mechanical trauma to ear canal breaks down barrier (S. aureus) -humidity + moisture + heat also contribute (Swimmers ear, pseudomonas) presentation: -ear canal inflammation: otalgia, pruritus, jaw pain -ear pressure or fullness -hearing loss (sometimes) -auricular discharge PE: pain w/ manipulation of the ear canal (resistant to insert otoscope) or tragus or pinna. -regional LAD -debris present in ear canal treatment: -1st line topical antibiotics: FQ ear drops -combination w/ steroids speeds up recovery -remove debris from canal
A 17-year-old boy presents with a 1-day history of an extremely painful and pruritic left ear after returning from a weekend trip to the beach. Physical examination of the left ear reveals an erythematous external canal without clear visualization of the tympanic membrane. The patient grimaces and expresses a painful sensation when the left pinna is manipulated. Physical examination of the right ear is benign. What is the likely diagnosis? -patho -presentation -PE -treatment
AC joint separation presentation: -tenderness to palpation + pain over the AC joint -swelling, bruising along clavicle -pain with abduction PE: positive crossover test diagnostic: bilateral AP treatment: sling or analgesia
A 17-year-old male high school football player presents after being tackled and slammed onto his right dominant shoulder forcefully 2 hours ago during a game. He had immediate pain, but was able to continue punting. He has full active and passive range of motion, but some pain (4/10) with abduction. There is no obvious deformity and the skin over the shoulder is intact and not tented. There is tenderness present over the AC joint. What is the likely diagnosis? -presentation -grading -PE -diagnostic
testicular torsion causes: cryptorchidism, post-epididymitis, iatrogenic, hydrocele (malpositioning), sports, trauma patho: twisting of the spermatic cord → obstruction of testicular artery/veins → decreased arterial blood flow causes ACUTE sharp pain → ischemia UROLOGIC EMERGENCY presentation: acute unilateral severe profound pain, swelling, stress response (N/V, low grade fever) diagnostics: A/V doppler ultrasound of scrotum shows decreased arterial blood flow to testes treatment: -requires surgical intervention within 6 hours . Detorsion of spermatic cord - > 24 hours likely need orchiectomy :(
A 17-year-old male is seen for the onset of excruciating right testicular pain 3 hours ago. There has been no fever, testicular trauma, or prior episodes of testicular pain. Testicle could not be palpated due to edema + pain. Elevation of the scrotum did not diminish the pain. He had lacrosse practice earlier in the day. What is the likely diagnosis? -causes -patho -presentation -diagnostics -treatment
tear of the ulnar collateral ligament of the thumb aka Gamerkeepers or Skiers thumb patho: forced abduction and hyperextension of the thumb (radial deviation) leads to rupture of ulnar collateral ligament tear presentation: -pain, swelling, and tenderness to the ulnar side of the 1st MCP joint PE: weak pincher grip, valgus stress test + tx: thumb spicca splint or surgical repair
A 20-year-old man presents after taking a fall during a skiing trip 2 days ago. The chief complaint is pain in the right thumb. On examination, you note gross laxity in the first metacarpophalangeal joint when moving the right thumb into abduction. An X-ray reveals no fracture. What is the likely diagnosis? -patho -presentation -tx
mastoiditis patho: infection of the mastoid air cells of the temporal bone; usually a complication of acute otitis media presentation: <2 y/o -fever -acute otitis media is almost always present -post-auricular pain, edema and erythema -protrusion of the auricle (late finding) -ear canal narrowed (pressure) diagnostics: -1st line CT scan w/ contrast. Late mastoiditis coalesce of mastoid air cells seen with bone destruction -early findings look same as AOM w/ opacification but not destruction of mastoid air cells treatment: Admit + Consult w/ ENT -IV Abx (ceftriaxone) if no improvement within 24-48 hours tympanostomy tube insert w/ cultures of fluid that drains
A 18 month old presents with a fever, uncontrolled fussiness, and continues to tug on her left ear. Physical exam shows the presence of a erythematous and bulging tympanic membrane. You also notice the child cringes away in pain when you palpate the left post-auricular area and that it too is erythematous. What is the likely diagnosis? -patho -presentation -diagnostics (early vs. late finding) -treatment
panic attack presentation: Sudden, abrupt, discrete episode of intense fear or discomfort that usually peaks within 10 minutes (rarely lasts more than 1 hour) -dizziness -trembling -sweating -SOB -chest pain -chills or hot flashes -palpitations, increased HR -nausea diagnostic: HAVE to r/o life threatening conditions such as MI, thyrotoxicosis. Presentation must include at least 4 of the above sx of sympathetic overdrive + sense of impending doom/dread tx: benzos (alprazolam)
A 19-year-old college student presents with pounding heart, sweating, coldness, and difficulty in breathing. He said that he was studying for an important exam and drinking lots of caffeine, and he then started to feel like he was going to die of an attack. Examination, ECG, and tests do not demonstrate any pathology. What is the likely diagnosis?
(B) The presence of headache associated with papilledema raises the concern for a brain tumor. The MRI excluded a mass lesion, raising a strong suspicion of pseudotumor cerebri aka benign intracranial hypertension risk factors: -obese women of childbearing age -meds: steroid withdrawal, growth hormone, OCP, vitamin A toxicity presentation: -pulsatile headache that worsens with straining or changes in posture -N/V/tinnitus -visual changes like double vision or vision loss PE: bilateral papilledema diagnostics: -CT r/o mass -lumbar puncture: elevated CSF pressure >25 cmH2O treatment: -weight loss -acetazolamide (decreases CSF production)
A 19-year-old woman presents to the emergency department complaining of headache. The headaches are generalized and increasing in intensity. They have not responded to over-the-counter (OTC) medications. She complains of approximately 1 week of blurred vision, intermittent diplopia, and vague dizziness. Her medical history includes obesity and acne. She takes accutane and oral contraceptives. She is found to have bilateral papilledema, visual acuity of 20/30 on physical examination, and a normal MRI of the brain. The next most appropriate step would be (A) CT scan of the head (B) Lumbar puncture (C) Therapy with high-dose prednisone (D) Stat cerebral arteriogram (E) Reassurance and follow-up in the office in 6 months -risk factors -presentation/PE -diagnostics -treatment
pyelonephritis more common in diabetics and elderly women; E. Coli presentation: same as cystitis w/ LUT sx w/ upper UT sx -CVAT + -fever -N/V -fatigue UA: pyuria, leukocyte esterase, nitrites, hematuria, cloudy urine -WBC casts treatment: -Outpatient: Cipro/Levofloxin, Bactrim -In patient: IV FQ, 3rd/4th cephalosporins
A 19-year-old woman presents with a 4-day history of fever accompanied by chills, nausea, vomiting, back pain & dysuria. On PE there is mild left flank tenderness and tenderness in the left costovertebral angle. What is the likely diagnosis? -presentation -UA -treatment
ANY neonate (0-60 days old) needs to be admitted, full sepsis work up & treated with empiric antibiotic therapy. This is due to the fact that this age group is at risk for serious bacterial infections + sepsis full sepsis work up: -CBC -blood culture -UA -urine culture -CSF culture (LP) -CXR <28 days old: ampicillin + gentamicin (synergistic effect) 29-90 days old: ceftriaxone (add vanco if concerned about resistance S. penumo)
A 2 week old presents to the ER with a 101 F for the past 24 hours. How should this patient be worked up + treated?
Croup aka Laryngotracheobronchitis patho: parainfluenza MCC → edema in the subglottic space accounts for upper airway obstruction presentation: -inspiratory stridor, wheezing -hoarse/barking cough -fever -tachypnea/tachycardia -severe: cyanosis, retractions, stridor @ rest xray: narrowing trachea in the subglottic region (steeple sign) treatment: mild: no stridor at rest + barking cough supportive therapy + oral hydration, can tx w/ oral dexamethasone severe: stridor at rest -supplemental O2 (>90 SaO2 goal) -nebulized racemic epinephrine: rapid onset of action 10-30 minutes + reduces need for intubation -IM dexamethasone
A 2 year old comes into the office with a barking cough. Their mom says that it has been going on for the past few days and has not gotten better. You notice that the child is using accessory muscles to breath and hear stridor on inspiration. Chest x-ray shows a steeple sign. What is the likely diagnosis? -patho -presentation -treatment
nursemaids elbow aka radial head subluxation patho: pulling injury causes the head of the radius to move under the annular ligament MOI: pulling injury (i.e. parent grabs arm of child to stop them from falling) presentation: 1-4 y/o =Arm is held closed to body in a slightly flexed and pronated position clinical dx treatment: closed reduction via -supination-flexion (most commonly used) -hyperpronation (pressure to radial head + hyper-pronate the forearm)
A 2 year old was about to fall off the curb when her mom pulled on her wrist to prevent her from falling. The child now is holding the arm close to the body in a flexed and pronated position. She cries when you try to move the arm from the position. What is the likely diagnosis? -patho -MOI -presentation -treatment
talus fracture **high force injury
A 20 y/o student comes into the ED after falling while snowboarding. He is unable to bear weight on his R foot. What type of fracture did he sustain?
tension pneumothorax patho: Trauma to chest wall generates one way valve → lets air in cannot get out of pleural space presentation: -tracheal deviation (AWAY from side of injury) -diminished breath sounds -hyperresonance (percussion) -hypotension -jugular vein distension -tachycardia CXR: atelectasis, mediastinal shift tx: needle decompression of chest 2nd ICS in the midclavicular line → f/up w/ chest tube insertion
A 20 year old patient is brought in by EMS after sustaining injuries following a motor vehicle accident. He is having difficulty speaking full as he gasps for air. On PE you see the trachea is deviated to the right, he is hypotensive, tachycardic and has jugular vein distension. Breath sounds are diminished. What is the likely diagnosis & treatment? -patho -diagnostics
opioids -patient will feel like crap, but withdrawal is not life threatening presentation: -lacrimation -flu like sx: myalgia, rhinorrhea -N/V/D -tachycardia -HTN -piloerections/goose bumps treatment: symptom management -clonidine: decreases SNS sx -loperamide: diarrhea -NSAIDS severe tx in detox w/ methadone or suboxone
A 20-year-old man presents because he is feeling nervous, cannot sleep at night, has a loss of appetite, a low-grade fever, a runny nose, and stomach cramps. On physical exam, the hair on his arms is standing on end, he is sweating, and his pupils are dilated. He states that he is accustomed to taking a certain drug, but he has not been able to obtain it for the last 72 hours. What drug is he likely withdrawing from? -presentation -treatment
(E) Magnetic resonance imaging; Multiple sclerosis (MS) can present in a myriad of ways but the most common presenting signs are optic neuritis and transverse myelitis. MRI is the test of choice to support the clinical diagnosis of MS. optic neuritis: -unilateral loss of central vision (peripheral is preserved) -pain behind the eye (exacerbated by movement) -color vision loss -fundoscopic: optic disc pallor + atrophy tx: urgent referral to opthmology + neurology -IV methylprednisone optic neuritis can also be caused by ethambutol
A 21-year-old female presents with blurred vision and pain with movement in one eye. In addition, she is concerned about an episode of numbness and tingling she had in both feet 6 weeks ago. Review of systems is positive for constipation, fatigue, and nocturnal leg cramps. She denies extremity weakness or urinary frequency. Which of the following is the test of choice in evaluation of this patient? (A) Serum antimyelin antibodies (B) Cerebrospinal fluid analysis (C) Computed tomography (D) Evoked potentials (E) Magnetic resonance imaging
flail chest Flail chest occurs when two or more ribs are segmentally fractured. -On physical examination, they will have paradoxical respirations where the affected segment rises with expiration and falls with inspiration. When respiratory failure occurs it is usually in association with an underlying pulmonary contusion. -Management typically includes tube thoracostomy for pneumothorax or hemothorax, pain management, pulmonary toileting, and mechanical respiration. Due to altered pulmonary parenchymal fluid dynamics at the point of pulmonary contusion fluid restriction is warranted when possible.
A 22-year-old male is brought to the ED after having been involved in a head-on motor vehicle accident as an unrestrained passenger. A chest radiograph performed in the ED reveals multiple segmental rib fractures of ribs 3 to 7 on the left side. His respiratory rate is 26/min (labored) with discordant motion on the left side. The most likely diagnosis is: (A) Cardiac tamponade (B) Flail chest (C) Pulmonary contusion (D) Tension pneumothorax
community acquired pneumonia presentation: -cough, dyspnea, pleuritic chest pain -fever -sputum production -hypoxia -rales, cracked + rhonchi -increased tactile fremitus -dullness to percussion CXR: infiltrates + lobular consolidation treatment uncomplicated outpatient: azithromycin + doxycycline outpatient + comorbidities: levofloxin, augmentin + azithro inpatient (not ICU): IV levofloxicin, ceftriaxone + azithro inpatient + ICU: ceftriaxone + levofloxacin
A 22-year-old man presents with a 1-week history of fever, chills, dry or mildly productive cough, and chest wall discomfort. He is on no medications, but he has a history of mild asthma for which he does not need chronic therapy.He decided to seek medical attention today because he has not been improving. On examination, vitals include a temperature of 102.6°F, BP of 126/76 mm Hg, pulse of 82 beats/min, and RR of 20/min. Lung exam reveals coarse rhonchi throughout the lung fields, with rales in the right lower lobe and egophony demonstrated over the right lower lobe. The results of a PA and lateral chest X-ray reveal right lower lobe infiltration. What is the likely diagnosis? -presentation -diagnostics -treatment (outpatient vs. inpatient)
pneumothorax patho: Abnormal collection of air enters pleural space → partial lung collapse presentation: Acute onset of dyspnea -pleuritic chest pain -cough -hypotension -hypoxia -asymmetrical chest expansion diagnostics: chest x-ray will show area of lung collapse -coags (chest tube placement) tx: if small (<3 cm chest wall + lung) + first episode observation + supplemental O2 in ED
A 22-year-old man presents with a sudden onset of shortness of breath and right-sided chest pain. Symptoms began yesterday, and he felt well prior to the onset of symptoms. He denies fever, hemoptysis, and upper respiratory symptoms. He is a 1 pack-per-day smoker; otherwise, he has a noncontributory past medical history. On physical exam, the patient is in mild respiratory distress, with a slightly elevated heart rate and respiratory rate. He is normotensive. His trachea appears deviated to the left. On pulmonary exam, breath sounds are diminished on the right. Hyperresonance is noted on percussion of the right chest compared to the left. Other than tachycardia, his cardiovascular exam is normal. What is the likely diagnosis?
anterior cruciate ligament tear patho: non contact sport, pivot injury where person stopped or changed direction is most common cause presentation: hear a "pop" then the knee swells, knee feels unstable Special tests: -Lachman's test (most sensitive) -pivot shift test -anterior drawer MRI diagnostic of choice treatment: brace + PT, surgery will be delayed 3-4 weeks to prevent arthrofibrosis
A 22-year-old woman presents for evaluation of knee pain following an injury during a basketball game where she landed wrong after a jump and heard a pop. The patient appears in moderate distress and is unable to fully bear weight as she moves to the exam table. Her physical exam reveals a swollen, tender left knee. She is tender at the lateral femoral condyle, lateral tibial plateau, and tibiofemoral joint line. Lachman and anterior drawer tests are positive. Posterior drawer, Apley grind, patellar grind, and McMurray tests are negative. There are no sag or apprehension signs. Valgus and varus stress tests are normal. An X-ray is obtained and shows no fractures. What is the likely diagnosis? -patho -presentation -PE tests -MRI -treatment
prophylactic antibiotic drops: Ciprofloxacin or bacitracin-polymyxin patho: damage to the cornea. Causes include: contact lenses, foreign bodies, trauma presentation: Painful red eye -blurred vision -watering eye -photophobia -foreign body sensation diagnostics: -Fluorescein stain diagnosis corneal abrasion; collects in abrasions or ulcers highlighting them (devoid of epithelium in areas of trauma) -Slit lamp examination -do NOT patch eye
A 22-year-old woman presents with a 1-day history of foreign body sensation in her right eye. She woke up with pain in the right eye, and she immediately had trouble opening her eye. She wears soft contact lenses and does not remember how long the last pair was in for. She removed her contact lenses the night before the pain started. There was no trauma. Visual acuity was 20/40 O.U. without corrective lenses, and extraocular movements were within normal limits. With fluorescein stain, a defect is noted; it is round and found at the center of the cornea. No foreign bodies are noted. What treatment should be initiated? -patho -presentation -diagnostics
opioid overdose/intoxication acute: IV naloxone aka Narcan: opioid antagonist Long term dependence -Methadone: long acting opioid receptor agonist -Suboxone: buprenorphine + naloxone. Buprenorphine is a partial opioid agonist + naloxone prevents intoxication from IV injection -Naltrexone: competitive opioid antagonist, precipitates withdrawal if used within 7 days of heroin use
A 23 year old man with no known significant past medical history is brought into the ER in an unconscious state. His physical exam reveals slow and shallow respirations, bradycardia, hypotension, cyanosis, and miosis of both pupils. Additionally, he is comatose, has diminished bowel sounds and distension with dullness to percussion over the suprapubic abdominal area, and has flaccid musculature. He was seen by friends at a party carrying a prescription bottle. What is the likely diagnosis? -acute vs. long term treatment
(C) Emergent referral to otolaryngology (ENT) provider
A 23-year-old male patient states that he was injured during an altercation about 5 to 6 days ago. He was hit in the face during the disagreement. He states that the nasal swelling that he had experienced has decreased, but he has had increasingly more difficulty breathing through his nose. Upon examination, you note soft fluctuant swelling of the septum bilaterally. There is no tenderness to palpation. What is the most appropriate intervention? (A) Intranasal steroids for 2 to 3 weeks to reduce nasal inflammation (B) Broad-spectrum antibiotic to prevent abscess formation (C) Emergent referral to otolaryngology (ENT) provider (D) CT scan to rule out complex nasal fracture (E) Needle aspiration of the fluctuant area with cultures
endometriosis patho: presence of endometrial tissue outside of the endometrial cavity in uterus. MC site = ovaries presentation: women of reproductive age -dysmenorrhea -cyclic perimenstrual pain (1-2 days before not relieved w/ NSAIDS) -dyspareunia -infertility -chronic pelvic pain -dyschezia (painful defecation) PE: possible findings -uterosarcral nodularity -fixed or retroverted uterus -cervical motion MAY be present* (not only in PID!) -tender, fixed, adnexal mass diagnosis: direct visualization w/ laparoscopy+ biopsy -may see chocolate cysts treatment: goals suppression + atrophy of endometrial tissue -OCP -danazol (suppresses LH + FSH) only for 6 months d/t bone loss -surgical: ablation, excision, cauterization -abdominal hysterectomy + bilateral salpingo-oopherectomy definitive tx
A 23-year-old woman presents with a history of chronic pelvic pain. She attained menarche at 13, and she has had regular periods. She has been experiencing severe pain during menses; the pain has been increasing in frequency. She has never had inter-menstrual bleeding, and she regularly uses barrier contraception. However, she has refrained from intercourse for over 6 months due to the pain it causes her. Her vital signs and general physical examination appear to be normal. A vaginal exam reveals cervical motion tenderness, with reduced mobility and nodularity of the utero-sacral ligaments. What is the likely diagnosis? -patho -presentation -PE -diagnosis -treatment (pharm + surgical)
The correct answer is epinephrine, as this is the drug of choice in anaphylactic reactions. The definite indications are hypotension, bronchospasm, and urticaria. Epinephrine has significant beta-agonistic effects that cause bronchodilation, and alpha-agonistic effects that can reverse systemic hypotension. IM route is preferred over SC, as the time for maximum serum concentration is shorter by IM route.
A 23-year-old woman presents with an anaphylactic reaction after being stung by several bees. She complains of wheezing and shortness of breath. On examination, the patient is in acute distress. BP is 98/56 mm Hg, PR 110/min, RR 28/min, and temperature 98.7°F. She is immediately treated with supplemental oxygen. Question In treating this condition further, what drug is required most urgently?
endometritis patho: infection of the uterine endometrium. Preceded by disruption of the intrauterine cavity such as: IUD, C-section, vaginal deliveries, dilation + evacuation (abortion) presentation: -fever -abdominal pain -foul smelling + purulent vaginal discharge PE: uterine tenderness on bimanual exam labs: CBC shows leukocytosis treatment: -severe: IV clinda + gentamicin -chronic: doxycycline
A 24 year old G3P3 female presents 3 days following a C-section with a fever and foul smelling vaginal discharge. PE reveals uterine tenderness and lab results show elevated WBC count. What is the likely diagnosis? -patho -presentation -PE -diagnostics -treatment (severe, chronic)
costochondritis patho: Pain associated with inflammation of the costochondral junctions (cartilage attaches ribs to sternum). Can follow lifting heavy objects, URI, or chronic coughing. presentation: -pain worse with movement of deep breaths -reproducible pain on exam (chest wall area) treatment: reassurance + NSAIDS Tietze syndrome: visible enlargement of costochondral area known as "slipping rib" syndrome
A 24 year old patient presents to the ED with an acute onset of chest pain. He recently moved out of his apartment and was lifting heavy furniture two days prior. Pain worsens with movement and deep breaths. The area left to the sternal border is tender to palpation. EKG shows normal sinus rhythm and chest x-ray is normal. What is the likely diagnosis? -patho -presentation -treatment
left epidural hematoma cause: sequalae of a head injury (MVA, TBI, fall) usually in younger individuals patho: usually arterial hemorrhage between the skull and dura mater. Usually middle meningeal artery is the culprit as such can expand quickly and has a high risk of death presentation: -more acute presentation than subdural - brief loss of consciousness followed by lucid interval -N/V, headache -contralateral unilateral weakness -ipsilateral pupil fixed dilation diagnostics: -CT→ goose egg shape that does NOT cross suture lines and has convex shape (lens shape) treatment: SURGICAL EMERGENCY → hematoma evacuation or craniotomy
A 24 year old patient presents to the ER after sustaining a head injury following jumping off the roof at a party 30 minutes ago. Patient had a brief loss of consciousness but now appears to be alert and oriented. PE shows right sided hemiparesis and his left pupil is fixed and dilated. What is the likely diagnosis/finding on CT scan? -cause -patho -presentation -diagnostics -treatment
leiomyoma patho: benign uterine smooth muscle tumors (myometrium) risk factors: African American women, child bearing age (regress during menopause) presentation: most are asymptomatic -abnormal uterine bleeding -pressure related sx like fullness, pressure -constipation -lower back pain PE: non-tender irregular masses, enlarged uterus diagnostic: hypoechogenic areas among normal myometrial material treatment: majority don't need treatment. -NSAIDS for pain -OCP abnormal uterine bleeding
A 30 year old African American G0P0 with no significant past medical history presents with chronic pelvic pain. She denies any other symptoms. Physical exam reveals a mobile uterus that is non-tender with multiple discrete nodules. A transvaginal ultrasound shows hypoechoic masses within the uterine myometrium. What is the likely diagnosis? -patho -risk factors -presentation -PE -diagnostics -treatment
Guillain Barre Syndrome paho: acquired autoimmune demyelinating polyradiculopathy of the peripheral nervous system. Autoantibodies attack the myelin sheath of peripheral nerves after an infection, immunizations or post-surgery causes: C. jejuni most common, GI or respiratory infections, immunizations or post-surgery presentation: -symmetrical ascending weaknesses that evolve over hours-days accompanied by sensory changes (parasethesias, pain) -LE affected first -associated sx: pain in neck, shoulder, back PE: -LMN signs: decreased DTRS, weakness and flaccid paralysis -sensory deficits: CN-VII palsy -autonomic dysfunction: arrhythmias, tachy, breathing difficulties, blood pressure fluctuations (hypo or hyper) Diagnostics: -EMG: decreased sensory and motor nerve conduction velocities -CSF: all normal except elevated protein -check pulmonary functions! Respiratory muscles could be effected and pt might need to be intubated tx: -1st line: plasmapheresis (remove circulating antibodies) and IV immunglobulin -prednisone NOT indicated in management of GBS* (according to PPP)
A 24 year old patient presents with a 3 day history of worsening bilateral weakness that began in her feet and is now progressing towards her knees. She reports also experiencing parasthesia in her legs as well as pain in her neck. On PE, she is tachycardic, hypotensive and her DTRs are decreased. She had food poisoning from Campylobacter jejuni 1 week ago. What is the likely diagnosis? -patho -cause -presentation -PE -diagnostics -treatment
Thumb spica splint and referral to an orthopedic surgeon - splint: 6-8 weeks - return to sport in 11 weeks if a patient with a suspected scaphoid fracture has radiographs that are read as "normal'", but clinically there is a fracture (tenderness at anatomical snuffbox) place the patient in a long arm thumb spica splint and refer the patient to an orthopedic surgeon. Complication can be avascular necrosis.
A 24-year-old man presents with radial-sided wrist pain following a fall on his outstretched hand 3 days ago. He did not seek immediate medical attention due to a lack of swelling. Despite self-treatment with ice and analgesics his wrist still hurts, especially with ulnar deviation and while trying to open a jar or grip the steering wheel of his car. On examination, his tenderness seems to be in the anatomic snuffbox of the wrist. Radiographs are normal in all views. What treatment should be initiated at this time?
jones fracture seen in jumpers + sprinters presentation: midfoot and 5th metatarsal pain especially with weight bearing MOI: ankle sprains OR heel off ground while foot is planted xray: transverse fracture at the metaphysis-diaphysis junction (proximal diaphysis) tx: refer to ortho often requires pinning; posterior short leg splint and NON weight bearing status
A 25 year old college sprinter comes in because he is having significant pain in the midfoot that worsens when he bears weight on it for the past day. He was training and when he took off into his sprint he felt a severe pain here. On PE the pain is specific to the 5th metatarsal. X-ray shows a transverse fracture near at the proximal diaphysis of the 5th metatarsal. What is the likely diagnosis? -presentation -MOI -diagnostic of choice -tx
Morton's Neuroma patho: compressive neuropathy of interdigital nerve due to repetitive microtrauma → proliferation of nerve risk factors: wearing tight fitting or high heeled shoes, flat feet presentation: burning pain that worsens with weight bearing, usually between the 2 +3 or 3+4 distal metatarsals diagnostics: -Mulder Sign: clicking sensation when palpating/squeezing involved interspace -tinnels Treatment: -conservative: NSAIDS, broad toe shoes -refractory: steroid injections or surgical resection
A 25 year old female patient is presenting with a 1 month history of progressively worsening burning pain in her foot. She said it is localized near her 2nd and 3rd toe and radiates towards her body. On PE you are able to reproduce the pain with palpation. She reports that she wears high heels everyday to work. What is the likely diagnosis? -patho -risk factors -presentaiton -diagnostics -treatment
bacterial vaginosis aka gardnerella 40-50% of all vaginitis! diagnostics: -wet prep: epthelial cells covered by G. vaginalis (clue cells) -KOH to wet prep can enhance fish odor (whiff test) treatment: metronidazole
A 25 year old female presents with grey thin vaginal discharge with a distinctly fishy odor. Wet mount shows clue cells. What is the likely diagnosis & treatment? -diagnostics
aseptic meningitis patho: caused by enteroviruses like coxsackievirus and echovirus presentation: same as bacterial meningitis but oftentimes more mild and gradual -can lead to to inflammation of parameningeal region (sinusitis, otitis) diagnostics: -LP+CSF: normal glucose, normal or mildly elevated protein and pressure, elevated WBC with lymphocyte dominant tx: supportive tx
A 25 year old patient comes in with 5 day history of worsening headache, neck stiffness, nausea and photophobia. She has now felt sinus pressure the past day. Denies any dysphasia, memory loss, or paraesthesia. Lumbar puncture shows normal glucose, opening pressure and protein levels. WBC elevated with lymphocytes present. What is the likely diagnosis? -patho -presentation -diagnostics -tx
Boxer's fracture patho: direct trauma to a clenched fist (i.e. punch object or person) presentation: -swelling + pain of the hand -pain near 5th MC -bent claw like pinky -limited ROM of 4th + 5th fingers treatment: ulnar gutter splint w/ 60 degrees of flexion
A 25 year old patient comes into the ED with pain and swelling of his right hand. He hit a cement wall out of anger and cannot move his 4th and 5th fingers well. X-ray shows a fracture of the 5th metacarpal bone. What is the likely diagnosis? -patho -presentation -treatment
serotonin syndrome causes: initiation, change or additive sertonergic drugs like SSRIs, SNRIs, TCA, MAO-inhibitors, triptans, dextromethorphan Hunter's criteria: presence of 3 of the following sx -mental status change -agitation -autonomic instability: hyperthermia, tachy, BP changes, diaphoresis -neuromuscular hyperactivity: myoclonus, hyperreflexia, tremors -weakness -mydriasis -N/V/D treatment: d/c offending agents -supportive care: supplemental O2, IV fluids benzodiazepines: reduce agitation, hyperthermia, and corrects HR/BP
A 25 year old patient comes to the ER with a 16 hour history of nausea and vomiting. Vitals show that the patient has a temperature of 104 degrees, BP 140/95, and heart rate is 110 bpm. You see that he diaphoretic, tremors, and mydriasis. Neuro exam shows myoclonus. He is confused, uncoordinated, and agitation. He is finally able to tell you that he took 500 mg of Sertraline earlier because he did not want to "wait" to get to that dosage and wanted to just "feel better already. What is the likely diagnosis? -causes -Hunter's criteria -treatment
incarcerated: painful, enlargement of an irreducible hernia. May or may not become strangulated strangulated: irreducible hernia with compromised blood supply, severe painful bowel movement BOTH are emergencies and require surgical repair
Describe the differences between incarcerated and strangulated hernia
mastitis patho: regional infection of the breast. Seen in lactating women + caused by the patients skin flora or oral flora of the infant. Organism enter through erosion or crack in nipple S. Aureus is MCC (infants nose + throat) presentation: -fever, chills, malaise -localized breast tenderness, breast is red + warm to the touch labs: elevated WBC treatment: Cephalexin (Keflex) or dicloxacillin **continue to breastfeed**
A 25-year-old G1P1 Caucasian woman who recently delivered a healthy male infant presents with breast pain and tenderness on the right; associated symptoms include malaise and fever. The patient has been compliant with all of her prenatal and post-natal obstetrical appointments. No medical complications were incurred during or after the pregnancy. The patient is currently breast feeding. Physical inspection of the right breast shows focal breast erythema and tenderness. What is the likely diagnosis? -patho -presentation -labs -treatment
inevitable - + vaginal bleeding/cramping - cervical os open - POC remain - +/- cardiac activity
A 25-year-old G3P2 woman at eight weeks zero days of gestation presents to the emergency department complaining of pelvic pain and vaginal bleeding. Pelvic examination confirms the uterus as the source of bleeding and reveals a dilated cervix. No products of conception are seen. Pelvic ultrasound shows an embryo with a crown-rump length of 9 millimeters without fetal cardiac activity. What type of abortion classification is this?
cystitis pathogen: E. coli presentation: suprapubic pain, frequency, urgency, dysuria, cloudy urine UA: leuks, nitrites, blood micro: RBC, bacteria treatment: -phenazopyridine (pain) -nitrofurantoin/Bactrim -Recurrence: cipro/levofloxacin
A 25-year-old female patient presents with a 3 day hx of burning sensation on micturition and finds herself going to the bathroom every 15-20 minutes. As the day progresses, she notices an ache in the lower abdominal area. She has no fever, nausea, or diarrhea. What is the likely diagnosis? -pathogen -presentation -UA/micro -treatment
acute interstitial nephritis (intrarenal AKI) causes: -medications: NSAIDS, PPI, allopurinol, methicillin, tetracyclines -infections like pyelonephritis diagnostics: -UA w/ microscopy: isomorphic microhematuria, pyuria, WBC -CBC: eosinophilia -FeNa: HIGH b/c issue is within the kidneys treatment: fix the underlying cause, fluids if necessary
A 25-year-old man has malaise and decreasing urine output 6 weeks after completing a course of tetracycline for acne. He is afebrile and has a normal physical exam. UA w/ microscopy is positive for isomorphic microhematuria, pyuria, and WBC casts. CBC shows elevated blood eosinophils. FeNa is 3%. What is the likely diagnosis? (be specific!) -causes -diagnostics -treatment
Mallory Weiss Tear patho: laceration at the GE junction presentation: Persistent retching or vomiting → upper GI bleed -hematemesis -melena -hematochezia -syncope diagnostic: upper endoscopy treatment: -not actively bleeding: supportive -severe: endoscopic band ligation, balloon tamponade
A 25-year-old man presents with a 3-hour history of retrosternal chest pain. He reports that he has been vomiting since the previous evening and his last vomitus had fresh blood in it. He denies any trauma to the chest. On examination his BP is 90/60 mm Hg and PR 110/min. What is the most likely cause of this patient's chest pain?
acute endocarditis Duke's criteria: -definitive IE: 2 major criteria or 1 major + 3 minor criteria -possible IE: 1 major + 1 minor or 3 minor criteria major criteria -2 blood cultures 12 hours apart + for S. aureus, S. viridans or Strep -echocardiogram showing vegetations -new regurgitant murmur minor criteria: -fever -Osler nodes (raised "ouchy" nodules() -Janeway lesions: painless macules -Roth spots: exudative lesions on retina -splinter hemorrhages on nailbeds -nail clubbing treatment: 4-6 weeks of therapy -empiric: ampicillin PLUS aminoglycoside -IVDU: vanco + aminoglycoside -prosthetic valve: vanco + gentamicin + rifampin
A 25-year-old presents with a fever. She has a history of intravenous drug use and had previously been treated for osteomyelitis. On physical exam, she is febrile, and heart auscultation reveals a new systolic murmur at the lower left sternal border. On physicial exam you notice painless red macules on her fingers. What is the likely diagnosis? -criteria (major + minor) -treatment (empiric)
pneumomediastinum presentation: -pleuritic chest pain -dyspnea -cough -neck pain -odynophagia, dysphagia PE: -subcutaneous emphysema (crepitus in neck) -Hamman sign: crunching, rasping sound synchronous with <3 beat CXR: gas in mediastinum + subQ emphysema (neck) treatment: no treatment is needed because the body will gradually absorb the air
A 27 year old presents to the emergency department with dyspnea, dysphagia and chest pain. He recently recovered from pertussis and was coughing for weeks. He describes the chest pain as sharp it improves when he sits up but worsens when he lies down. On PE, there is a crunching raspind sound in line with the heart beat. Palpation of the neck elicits crepitus. What is the likely diagnosis? -patho -presentation -diagnostics -treatment
meniscal tear patho: twisting or rotating knee when foot is planted (medial most common) presentation: "pop lock + drop" -knee pops -knee locks (cannot fully extend or locks when in extension" -drop: cannot weight bear PE: -joint line tenderness (medial) -McMurray positive (hear pop or click with repeated flexion +extension) -Apley test diagnostic of choice: MRI (it will actually show tear) tx: RICE, brace, surgery
A 27-year-old man presents with knee pain that started while he was playing basketball the previous afternoon. The patient states that as he attempted to pass the ball to a teammate, he twisted his left knee with his left foot still planted on the floor. He describes a popping sensation at the time of the injury, but he could still bear weight on his leg. He does not recall any immediate swelling. The morning of presentation, the patient noted swelling of the knee and pain that was significantly worse than it was the day before. Upon examination of the left knee, there is a limitation in range of motion, the presence of medial joint line tenderness, and an effusion. McMurray's test is positive. What is the likely diagnosis? -patho -presentation -PE -diagnostic of choice -tx
adenomyosis patho: ectopic endometrial tissue grows in other parts of the uterus such as in the myometrium presentation: -dysmenorrhea -menorrhagia -infertility PE: boggy enlarged tender uterus is a classic finding diagnostics: MRI to identify -hysterectomy is the only definitive dx treatment: -mild: analgesics (NSAIDS) -definitive tx: hysterectomy
A 35 year old female with a history of heavy menses and cramps presents with progressively worsening symptoms for the past few months. She has also gained weight. Denies any medication use at this time. On PE the uterus was enlarged, smooth, boggy, and tender. No masses were noted during the bimanual examination. Speculum examination was normal. What is the likely diagnosis? -patho -presentation -PE -diagnostics -treatment
ectopic pregnancy patho: implantation of embryo outside of the uterus (fallopian tube) presentation: -unilateral pelvic or lower abdominal pain -vaginal bleeding -amenorrhea -abdominal or adnexal tenderness -adnexal mass labs: -inappropriate rise in hCG (not doubling every 48 hours) -HCg >1500 but no feterus -<5 mg/mL progesterone = abnormal pregnancy -U/S: adnexal mass with empty uterus, Ring of Fire lesion (hypervascular lesion) treatment: -methotrexate (unless beta HCG > 5000 cardiac activity noted or gestational sac >3.5 cm) -laparoscopic: if MTX contraindicated, emergent situations (rupture)
A 27-year-old woman presents with cramping abdominal pain and vaginal bleeding. Further history reveals amenorrhea for the past 2 menstrual cycles. On examination, she is found to have left lower abdominal tenderness and an adnexal mass. Lab values reveal an elevated Beta HCG level but pelvic ultrasound shows an empty uterine cavity. What is the likely diagnosis? -patho -presentation -labs -treatment (medical vs. surgical)
trichomoniasis presentation: malodorous green discharge, strawberry cervix diagnostics: wet mount shows motile protozoa treatment: metronidazole x 7 days
A 28 year old female presents with a malodorous green vaginal discharge. On speculum examination you see a strawberry cervix. Wet mount shows motile protazoa. What is the likely diagnosis? -presentation -diagnostics -treatment
ruptured ectopic pregnancy presentation: -severe abdominal pain + - can have left shoulder pain (Kehr sign) -hypotension -syncope treatment: laparotomy
A 28-year-old woman presents with abdominal pain and vaginal bleeding. The pain began last night as a dull ache in the right lower quadrant, but this morning it became much more severe. She also complains of dizziness and nausea. She cannot recall when her last menstrual period was, but she says that she began bleeding yesterday and has a light menstrual flow. On examination, the patient is afebrile; pulse is 100/min, BP is 86/60 mm Hg, and RR is 20/min. Physical exam reveals moderate to severe tenderness in the right lower quadrant with rebound. Pelvic exam reveals a small amount of blood at the cervical os with cervical motion tenderness. What is the likely diagnosis? -presentation -treatment
ulcerative colitis patho: continuous lesions, mucosal surface only presentation: -hematochezia -tenesmus -pus-filled diarrhea labs: -barium enema: lead pipe appearance -ANCA + treatment: -prednisone and mesalamine -colectomy
A 28-year-old woman presents with bloody diarrhea for 1 week. After further questioning, she reveals that she had similar attacks in the past that subsided on their own. Her stool specimen is negative for ova and parasites. Stool culture did not grow any pathogens. A sigmoidoscopy is performed. There is friable erythematous mucosa extending from the rectum to the mid-descending colon, with broad-based ulcers in the descending colon. A biopsy taken from the rectum shows diffuse mononuclear inflammatory infiltrates in the lamina propria with crypt abscesses. However, no granulomas are seen. What is the likely diagnosis? -patho -presentation -labs -treatment
Bell's Palsy patho: unilateral CN-VII palsy leading to hemifacial weakness and paralysis. Weakness + paralysis ONLY affects the face. Lower motor neuron issues risk factors: -pregnancy (3rd trimester <28 weeks) -diabetes mellitus -HSV reactivation -post-URI presentation: sudden onset of ipsilateral hyperacusis (ear pain) 24 hours later followed by unilateral faccial paralysis and weakness involving forehead (i.e. cannot raise eyebrow or close eye), lower face (cannot smile, mouth droop) -changes in taste anterior 2/3 tongue, decreased lacrimation/eye irritation due to inability to close eye) Treatment: -no treatment is required as most cases resolve within 1 month supportive includes artificial tears or eyepatches if closing eye not possible -prednisone
A 29 week pregnant patient is presenting with a sudden onset of right sided facial weakness and paralysis. She is unable to life her eyebrow, smile, reports dry eyes, and has taste loss in the front part of her tongue. A day before the facial weakness presented, she had right sided ear pain. Denies any other site of weakness or paralysis, confusion, or ataxia. What is the likely diagnosis? -patho -risk factors -presentation -treatment
tympanic membrane perforation most commonly due to AOM or penetrating trauma presentation: -acute ear pain -hearing loss -patients w/ AOM prior the rupture there is bloody otorrhea with sudden pain relief diagnostics: CT of temporal bones, audiogram do NOT do a pneumatic otoscopy in a patient w/ TM perforation treatment: -heal spontaneously most of the time (if not surgical repair may be needed; wait until child is older when ET is mature) -ototopical ABx and referral to otolaryngoloist 2-3 weeks post rupture to evaluate hearing -AVOID: water and topical aminoglycosides (ototoxic)
A 3 year old was seen a week ago and diagnosed with acute otitis media. Mom was given the option to observe or to treat with antibiotics. Mom wanted to observe but is now frantic because this morning she saw blood all around his left ear where the infection was. The child has been a lot less fussy ever since this morning. What is the likely diagnosis? -cause -presentation -treatment
Acute epiglottitis patho: inflammation of the epiglottis and adjacent supraglottic structures → can lead to life threatening airway obstruction without treatment MCC: HIB bacteria (if not vaccinated) presentation: young child < 5 y/o abrupt onset and rapid progression of upper airway obstruction characteristic of HIB - respiratory distress: tripod or sniffing posture → maximize diameter of obstructed airway -reluctant to lie down -drooling -high fever -stridor -hot potato voice -severe sore throat + dysphagia diagnostics: -visualization of epiglottis confirms dx -soft-tissue lateral xray of neck: thumbprint sign treatment: MEDICAL EMERGENCY! -ABCs -prepare to intubate + admit patient -do NOT agitate child and leave them sitting upright
A 3-year-old child presents with sudden onset of high-grade fever, sore throat, pain during swallowing, and drooling of saliva. There is no history of a cough. The child appears toxic and is dyspneic with inspiratory stridor. The child is sitting upright, leaning forward with chin up and mouth open. Suprasternal and intercostal retractions are present. Chest is clinically clear. Blood count shows polymorphonuclear leukocytosis. Lateral radiograph of upper airway shows "thumb sign". What is the likely diagnosis? -patho -presentation -diagnostics -treatment
acute bronchitis patho: inflammation of tracheobronchial tree presentation: Persistent cough 1-3 weeks no COPD or pneumonia -cough, fever -pleuritic chest pain -resolving ronchi after cough or clearing** diagnostic hallmark tx: Supportive treatment -antipyretics -antitussives -albuterol if wheezing or underlying pulmonary dx -smoking cessation
A 35 year old patient presents to the urgent care because they have had a persistent cough and chest pain for the past 2 weeks. Describes that the chest pain worsens when he lays down but improves if he sits up. Physical exam is normal aside from non-productive cough. Chest x-ray is normal. No pertinent medical history or medications. What is the likely diagnosis? -presentation -treatment
ovarian cysts patho: functional cysts resulting from normal physiological function (follicle grows but does not rupture to release egg) . 2 types are follicular + lutein cysts (follicular MC) presentation: asymptomatic unless large -follicular: pelvic pain, dyspareunia, can lead to ovarian torsion -lutein cysts: amenorrhea, delayed menses, pelvic pain (larger = more firm than follicular) diagnostics: transvaginal u/s -CA 125 if wanting to r/o ovarian cancer treatment: many spontaneously resolve on own -<5 cm: observation -> 5 cm: laparoscopy uncomplicated cyst rupture: NSAIDS + supportive (hemodynamically stable)
A 30 year old female patient presents to the ED with pelvic pain that has been present for 1 week. She has also been experiencing dyspareunia as well. hCG tests come back negative for pregnancy. UA is negative for leukocyte esterase and nitrites. A pelvic ultrasound is ordered shows the presence of 3 cystic masses on the left ovary that are 5cm in size. What is the likely diagnosis? -patho -presentation -diagnostics -treatment
myasthenia gravis risk factors: abnormal thymus gland (hyperplasia or thymoma), HLA-B8 antigen, young women, older men presentation: ocular weakness + generalized weakness -ocular: diplopia and ptosis, pupils are spared/normal -generalized: fluctuating muscle weakness that worsens with repeated muscle use. --Bulbar (oropharyngeal): weakness with prolonged chewing, dysphagia, dysphonia or dysarthria --Respiratory weakness can lead to respiratory failure (myasthenia crisis) diagnosis: -ACh receptor antibodies initial test -chest imaging CXR, CT or MRI to check for thymus hyperplasia or thymoma -EMG: decreased muscle response treatment: -acute: plasmapheresis or IV immunoglobulin -long term: ACh inhibitors like pyridostigmine -thymectomy (even if it is normal) can improve sx and remove source of antibodies -anti-immune meds: steroids, immunosuppressants
A 30 year old female presents to the clinic worsening diplopia, eyelid droop and muscle weakness. She is a personal trainer and halfway throughout the day she feels as if her muscles cannot move anymore. She has also noticed difficulty chewing and swallowing. On PE, her pupils are PERRLA and fundiscopic exam is normal. An ice pack test improves her diplopia. What is the likely diagnosis? -risk factors -presentation -diagnosis -treatment
Labrynthitis cause: post-viral infection patho: inflammation of BOTH vestibular and cochlear portions of CN-VIII presentation: vestibular and cochlear sx -continuous peripheral vertigo -dizziness -N/V -gait issues -unilateral hearing loss & tinnitus -horizontal nystagmus treatment: sx resolve in weeks without tx but can give steroid
A 30 year old female presents with a 1 week history of continuous feeling of dizziness as if the room is spinning, nausea and this morning she vomited. On PE you see horizontal nystagmus. She reports ringing in the right ear and has difficult hearing out of it. She reports she recovered from a viral upper respiratory infection two weeks ago. What is the likely diagnosis? -cause -patho -presentation -treatment
vestibular neuritis patho: inflammation of the vestibular part of CN-VIII cause: usually post-viral infection presentation: -continuous peripheral -vertigo -dizziness -N/V tx: sx resolve in weeks without tx but can give steroids
A 30 year old female presents with a 1 week history of continuous feeling of dizziness as if the room is spinning, nausea and this morning she vomited. On PE you see horizontal nystagmus. She reports she recovered from a viral upper respiratory infection two weeks ago. Denies any hearing loss or tinnitus. What is the likely diagnosis? -patho -cause -presentation -tx
orchitis likely caused by the mumps virus MUMPS presentation: unilateral involvement then contralateral involvement in 1-9 days **orchitis rarely seen w/out epididymitis unless it is caused by MUMPS diagnostics: always order an ultrasound to rule out TT treatment: no abx or antivirals if caused by mumps virus -bacterial: treat same as epididymyitis --<35: ceftriaxone PLUS doxy or azithromycin -- >35: Levofloxacin
A 30 year old male is presenting with an acute onset of testicular pain. On PE you see that the L testicle is swollen and red. Ultrasound rules out testicular torsion. He tells you last week he had a fever and his cheeks looked swollen, he has also never been vaccinated. What is the likely diagnosis? - causes -presentation -diagnostics -treatment
tension headache risk factors: mental stress, sleep deprivation, eye strain, hormones, dehydration or anxiety presentation: -band like pain bilaterally that is steady and achy -patient does NOT have sx of migraine such as N/V, photophobia, auras treatment: -NSAIDS or acetaminophen -if recalcitrant or stressors are persistent: SSRIs or TCA
A 30 year old patient comes in with a 3 day history of headache. She reports that it feels like a tight band across her head and is bilateral. It started in the neck and progressively has moved forward. Denies photophobia, N/V or that it is worsens with activity. She has had a lot of stress in her new job lately. What is the likely diagnosis? -risk factors -presentation -treatment
ovarian torsion- EMERGENCY! patho: complete or partial rotation of the ovary on its ligamentous support → ischemia risk factors: ovarian mass (cyst or malignancy), history of recent vigorous activity presentation: acute onset of unilateral pelvic pain, N/V, low grade fever -PE: pelvic or abdominal tenderness, palpable mass labs: pelvic ultrasound shows enlarged ovary anterior to uterus, string of pearls appearance
A 30 year old patient presents to the ER with acute onset of right sided pelvic pain, nausea and vomiting. It is so painful she is unable to sit up straight or move. On PE you feel a palpable mass on her right lower quadrant near the pelvis. She has a positive history of ovarian cysts. Pelvic ultrasound shows enlarged right ovary with a "string of pearls" appearance. What is the likely diagnosis? -patho -risk factors -presentation -labs
diabetes insipidus (central, MC) causes: -surgery (removal of posterior pituitary adenoma) -head trauma -CNS tumor presentation: -polyuria -polydipsia -nocturia PE: dehydration (poor turgor) labs: -increased serum osmolarity -decreased urine osmolality + specific gravity treatment: desmopressin
A 30 year old patient was recently in a car accident where she sustained significant head trauma. One month later she is presenting with polyuria, polydipsia and nocturia. She feels extremely lethargic. Her blood pressure is 97/60 and she has poor skin turgor. Labs are normal except sodium is 150. What is the likely diagnosis? -causes -presentation -labs -treatment
hypertensive emergency: BP >180/120 with signs of end-organ damage tx → sodium nitroprusside (DOC) urgency: BP >180/120 NO signs of end-organ damage tx → clonidine (DOC) malignant hypertension: diastolic reading 140 associated with papilledema AND encephalopathy or nephropathy tx → hydralazine
A 57 y/o patient comes to the ER due to headache for the past 24 hours and intermittent loss of vision in the right eye. Blood pressure is 190/130. What is the likely diagnosis? -urgency vs. emergency. vs. malignant -treatment for each
Addison's disease patho: adrenal gland dysfunction or destruction (primary adrenal insufficiency) -autoimmune disease** -infiltrative or infections: TB, HIV -vascular: thrombosis or bleed -meds: ketoconazole, rifampin, phenytoin, ACE/ARBs presentation: -hyperpigmentation* -salt craving* -orthostatic hypotension* -weight loss, anorexia -fatigue -amenorrhea -loss of libido -hypoglycemia diagnostics: high ACTH, low cortisol 1. measure 8 amm serum cortisol 2. plasma ACTH after ACTH stimulation test aka Cosyntropin test: high dose ACTH → low or no rise in serum cortisol 3. low aldosterone other labs: -hypoglycemia -increased renin, low aldosterone -low Na, high K, metabolic acidosis treatment: b/c primary affects BOTH mineral and glucocorticoids must replenish both -fludrocortisone -hydrocortisone
A 30 year old patient with rheumatoid arthritis comes in because she has had a lack of a period and 15 pound weight loss for the past 3 months. She has felt weak, lethargic, and has abdominal pain. Describes that she has had an insatiable salt craving lately. PE is positive for patches of hyperpigmentation and hypotension. Denies fever, chills, chest pain or shortness of breath. CMP shows slightly lowered Na and elevated K. What is the likely diagnosis? -patho -presentation -diagnostics -labs -treatment
hip dislocation patho: large force trauma (anterior force), football, skiing **usually a posterior dislocation presentation: -pain -shortened limb -adducted + internally rotate tx: EMERGENCY! secondary injuries to the surrounding blood vessels, nerves, ligaments and tissues. Hip dislocation can cause long-term damage, especially if it's not treated right away. reduce as soon as possible w/ closed reduction
A 30 year old presents to the ED following a car accident. She has significant pain in her right groin and hip pain. Her friend hit the car in front of her and her legs hit the dashboard. On PE you notice that her right leg is shorter than the right and her right hip is internally rotated. She has gross deformity of the hip and limited range of motion. What is the likely diagnosis? -patho -presentation -treatment
posterior cruciate ligament tear patho: Most commonly caused from a bent/flexed knee receiving a anterior directed force → dashboard injury (car accident), football injury presentation: -anterior bruising -large effusion -knee giving out PE: posterior drawer test positive MRI diagnostic tx: RICE, brace, surgery
A 30 year old presents to the ED following a car accident. She has significant pain in her right knee. Her friend hit the car in front of her and her legs hit the dashboard. On PE the knee has anterior bruising and a large effusion. No deformity is notced. She is tender at the lateral femoral condyle, lateral tibial plateau, and tibiofemoral joint line. Posterior drawer test is positive. Lachman and anterior drawer tests are Negative. Posterior drawer, Apley grind, patellar grind, and McMurray tests are negative. There are no sag or apprehension signs. Valgus and varus stress tests are normal. What is the likely diagnosis? -patho -presentation -PE -diagnostic of choice -tx
tibiofemoral dislocation (knee) patho :high energy injury (fall from high height, MVA) -anterior: hyperextension injury -posterior: "dashboard injury" posterior force on tibia with knee flexed presentation: -pain + deformity of the lower extremity -diminished or absent distal pulses may occur treatment: MEDICAL EMERGENCY patients could lose their limb from this type of injury must do reduction ASAP (either open or closed) complications: -popliteal artery tear -peroneal nerve injury
A 30 year old presents to the ED following a car accident. She has significant pain in her right knee. Her friend hit the car in front of her and her legs hit the dashboard. On PE there is excessive pain of the right knee with gross deformity. What is the likely diagnosis? -patho -presentation -treatment -complications
Brown-Sequard Syndrome mechanism of injury: penetrating trauma like tumors, stab wound (hemisection of SC) presentation: -Ipsilateral: motor + vibration/proprioception defects -Contralateral: pain + temp deficits
A 30-year-old man is brought to the emergency department with a gunshot to the right of his mid-back and a large laceration on his left hip. He states that his right side feels heavy and weak. He states that his left hip is not painful. On physical exam, there is a loss of vibratory sensation and proprioception of the right side. His left side has a loss of pain and temperature sensation. The patient appears to have control of his bladder. What type of spinal cord injury does this patient have? -mechanism of injury -presentation
lateral epicondylitis aka tennis elbow patho: repetitive use of the wrist extensors. It is an overuse injury of the extensor tendons of the forearm at their attachment at the lateral epicondyle (extensor carpi radialis brevis) presentation: -lateral elbow pain -TTP -pain w/ wrist extension + supination xray is normal treatment: RICE + NSAIDS
A 30-year-old man presents with pain on the outer aspect of the right elbow for the past few days. The pain was mild and initially associated with vigorous activity; however, for the past 2 days, he has been experiencing pain during daily activities (e.g., lifting objects with his palm down, brushing his teeth, and shaking hands). He started doing carpentry as a hobby 2 months ago. He has no other symptoms and is otherwise healthy. On examination, there is point tenderness at a specific point on his right lateral elbow. An X-ray of the elbow does not reveal any abnormal finding. What is the likely diagnosis?
breast abscess patho: complication of acute mastitis; many are lactational abscess due to S. aureus. presentation: -painful, erythematous cystic mass in breast -fever, malaise -occasional drainage can occur via skin or nipple diagnosis: BIOPSY to exclude carcinoma! treatment: -FNA, I & D (cheaper to do FNA) -lactational abscess: naficillin, cefazolin or vanco
A 31-year-old woman G1P1 presents with fever, malaise, and a mass in the right breast. On physical examination, a 4 cm (in diameter) cystic mass is present in the right breast under the nipple. The mass is tender to palpation. She is currently breastfeeding her 6 month old son. Her vital signs are pulse 95/min, BP 120/80 mm Hg, and temperature 99.9°. A fine needle aspirate is performed on the breast cystic lesion. Carcinoma is excluded via biopsy. What is the likely diagnosis? -patho -presentation -diagnosis -treatment
achilles tendon rupture achilles tendonitis can be caused by increased activity, flat feet, calf tightness rupture: sudden severe pain in the back of the ankle preceded by a pop -forceful plantar flexion PE: thompson test + ultrasound is diagnostic of choice if on field MRI: shows retracted tendon edges treatment: surgical repair, splint in plantar flexion and keep pt nonweight bearing
A 35 year old man who was playing in his sunday soccer league comes into the ED as he cannot walk using his left leg. He describes that as he was going to run after the soccer ball he felt a pop behind his ankle followed by intense pain. He cannot walk or run using that leg now. PE shows a positive thompson test. What is the likely diagnosis? -tendinitis -presentation -PE -diagnostic of choice
secondary adrenal insufficiency due to long term steroid use causes: exogenous steroid use, hypopituitarism presentation: same as Addisons except no hyperpigmentaiton, salt craving or orthostatic hypotension -weight loss, anorexia -fatigue -amenorrhea -loss of libido -hypoglycemia labs: -8 am serum cortisol -low ACTH, low cortisol -normal aldosterone -hypoglycemia treatment: replacement glucocorticoids: hydrocortisone or prednisone
A 32 year old patient with a history of Crohns disease is presenting with amenorrhea for the past month and a 4 pound weight loss. She describes that she has also experiencing nausea, lethargy and abdominal discomfort. She was prescribed prednisone for 8 weeks to treat a terrible Crohn's flare and stopped about a month ago when all these symptoms started. Denies salt craving or hyperpigmentation. CMP is normal except low glucose. Other labs show low ACTH and low cortisol. What is the likely diagnosis? -presentation -labs -treatment
acute hepatitis presentation: recent travel + sudden jaundice -prodrome flu-like sx -jaundice -hepatosplenomegaly -RUQ pain -clay colored stools labs: -U/S first -elevated ALT + AST (ALT > AST unless alcoholic induced hepatitis then AST> ALT) treatment: supportive
A 32-year-old female presents to the emergency department with complaints of severe abdominal pain, fatigue, and nausea. Physical examination is significant for profound jaundice and tenderness to palpation of the right upper quadrant of the abdomen. The patient returned 2 weeks ago from a 1 month-long trip to India. What is the likely diagnosis? -presentation -labs -treatment
medial epicondylitis aka golfer's elbow patho: Inflammation of the pronator teres-flexor carpi radialis muscle (attach @ medial epicondyle) presentation: -Tenderness over medial epicondyle worse w/ pulling activities -pain w/ resisted flexion + pronation xray is normal treatment: RICE + NSAIDS
A 32-year-old man presents with pain on the inner aspect of the right elbow that began a few days ago. The pain was initially mild; however, for the past 2 days, the man says he has been experiencing it during daily activities such as lifting objects, brushing his teeth, shaking hands, etc. He recently became a rock-climbing instructor, and he climbs 3-5 days a week. On examination, there is point tenderness at a specific point on his medial elbow. Pain increases on wrist flexion and forearm pronation against resistance. An X-ray of the elbow is normal. What is the likely diagnosis? -patho -presentation -treatment
Hashimoto's thyroiditis patho: autoimmune dx where anti-thyroid peroxidase antibodies and anti-thyroglobulin antibodies destroy the thyroid gland risk factors: female, 30-50 y/o, family hx, smoking presentation: hypothyroidism plus: -goiter: dyspnea d/t tracheal compression -loss outer 1/3 of eyebrow -myxedema: periorbital -females: galactorrhea b/c increased prolactin labs: ↑ TSH, ↓ T3, T4 + antithyroid peroxidase AB and/or + anti-thyroglobulin antibodies -radioiodine scan: diffuse decreased iodine uptake -↑ LDL b/c T3/T4 promote LDL receptors to remove LDL from circulation treatment: levothyroxine (T4) best to take on empty stomach in the morning
A 32-year-old woman presents with constipation, weight gain, and dry skin. She has been experiencing the symptoms for a few months. Examination findings include dry rough skin, diffuse thyroid enlargement, bradycardia, and edema of hands and feet. Also reports dyspnea and facial puffiness. She reports that she gets cold very easily too. What is the likely diagnosis? -patho -risk factors -presentation -labs -treatment
hypercalcemia causes: -thiazide diuretics -serum acidosis (H binds to albumin, Ca cannot bind) -hyperalbuminemia/dehydration -hyperparathyroidsim presentation: "Bones, Groans, Stones, Thrones Psychiatric Overtones" -bone pain -groan abdominal pain & constipation -kidney stones -increased urinary frequency -AMS -muscle weakness vitals: hypotension + bradycardia treatment: -rehydrating patients -loop diuretics -steroids <GI reabsorption -biphosphonates < osteoclast activity
A 33-year-old female woman develops a loss of appetite. She reports constipation and lethargy. She thinks that her urine output has increased. Her history is significant for the presence of hypertension. She takes HCTZ and recently increased her dosage. On PE she has slow/absent DTRs. Vitals show BP of 110/70. What electrolyte abnormality is she likely to have? -causes -presentaiton -treatment
neuroleptic malignant syndrome causes: antipsychotic medications (1st gen > 2nd gen) or concurrent use of antipsychotics with lithium can result in NMS presentation: tetrad of -fever -altered mental status -lead pipe rigidity -autonomic dysfunction such as HTN, tachy, diaphoresis labs: serum CK ++++ elevated (rhabdomyolysis) treatment: STOP ANTIPSYCHOTIC -transfer to ICU -supportive therapy such as cooling blankets, IV fluids -dopamine agonist: bromocriptine or amantadine
A 34 year old male presents to the ER with a 104 degree fever and altered mental status. He is diaphoretic, arms exhibit lead pipe rigidity, he is excessively drooling and exhibiting tremors. Vitals show 150/95 blood pressure and 120 heart rate. Blood work shows normal CBC and his creatinine kinase is highly elevated. His brother who brought him in lets you know he recently changed medications because his old schizophrenia medications weren't working. What is the likely diagnosis? -patho -presentation -labs -treatment
compartment syndrome patho: Muscle + nerve ischemia (decreased tissue perfusion) when closed muscle compartment pressure > perfusion pressure -pain and paraesthesia earliest sx -pain out of proportion to injury is most specific PE: -pain w/ passive stretching -tense compartment: firm or wood likeater findings: pulsnessess, decreased sensation, pallor, paresis diagnostics: -increased intracompartmental pressure >30 mmHg -increased CK + myoglobin -Delta pressure <30 EMERGENT decompression via fasciotomy While waiting for compression place limb at level with <3
A 34-year-old man was the driver in a single car motor vehicle accident. Preliminary radiologic studies show a comminuted fracture of the right tibia. Currently, the patient is describing a substantially increasing amount of pain felt in the injured extremity. He describes the pain as being a 10/10; it seems as if it is becoming worse with each passing minute. With anguish on his face, he describes it as a deep, achy, burning pain. You quickly examine the right leg; you note pallor, a tense 'wood-like' feeling of the extremity, diminished sensation, and muscle weakness. What is the likely diagnosis? -patho -presentation -PE -diagnostics -treatment
adrenal crisis *usually precipitated by illness treatment: -IV hydrocortisone ASAP -IVF - fix electrolytes + glucose -after IV saline is stopped administer fludrocortisone (b/c primary issues need both steroids)
A 35 year old female patient with a history of Graves comes into the ER with fever, diarrhea, nausea, back pain and fatigue. She had a urinary tract infection 1 week prior. Finger-stick glucose is 65 and BP is 97/68. Dextrose is initiated but her symptoms continue. Lab work comes back with normal TSH, T3/T4, high renin and low aldosterone. Cortisol is also low. What is the likely diagnosis and treatment?
DeQuervain's Tenosynovitis patho: Thickening of abductor pollicis longus, extensor pollicis brevis, and tunnel in the first extensor compartment (Apples with extra peanut butter are delicious) presentation: Pain @ radial side of wrist worse with thumb movement PE: Finkelstein test, pain w/ ulnar deviation of the wrist + thumb in fist tx: thumb spica, NSAIDS
A 35 year old woman presents to her PCP with constant pain that is at the radial aspect of her wrist and worsens with thumb movement. She reports no burning pain, muscle weakness or numbness. She has a 6month old child and 2 year old, as a stay at home mom she is constantly picking them up. On PE Finkelstein test is positive. What is the likely diagnosis? -patho -presentation -PE -tx
cluster headache triggers: alcohol, stress or certain foods presentation: younger or middle aged males -worse at night -severe unilateral periorbital headache that is sharp, lasts <2 hours -several times a day for weeks to months PE: -ipsilateral Horner's syndrome, nasal congestion, rhinorrhea or lacrimation treatment: -100% oxygen first line -5-HT agonists (Triptans) during attack (vasoconstriction) -prophylaxis: verapamil first line
A 35-year-old man presents due to headaches. He describes them as only occurring on 1 side of his head; they tend to have the sensation of a sharp stabbing pain just above his left eye. Each headache lasts for 2 hours at a time; they have been occurring daily for 2 - 3 weeks, and they have had the same pattern every 3 - 4 months for the past 2 years. During these periods, the patient admits to his nose feeling congested; his left eye appears red, and it excessively waters. Aggravating factors appear to be alcohol and stressful situations. He denies any known family history of headaches similar to his or migraines. The physical examination is within normal limits. What is the likely diagnosis? -triggers -presentation -PE -treatment
nephrolithiasis (not pyelo b/c there would be pyuria and/or significant leukocyte esterase + nitrites) risk factors: low fluid intake, high salt/protein diet, family hx, obesity, EtOH use presentation: -renal colic pain that radiates to the groin -difficult to find comfortable position -hematuria -oliguria -N/V -urinary frequency, urgency diagnostics: -UA -CT without contrast** -U/S: identify stones only in the kidney ,proximal ureter or UVJ treatment: <5mm: pass spontaenously 5-10 mm: not likely to pass spontaneously; IV fluids + alpha blockers (tamsulosin) to relax ureters & help stone pass. Elective interventions >10 mm: intervention warranted - ureteral stent or percutaneous nephrostomy if renal function jeopardized -shock wave lithotripsy NSAIDS or Ketorolac (IV NSAID) for pain
A 35-year-old woman is seen acutely in the office with severe right-sided flank pain that has persisted for the last 6 hours and is radiating to her groin. UA shows trace amount of blood in urine and trace leukocyte esterase. She is writhing on the exam table and cannot get comfortable. What is the likely diagnosis? -risk factors -presentation -diagnostics -treatment
Grave's Disease patho: autoimmune dx where autoantibodies bind to the TSH receptors and stimulate the gland to overproduce T3 and T4 (hyperthyroidism) risk factors: female 20-40 y/o, treatment with potassium iodine or amiodarone (contains iodine), smoking unique Grave presentation: -opthalmopathy: proptosis, exophthalmos, lid lag, diplopia, dry eyes -pre-tibial myxedema: non-pitting edema with red or brown patches labs: ↓ TSH, ↑ T3, T4 -↑ LFT, ferritin, and bilirubin + thyroid stimulating immunoglobulins (TSI) and/or + thyrotropin receptor antibodies (TRAb) treatment: -thioamides: methimazole or propylthiouracil these will ↓ TPO and inhibit formation of T3/T4 -propranolol: palpitations -glucocorticoids: exophthalmos definitive: radioactive iodine ablation of thyroid gland, will need hormone replacement thereafter
A 35-year-old woman presents with a 2-month history of palpitations and nervousness. She mentions that she always feels hot, even if the weather is cold. Her menses have been irregular lately. She has had no fevers recently. On examination, her blood pressure is 150/70 mm Hg and her pulse is 89 beats per minute. Her eyes show exophthalmos, and she also has lid lag on looking down. There is non-pitting swelling on her lower legs and it looks red in patches. What is the likely diagnosis? -patho -risk factors -presentation -labs -treatment
anterior uveitis patho: inflammation of the middle layer of eye (iris + ciliary body). Associated w/ inflammatory dx: sarcoidosis, IBD, reactive arthritis, ankylosing spondylitis presentation: Sudden redness of eye -unilateral pain + vision loss -blurry vision -severe photophobia -hypopyon within the anterior chamber (WBC) tx: Urgent referral to ophthalmology! -Steroid drops + dilation of pupil (relieve discomfort)
A 36 year old female with a history of sarcoidosis presents to the ED with a sudden onset of unilateral left eye pain and blurred vision. She has also been experiencing photophobia. On PE you notice hypophyon within the anterior chamber of the left eye. What is the likely diagnosis? -risk factors -presentation -treatment
Cushing's Disease ACTH secreting pituitary adenoma (high ATCH, high cortisol) -Cushing's Syndrome: prolonged exposure of excess cortisol; doesn't specify cause or source of excess diagnostics: -24 hour urinary free cortisol level -ACTH level to determine source of cortisol (primary or secondary causes) -dexamethasone suppression test (does not reduce cortisol levels) -other labs: hyperglycemia, dyslipidemia, leukocytosis, hypokalemia treatment: surgical resection of tumor (pituitary, adrenal, ectopic) followed by hydrocortisone medication therapy if surgery is not an option: -ketoconazole: inhibits CYP needed for synthesis of steroids -metyrapone: inhibits cortisol
A 36-year-old man with a past medical history of diabetes mellitus presents with weight gain and skin changes. His review of systems is positive for a diminished libido, impotence, depression, cognitive dysfunction, and emotional lability. Lately, his fasting glucose levels have been above normal. His physical exam notes increased adipose tissue in the face, upper back, and above the clavicles. His skin reveals ecchymoses, telangiectasias, and purpura along his back and lower extremities; there is also facial acne and cutaneous atrophy. His abdominal exam reveals severe purple stretch marks. What is the likely diagnosis? -causes/labs -diagnostics -treatment
B. intracerebral neoplasm presentation: -nausea -intractable vomiting (cannot stop) -neurological deficits -morning headache (red flag: wakes them up from sleep, seizure, pain worsens with valsalva) diagnostics: -MRI w/ contrast or CT w/ contrast
A 37 year-old male presents with headaches for the past 2 months. They occur daily and are worse in the morning. In the past week, he has noticed a tendency to drop things from his right hand. On examination, vital signs are normal, and general examination is unremarkable. Neurologic examination reveals mild weakness of the right upper extremity compared to the left. Which of the following is the likely diagnosis? A. focal seizure disorder B. intracerebral neoplasm C. transient ischemic attack D. amyotrophic lateral sclerosis
arterial embolization/occlusion patho: sudden decrease in limb perfusion that may threaten limb viability and is usually due to acute arterial occlusion causes: A-fib or mitral stenosis causes thrombus formation presentation: 5 P's -Pain -Pulse (lack of pulse) -Pallor -Paresthesia -Paralysis treatment: -non limb threatening: IV heparin → vascular surgeon for angioplasty/graft/endarterectomy
A 37-year-old diabetic patient with history of paroxysmal atrial fibrillation and hypertension presents with a sudden onset of severe pain in his left arm. On physical examination, the left arm is cool and pale with nonpalpable radial and ulnar pulses. What is the most likely cause for his pain? -patho -causes -presentation -treatment
acute pancreatitis patho: digestive enzymes start digesting pancreas itself presentation: -epigastric pain that radiates to the back lessens when patient leans forward + increases after large meals or laying down -N/V -Grey turner: flank bruising -Cullen: bruising near umbilicus diagnostics: -elevated lipase + amylase -abdominal CT: localized dilation of upper duodenom -ERCP (chronic pancreatitis) treatment: IV fluids, analgesics, bowel rest
A 37-year-old male complaining of rapid onset of severe mid-epigastric pain with radiation to the back after eating a large meal. The pain lessens when she leans forward or lies in the fetal position. Physical exam shows low-grade fever, epigastric tenderness, diminished bowel sounds, and bruising of the flanks. What is the likely diagnosis? -patho -presentation -diagnostics -treatment
Mallet Finger patho: A mallet finger deformity is due to the rupture of the extensor tendon at the base of the dorsal distal phalanx presentation: -inability to extend the dip joint resulting in flexed DIP -if untreated can progress to a swan neck deformity xray: bony avulsion at the distal phalanx treatment: extension, splinting of DIP joint 6-8 weeks 24 hours a da
A 38-year-old man presents with pain and the inability to extend his middle finger DIP joint following a sudden jamming-type injury when attempting to catch a football 3 hours ago. Based on the patient history, what is the most likely diagnosis? -patho -presentation -diagnostic of choice -treatment
azithromycin to treat underlying pertussis phases: gram negative bacterial cough > 2 weeks (adults) 1. catarrhal: cough, rhinorrhea, conjunctivitis, 2. paroxysmal: 2nd week of illness; Rapid-fire repetitive coughing followed by an inspiratory whoop and post-tussive emesis 3. reduction of sx 1-2 months CXR: normal, nasopharyngeal swab + tx: azithromycin, Bactrim. Can give household contacts prophylaxis
A 4-year-old boy who is unvaccinated presents to the pediatric clinic with worsening cough for two weeks. His mother reports episodes of frequent coughing that are followed by a gasp and posttussive vomiting. On physical examination, there are no signs of respiratory distress. Chest radiograph is normal. Polymerase chain reaction by nasopharyngeal swab is collected and sent to the lab. Which of the following is the recommended treatment while waiting for results? -patho/presentation -treatment
supracondylar humerus fracture aka distal humerus fracture MOI: FOOSH w/ hyperextension of arm presentation: children 2-7 (MC pediatric elbow fracture) -pain worse w/ flexion or extension -deformity can occur -bruising, effusion, crepitus -cannot passively move elbow xray: fat pads present on x-ray (dark area on either side of bone) anterior and posterior fat pad treatment: long arm posterior splint followed by long arm casting -YOUNG: open reduction and internal fixation (good bone quality) -older pts w/ osteoporosis: complete elbow arthroplasty
A 4-year-old girl presents 30 minutes after falling from a slide (about a 4-foot fall) on her outstretched right dominant arm with her elbow fully extended. There was no loss of consciousness, but there was immediate extreme pain; there is an obvious deformity at the elbow. On presentation, the girl is holding her right elbow and she will not allow passive movement. What is the likely diagnosis? -moi -presentation -xray -treatment
migraine headache "common" no auras presentation: lasts 4-72 hours -unilateral throbbing/pulsatile HA -N/V -photophobia -photophonia "classic" aura presentation: <60 minutes -visual changes such as flashing lights or hemisensory loss symptomatic treatment: mild -NSAIDS, acetaminophenin -fluids, caffeine or being in a dark room may help moderate or refractory to initial tx -5-HT agonists that cause vasoconstriction and block pain pathways in brainstem such as Triptans or Ergotamines -dopamine blocker anti-emetics like prochlorperazine or metoclopramide
A 40 year old female presents with a 1 month history of on and off headaches. She describes them as lateralized to one side, pulsatile/ throbbing. When they occur she feels nauseated, sensitive to light and sound. It lasts around 4 hours then goes away. What is the likely diagnosis? -presentation (auras vs non-auras) -treatment: mild or moderate
Meniere's Disease patho: secondary to excess endolymph fluid in the inner ear causes: autoimmune dx (SLE, RA), infections, trauma presentation: -episodic vertigo (mins to hours) -sensoneural hearing loss -tinnitus -ear fullness tx: avoid salt, caffeine, nicotine & chocolate (these increase endolymph pressure) -diuretics can be used to decrease endolymph pressure
A 40 year old female with a history of rheumatoid arthritis presents with episodes of vertigo that lasts a few minutes over the past two weeks. Associated symptoms include ringing in her right ear and fullness. On PE, Weber test shows sound her best in the left ear and Rinne is normal (AC>BC). What is the likely diagnosis? -patho -causes -presentaiton -tx
Benign Proximal Positional Vertigo (BPPV) patho: displaced otolith particles within semicircular canals presentation: sudden episodic vertigo (lasts ~60 sec) occurs with head movements like rolling over in bed. -can have N/V -no hearing loss diagnostic: Dix Hallpike test: change position of pt from sitting to laying down and check for nystagmus treatment: Epley maneuver to reposition otoliths & stop vertigo
A 40 year old male comes in with complaints of nausea and episodes of dizziness. He notices that the dizziness happens when they do things like roll over in bed or look up. Denies hearing loss or ear fullness. On PE you see horizontal nystagmus. What is the likely diagnosis? -patho -presentation -diagnostic -treatment
Cauda Equina syndrome patho: compression of the cauda equina lumbar/sacral nerve roots presentation: -saddle anesthesia -urinary retention or incontinence -bilateral pain and paresthesias in leg -leg weakness diagnostics: -MRI: compression of sacral nerve roots/cauda equina treatment: EMERGENCY if true urinary incontinence/retention occurring surgical decompression
A 40 year old patient comes into the ER with an acute onset of pain along the front and back of her legs. It is a radiating burning type of pain. She also reports numbness in her groin and accidentally urinated in the car on the way to the ER. What is the likely diagnosis? -patho -presentation -diagnostics -treatment
tuberculosis history: unsheltered, recent immigration from high risk country, incarceration or healthcare presentation: -prolonged cough >3 weeks -hemoptysis -fever, night sweats, chills -fatigue, malaise, weight loss diagnostics: -CXR: cavitation + calcifications in the upper lobes. Can also show: atelectasis, pleural effusion, ghon complexes (healed primary infection) -NAAT -biopsy: caseating granulomas tx: RIPE -rifampin (orange tears) -isoniazid (give w/ b6: peripheral neuropathy) -pyrazinamide -ethambutol (color blindnesS)
A 40-year-old man presents with a 3-month history of a productive cough; the cough produces a scant amount of yellow sputum. He has also had an evening temperature rise (101 F) for the past 3 months. On occasion, the sputum was blood-streaked. He has lost a significant amount of weight during this period. Patient is a non-smoker. He immigrated from India 5 years ago, and his medical records are not available. Coarse upper lobe crackles and rhonchi are heard bilaterally. X-ray shows multiple bilateral upper lobe cavities with surrounding infiltrate. What is the likely diagnosis? -hx -presentation -diagnostics -treatment
hyperkalemia presentation: affects the muscles and <3 (muscle weakness, nausea, vomiting, paralysis, bradycardia, hypotension) decreases excitability of <3 tissue EKG: -peaked T waves -wide QRS -increased PR interval -sine wave acute treatment: 1. Ca Chloride/gluconate (stablize cardiac membrane) 2. Bicarb (tx acidosis) 3. albuterol (B2 agonist) 4. Insulin + dextrose more chronic tx (takes longer) 1. furosmeide 2. kayexalate, patiromer (chelators)
A 41-year-old diabetic woman is in the ER with an abrupt onset of muscle cramping, nausea and vomiting. Her EKG shows peaked T waves as well as a widening of the QRS complex. What electrolyte abnormality does she likely have? -presentation -EKG -acute vs. chronic tx
esophageal varices Bleeding from esophageal varices secondary to portal hypertension and obstruction of splenic blood flow is a common source of UGI bleeding presented to emergency departments. diagnostics: upper endoscopy tx: -IV ocetrotide (vasoconstriction) -ligation of varices -balloon tamponade (but must intubate pt to protect airway)
A 41-year-old man presents with acute hematemesis. Abdominal exam reveals distension, no rebound or guarding, hepatosplenomegaly, and dull fluid wave. What is the most likely source of the upper gastrointestinal bleeding (UGI)? -diagnostics -tx
anal fissure presentation: associated with tearing pain during defecation, constipation, hematochezia, and the presence of a crack in the epithelium on examination below the dentate line. tx: tx underlying constipation, Crohn's, sitz bath for comfort
A 41-year-old woman presents due to rectal pain. The pain is brought on by bowel movements and is described as an intense tearing pain. She notes a small amount of blood on the toilet paper, but none in the stool itself. She has a bowel movement approximately every 2 to 3 days and notes that her stool is somewhat hard. On examination, a small crack is noted in the posterior midline. Hemoccult is negative. There are no masses noted on rectal examination. What is the likely diagnosis?
carpal tunnel syndrome patho: compression of the median nerve between the carpal ligament & other structures risk factors: pregnancy, diabetes, hypothyroidism presentation: -burning pain + tingling in thumb, 2nd, 3rd and radial 1/2 of 4th finger -initially worse during sleep -muscle atrophy + sensory deficits worsens as compression worsens PE: + tinels +Phalens - check median nerve function w/ pincer grasp tx: -neutral wrist splinting at night -NSAIDS, steroids -release procedure if progressive sx + severe
A 42-year-old woman works full-time as a data entry clerk and often puts in many hours of overtime. She is experiencing numbness and tingling in her right thumb, index finger, middle finger, and half of her ring finger. The numbness and tingling initially comes and goes, but it is constantly present after a few months. What is the likely diagnosis? -patho -risk factors -PE -tx
cholesteastoma patho: Chronic otitis media w/ infolding of ear canal into the middle ear → secrete enzymes that destroy ossicles and surrounding bone presentation: -flulffy white mass w/ perforation of ear drum (painless, grows slowly) -may invade facial nerve or meninges or brain if left untreated diagnostics: CT temporal bones w/out contrast tx: refer to ENT (not an emergency)
A 43-year-old male presents with "lifelong" history of chronic ear infections and episodic purulent drainage from his right ear canal. The patient currently is without symptoms. Examination of the ear shows a clear external canal, but the tympanic membrane is retracted and there is a pocket of white material and an opacity of the pars flaccida. The Weber test lateralizes to the right and Rinne shows AC > BC on the left and BC > AC on the right. What is this finding called? -patho -presentation -diagnostics -treatment
(B) Dressler syndrome is an autoimmune pericarditis that can occur days to months post myocardial infarction (MI). It normally presents with low- grade fever, malaise, and chest pain, lessened by sitting up. Repolarization changes can be seen on ECG. Acute MI does not typically have diffuse J-point elevation with no reciprocal changes present and the pain is atypical to that of an acute MI. Although viral pericarditis is the most common cause of acute pericarditis, it is less likely given the patient's recent MI. This is not stable angina as the symptoms are not exertional but are positional and at rest.
A 43-year-old male with history of an anterior myocardial infarction with LAD stent placed 9 days ago presents to the clinic with chest pain. The pain is worsened when he is supine and improves with sitting up. His temperature is 100.9°F. On physical examination, you hear a high-frequency, scratching sound that is increased with inspiration. ECG shows sinus tachycardia with diffuse J-point elevation. What is this patient's diagnosis?
vasovagal episode patho: overactivity of parasympathetic nervous system in response to a trigger causes a fall in vascular resistance without a rise in cardiac output leading to hypotension and bradycardia most common cause of syncope presentation: vasovagal/neurally mediated syncope will ALWAYS present with prodromal symptoms of diaphoresis, weakness, blurry vision and dizziness
A 44-year-old man presents after an episode he describes as "passing out." He has a past medical history of hypertension, diabetes mellitus type 2, and osteoarthritis. Earlier in the day, the patient visited his primary care physician for routine blood work. During his blood draw, the patient had an acute onset of syncope. The patient has never had such an episode before. Vital signs upon presentation are as follows: BP 124/82 mm Hg, P 88 beats/min, R14/min. EKG is within normal limits. Cardiovascular examination reveals a normal S1 and S2, with no rubs, murmurs, or gallops. The patient is being treated with atenolol and hydrochlorothiazide for his hypertension. His blood sugar is 96 mg/dL. What is the most likely cause of the patient's syncope? -patho -presentation
radial head fracture the fat pad sign may be the only sign on xray if the radial fracture is non-displaced treatment: long arm splint at 90 degrees open reduction and internal fixation- in adults usually requires non-operative management
A 45 year old falls off a curb on her outstretched hand and is expierencing acute pain on the outside of her right elbow. She is unable to flex or extend the right elbow due to pain. She cannot supinate or pronate either. PE shows localized swelling and tenderness to the right lateral elbow. X-ray shows fat pads or "sail" sign. What is the likely diagnosis?
cubital tunnel syndrome patho: compressive neuropathy of the ulnar nerve along the medial elbow presentation: Paresthesia and pain along ulnar nerve distribution that is worse w/ elbow flexion → fifth finger + ulnar side of fourth finger PE: + tinnel sign + froment sign (pull paper away, ICP joint flexes) tx: conservative or surgery if severe sx
A 45 year old female presents with pain and tingling in her pinky finger that worsens with elbow flexion. What is the likely diagnosis? -patho -presentation -PE tx
gout risk factors: -men** -obesity -metabolic: hyperlipidemia, DM -HTN, ASCVD -alcohol -high purine diets presentation: -most common site is the great toe MTP (podagra), ankle, or knee -acute arthritis with swelling, redness and TTP of joint -chronic gout present with tophi which can ulcerate skin and excrete chalky material composed of MSU acute treatment: NSAIDS (indomethacin) , colchicine (diarrhea), or corticosteroids are all utilized chronic treatment: allopurinol
A 45 year old male presents to the clinic with a painful nodule on his right great toe. He said that it is extremely painful. On PE you notice that the great toe is red and swollen. Patient has a history of hypertension and stable angina. In his chart you see that he has a history of alcohol use disorder and he states that he has continued to drink. Joint fluid analysis shows rod shaped negatively birefringent urate crystals. What is the likely diagnosis? -risk factors -presentation -treatment (acute vs. chronic)
rhabdomyolysis causes: -statins** -trauma, seizures, snake bites -strenuous activity patho: Myoglobin from muscle breakdown is extremely toxic to the renal tubular cells, leading to Acute tubular necrosis (acute kidney injury). presentation: triad of muscle pain + muscle weakness or swelling + dark (tea-colored) urine (myoglobinuria) diagnostics: -EKG: hyperkalemia -UA/urine dipstick: + for heme but negative for RBC on microscopic exam (myoglobin in urine) -creatinine kinase elevated -eletrolyte imbalance: hyperkalemia, hyperuricemia, hypocalcemia tx: IV fluids first line & mainstay of treatment
A 45-year-old female with chronic kidney disease comes to the office for routine follow-up. Three months ago, therapy with a statin (HMG-CoA reductase inhibitor) was initiated, and since that time, the patient says she has been having muscle pain. Physical examination shows tenderness to palpation over the quadriceps muscles. Results of laboratory studies include elevated serum creatine kinase level. Urinalysis shows the presence of myoglobin. What is the likely diagnosis? -causes -patho -diagnostics -treatment
IV Lorazepam Status epilepticus = Medical emergency! -must treat with anesthetics -ABC -IV access (NS no D5W for concern of brain swelling) -monitor -treat hyperthermia (result of increased motor activity, circulating catecholamines) -Ativan (lorazepam) 4mg bolus (short acting anticonvulsant) then phenytoin then phenobarbital **place patient in left lateral decubitus position (suppresses gag reflex these pts are prone to aspiration)
A 45-year-old male is brought to the ED having an active grand mal seizure that has lasted greater than 15 minutes. The patient has a known history of seizure disorder. No treatment has been initiated up to this point, and the patient is still actively seizing. What is the best immediate treatment choice for the patient in this scenario? (A) Emergent bedside EEG (B) IV lorazepam (C) IV midazolam (D) IV phenobarbital (E) IV phenytoin
Pericarditis patho: inflammation of the pericardial sac (post-viral infecton, post MI) presentation: Pleuritic chest pain -worse lying down -better w/ leaning forward Tachycardia Pericardial friction rub diagnostics: -echo: diagnostic -EKG: diffuse ST elevation treatment: -NSAIDS -colchicine to prevent recurrence -steroids only if refractory/severe
A 45-year-old man presents with a 6-hour history of retrosternal chest pain. He describes it as a sharp and episodic, which is relieved by sitting upright and worsened by lying down. He denies any trauma to the chest. What is the most likely cause of his chest pain? -patho -presentation -diagnostics -treatment
Correct Answer: The patient abruptly stopped taking the medication The correct response is that the patient abruptly stopped taking the medication.Benzodiazepines, such as Xanax, can cause physical dependence. When alprazolam is stopped abruptly, the patient has signs and symptoms of withdrawal (e.g., sweating, tremors, and insomnia).Tolerance and side effects from the medication would result in signs and symptoms of anxiety (e.g., restlessness, irritability, and muscle tension).Since the patient has been on the medication for 6 months, an increase would not yet be required
A 45-year-old woman presents with sweating, tremors, and insomnia. About 6 months ago, the patient divorced her husband of 15 years; she was placed on Xanax 0.5 mg daily due to frequent panic attacks. A week ago, her court proceedings were resolved. What is the most likely cause of the patient's symptoms? 1 The patient is experiencing common side effects of the medication from chronic usage 2 The patient has developed a tolerance to the medication 3 The patient abruptly stopped taking the medication 4 The patient requires a increase in dosage of medication 5 The patient has overdosed on the medication
acute closed angle glaucoma patho: Iris bulges forward and seals off anterior chamber + prevents aqueous humor from being able to drain (increasing IOP) presentation: -severely red and painful eye -blurred vision -halo around lights -HA, N/V -red eye, teary, hazy cornea PE: -decreased VA -fixed pupil size -firm eyeball on palpation diagnostic: Tonometry shows increased IOP (>21 mmHg) tx: EMERGENT referral to ophthalmologist; requires surgery -Decrease IOP w/ eye drops such as timolol (BB), brimonidine (alpha agonist) + acetazolamide
A 45-year-old woman suddenly experiences severe pain in the right eye along with blurred vision, nausea, and vomiting. Intraocular pressure is 58 mm Hg and there is a moderately dilated right pupil, decreased visual acuity, shallow anterior chamber, and steamy cornea noted during physical examination. Based on this information, what is the most likely diagnosis? -patho -presentation -diagnostics -treatment
I&D by ENT specialist ASAP this hematoma can cause avascular necrosis of cartilage resulting in a cauliflower ear + prone to ear infections → oral ABx against staph <2 cm: needle aspiration, >2cm: I & D, refer to ENT
If a patient presents with subperichondrial hematoma, how should this be treated?
Small bowel obstruction causes: adhesions, incarcerated hernia, bariatric surgery presentation: -abdominal pain that is crampy, intermittent, and episodic -periumbilical or diffuse -vomiting may be bilious or feculent (proximal vs. distal) -constipation (not always) PE: -distension -abdominal tenderness -percussion: tympanic -bowel sounds: active, high pitched (early) -rebound tenderness: perforation diagnostics: -KUB: dilated loops of bowel with air fluid levels -CT w/out contrast -CBC, volume depletion (not absorbing fluids) tx: SBO: bowel rest, NPO, fluids, NG tube decompression
A 46-year-old woman presents with nausea, vomiting, crampy abdominal pain, and loud bowel sounds for the past several hours. She denies weight loss. She has had 1 normal bowel movement since the symptoms began, but this did not help her symptoms. She has a past surgical history of an abdominal hysterectomy 7 years prior. On physical exam, she is afebrile, with hyperactive and high-pitched bowel sounds localized to the left upper quadrant. She also has mild, diffuse abdominal tenderness. What is the likely diagnosis? -causes -presentation -diagnostics -treatment
pulmonary hypertension patho: The reduced oxygen concentration present at high altitudes can trigger (or worsen) the development of pulmonary hypertension in patients who were previously asymptomatic. AKA high altitude pulmonary edema presentation: -dry cough -decreased exercise tolerance -dyspnea on exertion PE: rales, low O2 sat, tachycardic, tachypnea EKG: RV strain pattern treatment: -Oxygen, descent to lower altitudes -Nifedipine: prophylaxis -Tadalafil: PDE-5 inhibitors
A 47-year-old Caucasian woman, previously healthy, presents for evaluation of a 4-week history of dyspnea. Her symptoms started approximately 1 month ago when on vacation in Colorado. She initially attributed these symptoms to the altitude; however, upon return to her hometown, she continued to have shortness of breath with mild activity, such as walking more than 100 feet, walking upstairs 1 flight, vacuuming, or sweeping. Her symptoms resolved with rest. She also reports mild exertional chest tightness and easy fatigability. She denies paroxysmal nocturnal dyspnea, orthopnea, edema, palpitations, and syncope. Physical exam reveals an alert white woman in no acute distress. Her vitals reveal temp 96.9°F, pulse 80 and regular, respirations 16, and BP 136/82. O2 sat is 96% on room air. There is no obvious jugular venous distention. Respirations are non-labored. Lung fields are clear to auscultation and percussion. No rhonchi, rales, or wheezes are present. Heart shows RV heave present, normal S1 with fixed, split S2 with prominent P2 component. Grade II/VI systolic murmur is present at the left upper sternal border at the second intercostal space. The remainder of a complete physical examination is within normal limits. CBC and BMP are unremarkable. Free T4 and TSH are within normal limits. EKG shows normal sinus rhythm with right ventricular hypertrophy, right atrial enlargement, and right axis deviation. There is an RSR in leads v1 and v2. Question What do you suspect is the primary cause of her dyspnea?
Adhesive Capsulitis (Frozen Shoulder) patho: painful and stiff glenohumeral joint risk factors: diabetes**, thyroid dx, females presentation: -diffuse shoulder pain -progressive stiffness -severe ROM limitations -passive ROM = active ROM -Apley Scratch test (restricted ROM) tx: -physical therapy + CSI
A 47-year-old grossly obese woman presents with left non-dominant shoulder pain and limited motion. The pain began about a month ago, and her shoulder has progressively lost motion during that time; now she can't reach overhead with that arm. There is no history of trauma; it doesn't wake her at night; and she can sleep on the left side. Her past medical history is significant for Diabetes Type I and hypothyroidism. She is on regular insulin and NPH insulin as well as levothyroxine (Synthroid). What is the likely diagnosis? -patho -presentation -risk factors -tx
hypoglycemia any blood glucose <70 requires treatment treatment: -IV dextrose, SC glucagon or intranasal glucagon if non-urgent --> 15/15 rule
A 47-year-old woman presents because she feels "weak and dizzy". She was feeling well up until about an hour ago, at which point she began to feel as if she were going to "pass out." She is presenting with confusion, anxiety and tremors too. On PE she is found to have tachycardia and generalized weakness. Other vitals are normal and labs show a blood glucose level of 47 mg/dL. The patient receives treatment and improves immediately. What is the likely diagnosis & treatment?
(E) Cornea This sign is known as Hutchinson's sign
A 48-year-old male patient is diagnosed with shingles involving cranial nerve V. During the examination you note that the tip of his nose is involved. At this point, it is crucial to rule out involvement of: (A) The opposite pinna (B) Nasal septum (C) Ipsilateral epitrochlear node (D) Tympanic membrane (E) Cornea
acute otitis media patho: infection of the middle ear; most commonly preceded by viral URI which leads to blockage of ET. MC patho = S. pneumoniae presentation: -fever -otalgia (ear pain or ear tugging in infants; less than 48 hours) -stuffiness -conductive hearing loss PE: bulging + erythematous TM with effusion. Redness d/t vascular flush d/t fever + crying diagnostics: pneumatic otoscopy shows decreased TM mobility treatment: -high dose amoxicillin (augmentin if no improvement 2-3 days or failed w/ amoxicillin) ---<2 y/o: tx 10 days --->2 y/o: tx 5-7 days -ibuprofen or acetaminophen prn (takes 1-3 days for ABx to kick in)
A 5-year-old boy presents with a sudden high fever and severe pain in the right ear. He has been very irritable since the initial onset of symptoms. He also seems to have some difficulty in hearing. The child has been suffering from a cough and cold for the last 2 weeks. What is the likely diagnosis? -patho -presentation -PE -diagnostics -treatment
acute tubular necrosis; tubular type of intrarenal AKI causes: -ischemia: renal hypoperfusion (pre-renal failure) -toxins: aminoglycosides, cisplatin, amphotericin B, NSAIDS, contrast, rhabdomyolysis -obstruction: stone UA: epithelial cells, muddy brown casts RUS: no hydronephrosis FeNA >1 treatment: -stop giving trigger (aminoglycoside, NSAID) -do NOT give pt more contrast -fluids (flush out the toxins/obstructive products)
A 50 year old HIV positive male was admitted 5 days ago for endocarditis infection. He was prescribed ceftriaxone + neomycin. He is now presenting with oliguria, peripheral edema and nausea. CMP shows SCr 2 and UA is positive for epithelial cells. What is the likely diagnosis? (be specific!) -cause -diagnostics -treatment
subarachnoid hemorrhage (SAH) risk factors: smoking and hypertension most important. Polycystic kidney disease, alcoholism, and ASCVD are others patho: most common = ruptured berry aneurysm at the anterior communicating artery (circle of willis) presentation: -worst headache pt has ever had/thunderclap -ocular hemorrhages and/or papilledema on PE -change in consciousness -N/V -signs of meningeal irritation like fever, neck stiffness diagnostics: -CT: subarachnoid bleeding (usually more central in the film than hematomas which are more peripheral) -LP: bloody CSF or xanthochromia if bleeding has been present > 2 hours (yellowish due to RBC breakdown) treatment: management of hypertension and increased ICP: strict bed rest, stool softeners to prevent straining, Nimodipine reduces cerebral vasospasms -surgical clipping + wrapping of aneurysm
A 50 year old female presents to the ER with a 3 hour onset of the worst headache she has ever had. It is unilateral on the right side of her head. She also complains of nausea, vomiting, confusion and has had some neck stiffness since the onset. She has 30 year pack history, hypertension and polycystic kidney disease. Imaging is inconclusive so a lumbar puncture is performed. It shows CSF is positive for xanthochromia and opening pressure is 30 cmH2O. What is the likely diagnosis? -risk factors -patho -presentation -diagnostics -treatment
mesenteric ischemia history: CAD, A-fib, clotting dx MC artery: superior mesenteric artery presentation: post-prandial abdominal pain -Fever -N/V -bloody stool -pain out of proportion to exam diagnostics: CT angiography treatment: -revascularization = gold standard; bowel resection for bowel infarction -bowel rest, fluids, ABx
A 50 year old male with a history of atrial fibrillation and coronary artery disease presents to the ED with acute onset of abdominal pain. He noticed some bright red blood in his stool and was worried which brought him in. On PE he has a slight fever, tachycardia and is in excruciating pain. CT of the abdomen shows thickened bowel wall. What is the likely diagnosis? -history -presentation -diagnostics -treatment
acoustic/vestibular neuroma patho: benign growth of nerve cells at the site of nerve injury, arises in the cerebellopontine angle and compresses against CN-VIII. As it grows can compress CN-V &/or CN-VII. presentation: -unilateral sensoneural hearing loss is an acoustic neuroma until proven otherwise! -tinnitus -facial numbess or paresis (CN V or 7 affecteD) -vertigo -ataxia diagnostics: MRI treatment: surgery or focused radiation therapy
A 50 year old patient is presenting with a 4 month history of worsening tinnitus and difficulty hearing in her left ear. She also reports times where the room seems to be spinning around her. On PE you note that she has unilateral left sided facial numbness on along the side of her face. Weber is positive with sound being heard better on the R ear, Rinne is normal. What is the likely diagnosis? -patho -presentation -diagnostics -treatment
SIADH patho: excess ADH → free water retention and hyponatremia causes: -SAH (most common) or head trauma -small cell lung cancer** ectopic secretion of ADH -carbamazepine, NSAIDS, HCTZ presentation: sx of hyponatremia -confusion -lethargy -seizure or coma or cerebral edema if severe labs: -low serum Osm & Na -high urine Osm treatment: admit pt & monitor tx -water restriction -IV 3% hypertonic saline + furosemide -vaptans (vasopressin antagonists) chronic: demelocycline
A 50 year old patient with a history of small cell lung cancer is brought into the ER by his wife after he suffered a seizure. He is confused, altered and lethargic in the exam room. His CMP comes back showing sodium levels to be 110. His UA shows a specific gravity that is very high. What is the likely diagnosis? -patho -causes -presentation -labs -treatment
Trigeminal neuralgia presentation: most common in middle aged women -intermittent, short lasting, unilateral sharp pains felt in trigeminal nerve distribution -triggered by movements such as tooth brushing, chewing, speaking -tender to palpation treatment: carbamezapine
A 52-year-old female bus driver presents to the clinic with a chief complaint of intense, shooting pains in her right cheek, each lasting for only a few seconds she avoids touching certain parts of her face and has started to chew food only on the right side of her mouth because she is afraid she will set off an attack of pain. In between attacks, the patient feels well. What is the likely diagnosis? -presentation -treatment
Psyllium is the correct response. The patient has an anal fissure. Anal fissure is exacerbated by constipation. First-line treatment is fiber supplementation, such as psyllium and sitz baths fail conservative treatment: Botulinum toxin A
A 52-year-old man presents with rectal pain during bowel movements. Patient describes his pain as tearing. Physical examination reveals a tear in the anal mucosa. What is considered first-line treatment in this case? 1. Psyllium 2. Topical nitroglycerin 3. Oral ciprofloxacin 4 Botox injection 5 Surgery
rectal abscess pain out of proportion to exam; requires CT imaging tx: anything that is not a superficial perianal abscess (that is seen on PE red, swollen, tender) must go to the OR as it could be a deep space infection
A 53-year old-man presents with a history of indolent, dull pain in his rectal area; it is associated with pruritus. The pain is increased upon movement, defecation, and prolonged sitting.He has experienced constipation and describes his stools are described as "hard", but they retain their normal brown color without evidence of blood. He denies ever having had these symptoms before.He denies any sexual contact, drug use, or fever, chills, or other gastrointestinal symptoms. Upon inspection, erythema is observed, and a digital rectal exam reveals a fluctuant, tender, and indurated mass. What is the likely diagnosis? -presentation -diagnostics -treatment
anterior cord syndrome mechanism of injury: flexion or vascular aortic dissection, compression or crush injury presentation: -Complete loss of motor, pain + temp below injury -Retain proprioception + vibration
A 53-year-old man presents to the emergency department after a diving accident. He has no motor function of the lower extremities and has partial impairment of motor function of the upper extremities. He has no sensation to pain or temperature below the C5 level. He has intact sensation to proprioception and vibration in the extremities. X-rays and CT scan show a burst fracture of the C5 vertebra with retropulsion of bone into the spinal canal.What type of spinal cord injury does this patient have? -mechanism of injury -presentation
Overflow incontinence patho: impaired detrusor contractility causes: -neurogenic bladder: diabetic, lower motor neuron lesion -medications: anticholinergics, alpha agonists -obstruction (BPGH, kidney stones) treatment: -intermittent self catherization 1st line -cholinergic (Bethancol) : increase bladder contractions -alpha blockers: relax urinary sphincters
A 55 y/o patient with diabetes presents with urinary incontinence. She describes she has the urge to go but then feels like her bladder is not fully emptied. She constantly has accidental leakage of urine and nocturnal enuresis. PVR shows high post-void volume. What is the likely diagnosis? -patho -causes -treatment
Pentoxifylline (Trental)
What medication can be used to reduce mortality and hepatorenal syndrome in severe alcoholic hepatitis patients?
The correct response is a CT scan of the abdomen. Patients with acute diverticulitis can be diagnosed based on the clinical presentation. Since other acute surgical conditions can present in a similar clinical pattern, confirmation of the diagnosis is important.
A 57-year-old man presents with a sudden onset of left lower quadrant pain, fever, and guarding. The patient appears septic and ill. His hemoglobin is 12.2 g/dL, and hematocrit is 37%; white blood cell count (WBC) is 15 x109/L, and platelets are 150 x109/L. You suspect diverticulitis, with a probable perforation. What study would best confirm the diagnosis? 1 Barium enema 2 Colonoscopy 3 CT scan of the abdomen 4 Flexible sigmoidoscopy 5 Sonogram
bradycardia with complete AV <3 block (3rd degree) EKG: AV dissociation (atrium + ventricles are contracting at different times and are not communicating) tx: permanent pacemaker (3rd degree <3 block can result in <3 failure)
A 57-year-old white man presents to the emergency department reporting a syncopal spell 1 hour ago. He was in his normal state of health until approximately 7 days ago, when he developed acute diarrhea that lasted 4 days and then resolved; however, for the past 3 days, he has had progressively worsening nausea and vomiting.Past medical history is significant for congestive heart failure secondary to non-ischemic cardiomyopathy, atrial fibrillation, hypertension, chronic renal insufficiency (with baseline creatinine 2.0), and benign prostatic hypertrophy. He has no known drug allergies but indicates an intolerance to diltiazem due to hypotension. Current medications include furosemide, digoxin, enalapril, carvedilol, and tamsulosin. On physical examination, temp 98.8°F, pulse 40 and slightly irregular, respirations 16, blood pressure 108/60. The patient appears alert with nausea and some vomiting during interview and exam. Bradycardia is present with slightly irregular heart rhythm. Normal S1 and S2 without obvious rub, murmur, or gallop. Lungs fields are clear without rales, rhonchi, or wheezes. The remainder of complete physical examination is unremarkable. EKG demonstrates complete AV dissociation present while CXR shows no acute disease and abdominal X-ray displays normal findings. What is the most likely cause of his syncope? -tx
delirium tremens (severe form of alcohol withdrawal) ALCOHOL WITHDRAWAL CAN BE FATAL! REQUIRES HOSPITALIZATION 6-24 hours: increased CNS activity like tremors, N/V, insomnia, tachy, HTN, seizures 2-5 days: delirium tremens -AMS -hallucinations -seizures treatment: -IV benzodiazepines (GABA agonist- CNS inhibition) diazepam, lorazepam -IV thiamine -IV Mg, B12, + folate
A 58 year old woman presents to the ER brought in by her daughter after her daughter discovered her mother confused, shaking at home. Patient's symptoms consistent of hallucination, tremors, nausea, anxiety, and seizure. Vitals show 120 bpm and 135/90 blood pressure. Patient admits she is a chronic alcoholic and tried to quit 'cold turkey' two days ago after drinking a fifth of vodka everyday for 10 years. What is the likely diagnosis? -presentation -treatment
Wernicke's Encephalopathy patho: thiamine deficiency usually d/t alcoholism presentation: triad -encephalopathy: disorientation, inattentiveness -ocular motor dysfunction: nystagmus, lateral rectus palsy -ataxia tx: IV thiamine
A 58-year-old man presents for further evaluation after being brought in by the local police because of an unstable gait and disheveled appearance; his breathalyzer test is 0.08 g/dL. On exam, the man is gaunt and clearly malnourished; he is semiconscious and unable to respond to questioning. His right elbow and knee are bruised, but he appears to have no other significant injuries; however, his eye movements are uncoordinated and he is unable to fixate his gaze. Vital signs are as follow:Blood pressure 138/89 mm Hg Pulse 109. What is the likely diagnosis? -patho -presentation -treatmet
pulmonary embolism presentation: -pleuritic chest pain -dyspnea -cough, hemoptysis -tachycardia -low grade fever -nausea diagnostics: -high suspicion: CT pulmonary angiogram -low suspicion: D-dimer, U/S doppler legs treatment: hemodynamically stable: anticoagulation -heparin acute phase -after acute phase: factor Xa or oral direct thrombin inhibitors hemodynamically unstable: fibrinolytics (tPA)
A 58-year-old woman with a past medical history of hypertension, hyperlipidemia, and breast cancer is being evaluated in the emergency room after presenting with a 2-hour history of acute-onset severe left-sided pleuritic chest pain. The pain is associated with feelings of anxiety, hemoptysis, shortness of breath, and nausea. She "feels warm" but denies other complaints. She endorses a 30 pack-year smoking history. Upon physical exam, she is found to be febrile, normotensive, tachypneic, diaphoretic, and in acute painful distress. There is perioral cyanosis and a pleural friction rub to the left lung fields; the remainder of the exam is normal. What is the likely diagnosis? -patho -presentation -diagnostics -treatment (stable vs. not)
aortic stenosis murmur: systolic crescendo-decrescendo murmur @ RUSB radiation to carotids -increases with increased preload: squatting, leg raise, expiration -decreases with decreased preload: standing, valsalva, hand grip, inspiration
A 59-year-old male with chest pain, dyspnea, and presyncope. The symptoms occurred after climbing a flight of stairs. He has a late systolic-ejection murmur heard in the second intercostal space (ICS) at the right sternal border with radiation to the carotids and the apex. The murmur is decreased with Valsalva maneuver. EKG is suggestive of LV hypertrophy. What is the likely diagnosis? -describe murmur (increase vs. decrease)
complete expulsion of ALL the POC, cervical os closes by time you will inspect
A 6 week G1P0 20 year old pregnant female presents to the clinic after she noticed blood all over her sheets this morning. She has been cramping ever since this morning as well and has continued to bleed. Cervical os is closed and you see expulsion of products of conception in the vaginal canal. What type of abortion classification is this?
Type 1 Diabetes patho: autoimmune destruction of beta islet cells →insulin deficiency presentation: you see polyuria, polydipsia, polyphagia in a young person think type 1 treatment: insulin dependent
A 6-year-old boy presents due to lethargy, polyuria, nocturnal enuresis, and polydipsia. His mother tells you that he complains of being tired and thirsty all of the time. You note that he has lost 5 pounds since his last visit 6 months ago. What is the likely diagnosis? -patho -presentation -treatment
Talar fracture
A patient was dropped during cheerleading practice yesterday and landed on her left foot. She immediately had swelling of the right foot behind her ankle and excruciating pain with plantar flexion and flexion. What is the likely diagnosis?
lung cancer via CT of chest negative chest xrays do not rule out lung cancer
If a patient with a history of smoking presents with hemopytsis what must be ruled out
thyroid storm (thyrotoxic crisis) presentation: -fever -dehydration -tachycardia -tachypnea -delirium -coma treatment: -PTU to suppress thyroid hormone formation and secretion -IV propranolol or esmolol: lessens adrenergic sx and reduce peripheral conversion T4 to T3 -IV dexamethasone: reduces peripheral conversion of T4 to T3 supportive: -fever management: acetaminophen + IV fluids -tx any precipitating factors like infection, trauma, radioactive iodine, withdrawal from antithyroid drugs
A 60 year old female is presenting to the ER with fever, delirium and vomiting. She denies any sick contacts or recent illness. The only change is that she was prescribe amiodarone two weeks ago for atrial fibrillation. Her vitals show tachycardia, tachypnea, and fever. Labs show very elevated T3 and T4. What is the likely diagnosis and treatment (including supportive)? -presentation -treatment
thyroid storm (thyrotoxic crisis) presentation: -fever -dehydration -tachycardia -tachypnea -delirium -coma treatment: -PTU to suppress thyroid hormone formation and secretion -IV propranolol or esmolol: lessens adrenergic sx and reduce peripheral conversion T4 to T3 -IV dexamethasone: reduces peripheral conversion of T4 to T3
A 60 year old female is presenting to the ER with fever, delirium and vomiting. She denies any sick contacts or recent illness. The only change is that she was prescribe amiodarone two weeks ago for atrial fibrillation. Her vitals show tachycardia, tachypnea, and fever. Labs show very elevated T3 and T4. What is the likely diagnosis? -presentation -treatment
lisfranc injury patho: forced plantarflexion injury causes separation of base of 1st and 2nd metatarsals leading to forefoot instability (tarsometatarsal) presentation: -severe midfoot pain -inability to wear bear -significant swelling + brusing of midfoot xray: bilateral WEIGHT BEARING -see widening of 1st and 2nd metatarsal space tx: prompt referral to ortho for surgery for open reduction + internal fixation (can lead to OA)
A 60 year old female with osteoporosis comes in with left foot pain. She missed a step while going down her staircase and now is unable to weight bear. She has significant midfoot pain. On PE you see bruising and swelling of the midfoot. X-ray shows widening between the 1st and 2nd metatarsal bases. What is the likely diagnosis? -patho -presentation -xray -tx
nephrogenic (diabetes insipidus) partial or complete insensitivity to ADH causes: -lithium** -hypokalemia (disrupts kidney concentrating ability) -hyperparathyroidism -iatrogenic: hysterectomy, abdominal surgery labs: -increased serum osmolarity -decreased urine osmolality + specific gravity treatment: -sodium restriction -HCTZ
A 60 year old patient is one month post-op from a total hysterectomy. She is presenting with polyuria, polydipsia and nocturia. She feels extremely lethargic. Her blood pressure is 97/60 and she has poor skin turgor. Labs are normal except sodium is 150. What is the likely diagnosis? -causes -presentation -labs -treatment
A. Broad-spectrum antibiotic: Sputum production is extremely variable from patient to patient, but any increase in sputum with a history of COPD reported by a patient must be regarded as potentially infectious and treated promptly.
A 60 year-old patient with COPD characteristic of emphysema presents with a cough and increased sputum production. The following information is noted: Temperature 100°F (37.8°C); Respiratory rate 20/min; Heart rate 88 beats/min; pH 7.44; PaO2 75 mmHg; PaCO2 40 mmHg; O2 saturation 92%. Physical examination is remarkable for increased AP diameter, diminished breath sounds without wheezes, rhonchi, or other signs of respiratory distress. Which of the following would be an appropriate treatment for this patient? A. Broad-spectrum antibiotic B. Admission to the hospital C. Oxygen at 6 L/min by nasal cannula D. Brief course of oral theophylline
retinal detachment patho: separation of retina. Causes include: -extreme myopia -trauma -inflammatory changes presentation: acute onset painless blurred vision → complete or partial monocular blindness -vertical curtain coming down -floaters or flashes funduscopic: asymmetric red reflex treatment: stay supine with head turned towards the side of the detached retina -ophthalmologist: pneumatic retinopexy → involves cryoretinopexy followed by injection of an air bubble in the vitreous
A 61 year-old male complains of "flashing lights behind my eye" followed by sudden loss of vision, stating that it was "like a curtain across my eye." He denies trauma. On funduscopic exam, the retina appears to be out of focus. What is the likely diagnosis? -patho -presentation -funduscopic -treatment
Misoprostol Besides NSAID use, this patient has several risk factors for gastric bleeding (a previous episode, advanced age, use of anticoagulants and low-dose aspirin). Therefore, prophylaxis against bleeding must be seriously considered. Prostaglandin analogues are the most effective drugs for reducing the risk of rebleeding.
A 62-year-old man passed a foul-smelling, tarry stool 1 week ago. 3 weeks ago, he began taking diclofenac because of worsening osteoarthritis. His past medical history includes primary hypertension, coronary heart disease, and 2 episodes of venous thrombosis in the calf. Besides diclofenac, his medications include warfarin, enalapril, low-dose aspirin, propranolol, and sublingual nitroglycerin. A screening colonoscopy performed 2 weeks ago was entirely normal. Vital signs are stable, and the physical examination is unremarkable. Upper endoscopy shows only pangastritis and erosions. What is the most effective way of reducing his risk of rebleeding? 1 Misoprostol 2 Omeprazole 3 Famotidine 4 Switch to valdecoxib 5 Take the diclofenac tablets with at least 250 ml of water
central retinal vein occlusion patho: Thrombus in ventral retinal vein → venous obstruction → retinal capillary nonperfusion + ischemia presentation: painless unilateral vision loss Fundoscopic exam -optic disc swelling -"blood + thunder" retinal hemorrhages and tortuous veins -cotton wool spots treatment: Intravitreal injection of VEGF inhibitors (stops growth of new blood vessels) , Laser surgery
A 62-year-old woman presents to the emergency department with acute unilateral loss of vision for 1 hour. Fundoscopic examination demonstrates vein dilation, intraretinal hemorrhages, and cotton-wool spots with optic disc swelling. What is the likely diagnosis? -patho -presentation -diagnostic -treatment
1. aspirin ASAP 162-325 mg non enteric coated 2. NTG for angina reperfusion therapy ASAP 1. Percutaneous coronary intervention first line; within 90 minutes is the goal 2. fibrinolytics (alteplase) if PCI not attainable
A 62-year-old woman presents with extreme fatigue and shortness of breath. The symptoms began about 24 hours ago and have progressively worsened within the last 4 hours. Vital signs on arrival are as follows: HR 90 beats per minute; BP 165/72 mm Hg; RR 16/min; SpO2 98% on 4L/min supplemental oxygen by nasal cannula. 12-lead ECG demonstrates ST-segment elevation of 2 mm in leads V4-V6. What treatment (including medications) should be initiated?
central retinal artery occlusion patho: Considered a form of stroke; occurs due to carotid artery atherosclerosis or cardiogenic embolism presentation: SUDDEN painless unilateral vision loss (amaurosis fugax →curtain coming down) Fundoscopic exam -pallor of optic disk -cherry red fovea -retinal arteries → "boxcar" segmentation (RBCs separate from serum) tx: OCULAR EMERGENCY! Revascularization ASAP. Can result in permanent blindness if circulation is not restored within 30-60 minutes. -reduce IOP -digital massage r/o cartoid artery stenosis w/ carotid ultraound (amaurosis fugax)
A 64-year-old African-American man presents to the emergency department after he went blind in his right eye "out of the blue" 20 minutes ago. There is no pain associated with his symptoms and he is not nauseated. Past medical history is positive for DMII for the past ten years. The pupil reaction on the left side is normal with pressure of 17mmHg. Right pupil evaluation reveals no reaction to light or accommodation with pressure of 20mmHg. Right eye ophthalmoscopy reveals arteriolar narrowing, vascular stasis, and "boxcar" pattern. What is the likely diagnosis? -patho -presentation -PE -treatment
left brain stem Although precise localization of neurologic deficits is often difficult, certain patterns are important to recognize. Brain stem involvement results in 'crossed findings' (e.g., right facial weakness and left arm weakness) because the lesions affect the brain stem nuclei directly (uncrossed) and the corticospinal tract as it is crossing to the opposite side of the body. Because of the unique anatomic arrangement of the brainstem, a unilateral lesion within the structure can cause 'crossed findings' in which ipsilateral dysfunction of 1 or more cranial nerves is associated with hemiplegia and hemisensory loss on the contralateral side of the body, as described in this case.
A 64-year-old man presents with difficulty speaking, trouble swallowing, and right arm weakness. Physical examination reveals dysarthria, tongue deviation to the right, paralysis of the left soft palate, left facial droop, left facial sensory loss, and right arm weakness. These findings suggest involvement in what area?
giant cell arteritis patho: Large and medium vessel granulomatous vasculitis of the extracranial branches of the carotid artery → inflamed temporal artery presentation: -unilateral headache + lot of side of face pain including claudication w/ mastication -vision changes (monocular vision loss, amaurosis fugax) -scalp tenderness diagnostics: Diagnose ⅗ criteria - Age >50 -new HA -temporal artery tenderness -ESR >50 -abnormal temporal artery bx tx: High dose steroids ASAP once GCA suspected to prevent blindness** Refer to ophthalmology + rheumatology
A 64-year-old woman presents to the ED with complaints of left-sided headache, low-grade fever, malaise, pain with chewing, and decreased vision in her left eye. What is the likely diagnosis? -patho -presentation -diagnostics -treatment
GERD patho: reflux of gastric contents presentation: -chest discomfort, sternal pain (similar to ACS) -sour taste -chronic cough -throat pain -nasal irritation -N/V MUST r/o ACS, GI bleed tx: start w/ H2RA blocker such as ranitidine, PPI omeprazole if necessary
A 65 year old male presents to the ED with worsening chest pain for the past 2 days. Describes that the pain is substernal, burning and worsens when he lays down. He has felt nauseated during this time and feels like he has to constantly clear his throat. Vitals, EKG, CBC, CMP, chest-xray are all normal. What is the likely diagnosis? -patho -presentation -diagnostics -treatment
acute decompensated heart failure triggers: FAILURES -Forgetting (not taking <3 failure meds) -Arrhythmias (A-fib) -Ischemia -Lifestyle (fluids, alcohol, salt) -Up-regulation (pregnancy, hyperthyroidism) -Renal Failure -Embolism -Stenosis (Aortic stenosis, RAS) tx: diuretics (loop diuretics) since the patient shows normal cardiac output but signs of congestion
A 65 year old patient with a history of coronary artery disease, 2 stent placements, and COPD presents to the ED with worsening dyspnea at rest for the past 24 hours and a productive cough with frothy sputum. On PE you hear a loud S3 and crackles. Chest xray shows pulmonary edema and cardiomegaly. Troponin levels are normal, EKG shows sinus tachycardia. BNP levels are elevated. Blood pressure is 135/85, he is not diaphoretic or pale. What is the likely diagnosis? -triggers -treatment
hypocalcemia causes (4) -serum alkalosis -hypoparathyroidism -hyperphosphatemia -hypovitaminosis D3 treatment: -replete Ca levels IV calcium gluconate or PO calcium carb
A 65 yeear old women is presenting with muscle cramps, fatigue, and abdominal pain that has been going on for a few weeks. She has also had tingling around her mouth. She has a history of Stage 4 CKD. You noticed unconscious wrist flexion when applying BP cuff to the patients arm and signs of tetany. What electrolyte abnormality does this patient likely have? -causes -treatmemt
peripheral artery disease presentation: -claudication** -pallor or cyanosis -lower extremitiy loss of hair -brittle nails -ulcers can occur diagnostics: -ankle-brachial index <0.9 -arteriography (gold standard) treatment: -antiplatelets, statins -definitive: arterial bypass
A 68 y/o female with a history of atrial fibrillation presents to the ED for recurrent pain in her lower legs during her morning walks that then alleviates with rest. On PE you notice there is a loss of hair on her lower legs, pallor and the skin feels cool. What is the likely diagnosis? -presentation -labs -treatment
septic arthritis patho: infection in the joint cavity. MCC S. aureus, N. gonorrhea sexually active pts, pseudomonas IVDU presentation: -MC site in adults = knees -MC site in kids = hips -swollen, warm, painful + tender joint with decreased ROM -fever, chills, myalgia diagnostics: -MUST get sample of synovial fluid. Analysis will show: WBC > 50,000, culture is usually +, if bacterial PMN will dominate -x-ray will only show soft tissue swelling treatment: MEDICAL EMERGENCY b/c can destroy joint -empiric therapy with ceftriaxone + vancomycin (2 weeks) followed by 4 weeks oral Abx -surgical debridement or drainage as necessary
A 68 year old male patient presents with an acute onset of left sided knee pain. He has a history of rheumatoid arthritis of both knees, he received a CSI in both knees prior to this pain. He said that he has been feeling "sick" with chills and muscle aches. On PE you see a swollen, warm, and tender to palpation knee joint. He is unable to full extend the joint. Arthrocentesis and analysis of synovial fluid shows 56,000 WBC/uL with 75% of them being PMNs. Synovial culture is positive for gram positive cocci in clusters. What is the likely diagnosis? -patho -presentation -diagnostics -treatment
ulnar shaft fracture direct blow to the forearm
If a person held their arm up to block being hit with a weapon (like a night stick), which bone would likely be affected?
Urge incontinence (overactive bladder) patho: overactivity of detrusor muscle MC cause of incontinence in elderly/nursing home patients presentation: -sudden urge to urinate -large volume loss -nocturnal wetting diagnostics: -PVR: normal or low -urodynamic studies: increased bladder contractions during the filling phase (stress = normal bladder contractions) treatment: -bladder training exercises to increase time in between voiding -Oxybutinin (anticholinergic) -Impramine (TCA)
A 68 year-old woman comes to the office for evaluation of urinary incontinence. For the past few months, she has had an intense urgency to urinate, followed by leakage of urine. What is the likely diagnosis? -patho -MCC -presentation -diagnostics -treatment
A. The patient's surgical history, presenting symptoms and abdominal imaging are consistent with small bowel obstruction. Peritoneal adhesions are the most common cause of small bowel obstruction.
A 68 year-old woman presents with intermittent crampy abdominal pain and vomiting of 6 hours' duration. She feels bloated but denies rectal bleeding. Examination of the abdomen reveals no palpable mass and you note an upper midline scar from previous abdominal surgery. An abdominal radiograph demonstrates dilated loops of small bowel with air-fluid levels. Which of the following is the most likely cause of this patient's symptoms? A. Adhesions B. Sigmoid volvulus C. Incisional hernia D. Intussusception
cholangitis patho: infection of biliary tract secondary to obstruction → biliary stasis + bacterial overgrowth (usually choledocolithiasis stone in CBD) presentation: -Charcots triad: RUQ pain, jaundice, fever -Reynold's pentad: Charcots + altered mental status + hypotension labs: -U/S (initial) -ERCP (gold standard) treatment: EMERGENCY tx (can be life threatening) -Cipro + metronidazole -ERCP remove stones, insert stent + repair sphincter -cholecystectomy (performed post-acute)
A 68-year-old male with acute onset of abdominal pain associated with fever and shaking chills. The patient is hypotensive and febrile with a temperature of 102.2 ° F. Although he is confused and disoriented, he complains of right upper quadrant pain during palpation of the abdomen. His sclerae are icteric and the skin is jaundiced. What is the likely diagnosis? -patho -presentation -lab -treatment
B Implanatable cardioverter-defibrillator (ICD)implantation
A 68-year-old man with a past medical history of diabetes mellitus type II, hyperlipidemia, myocardial infarction 1 year ago, and congestive heart failure with left ventricular ejection fraction of 35% is rushed to his local emergency room by his wife after he collapsed and became unresponsive at their residence. He admitted to her that he had been experiencing severe chest pain and pressure, fatigue, palpitations, diaphoresis, and lightheadedness for several minutes prior to his collapse. His present medications include aspirin, atorvastatin, lisinopril, glipizide, and carvedilol. Upon physical exam, he is found to have a blood pressure of 60/palpable, is pulseless, and has gasping respirations. His troponin T level was found to be elevated at 0.2 ng/ml, and troponin I level elevated and measured to be 0.25 ng/ml. The admission ECG revealed bizarre, irregular, random waveform, no clearly identifiable QRS complexes or P waves, and a wandering baseline.Following appropriate stabilization, What is best next step for this patient? A Prophylactic lidocaine B Implanatable cardioverter-defibrillator (ICD)implantation C Long-term metoprolol use D Percutaneous coronary intervention (PCI) E Discontinue aspirin, atorvastatin, and lisinopril
preseptal (periorbital) cellulitis patho: infection of the eyelid + periocular tissue anterior to orbital septum presentation: -unilateral ocular pain, eyelid erythema and edema -absence of proptosis, ophthalmoplegia (EOM weakness), ocular pain with EOM restriction diagnostics: clinical; if uncertain CT w/ contrast treatment: -oral abx + outpatient if >1 y/o -oral Clinda to cover for MRSA (PPP)
A 7 year old presents to the clinic with a 1 week history of red, swollen left eyelid and pain. PE shows normal EOM, orbit itself is not injected and there is no proptosis. Visual acuity is 20/20 in both eyes. Patient had a cold last week. What is the likely diagnosis? -patho -presentation -diagnostics -treatment
Strep pharyngitis patho: group A beta-hemolytic streptococcus associated symptoms can include: -headache -stomach-ache -nausea -vomiting diagnosis: definitive diagnosis is a throat culture -rapid test = low sensitivity; negative rapid result requires confirmation via culture treatment: -penicillin or amoxicillin
A 7 year old presents with an extremely painful sore throat. His mom said he has been unable to eat and is "out of it". Denies any cough, chest pain or abdominal pain. Vitals show 102 F and tachycardia. On PE you palatal petchiae, a bright red uvula and tonsillar exudates. There is also cervical lymphadenopathy. What is the likely diagnosis? -patho -associated sx -diagnosis -treatment
retropharyngeal abscess patho: deep space infection behind the pharynx presentation: -fever -throat pain -dysphagia -can lead to airway compromise diagnostics: -AP + Lateral Neck x-ray → retropharyngeal swelling at C4 vertebral body -CT scan of neck w/ contrast diagnostic tx: Can be life threatening + requires admission (esp kids) → Stat ENT consult + admission, IV broad spectrum antibiotics
A 7-year-old child presents to the ED with fever, neck pain, dysphagia, and a "duck-like" voice. AP and lateral x-ray shows retropharyngeal swelling at the C4 vertebral body. What is the likely diagnosis?
humeral head fracture worry about axillary + radial nerve xray: can show posterior fat pad/sail sign if it is near the elbow joint treatment: sugar tong splint (distal) and coaptation splint (shaft) with ortho follow up in 24-48 hours
A 70 y/o male presents to the emergency department with L shoulder pain after a falling off the sidewalk. He is unable to move the arm without pain and experiences tenderness to palpation. X-ray shows the following. What is the likely diagnosis? -nerve to worry about -treatment
Abdominal Aortic Aneurysm (AAA) risk factors: smoking, male presentation: -abdominal, flank or back pain -pulsatile abdominal mass -hypotension -syncope labs: ultrasound, CT angiography treatment: -surgical repair if >5.5 cm OR expands >0.6cm per. year -beta blockers (reduce arterial wall stress)
A 70 y/o male with a history of smoking presents to the emergency department with severe abdominal pain. On physical exam, he is hypotensive and has a tender pulsatile abdominal mass. What is the likely diagnosis? -presentation -diagnostics -treatment
posterior cord syndrome causes: many! -multiple sclerosis -trauma presentation: gait ataxia + paraesthesias -acute: muscle flaccidity and hyporeflexia (LMN) -chronic: muscle spasticity and hyperreflexia (UMN)
A patient with a history of multiple sclerosis is presenting with a MS flare. She is having difficulty walking due to gait ataxia, reports pins and needles down her legs. On PE her muscles are flaccid and hypo- reflexive. What type of spinal cord injury does she likely have?
toxic multinodular goiter (Plummer Dx) patho: focal hyperfunctioning follicular cells independent of TSH due to either a mutation of TSH receptor or chronic iodine deficiency 2nd MCC cause of hyperthyroidism, especially in the elderly presentation: NO eyes or skin changes (different from Graves) -thyroid: diffusely enlarged, palpable nodules and asymmetric labs: ↓ TSH, ↑ T3, T4 radioiodine uptake: multiples areas of different uptake (high and low patches) treatment: same as toxic adenoma -radioactive iodine ablation of thyroid gland, will need hormone replacement thereafter -if not RI, methimazole or PTU -propranolol for palpitations + HTN
A 70 year old female comes in with a 2 month hx of weight loss, insomnia, and jitteriness. She says that she gets hot very easily during this time and has noticed she has had diarrhea more frequently. PE is positive for tachycardia, hypertension and increased DTRs. Thyroid inspection shows palpable nodules and a diffusely enlarged thyroid. You order a radioiodine uptake which shows multiple patches of high and low uptake. What is the likely diagnosis? -patho -risk factors -presentation -labs -treatment
pseudogout aka calcium pyrophosphate dihydrate deposition disease patho: calcium pyrophosphate crystal deposition in articular tissues leading to inflammation (neutrophil infiltration) + erosion of bone presentation: most common site is the knee; unlike gout NO tophi, elderly + female is the usual patient population -severe joint pain, erythema, warmth, swelling and tenderness diagnostics: -must do arthrocentesis as the knee looks infected! shows positive birefringment rhomboid shaped crystals -xray: chonedrocalcinosis treatment: colchicine preferred for acute attacks + chronic dx -can also use NSAIDS and corticosteroids
A 70 year old female presents to the clinic with acute pain, swelling, and tenderness of her right knee. She denies any fever, chills, muscle weakness or weight loss. Arthrocentesis is performed and it shows positively birefringent rhomboid shaped calcium, pyrophosphate crystals. What is the likely diagnosis? -patho -presentation -diagnostics -treatment
pseudogout chonedrocalcinosis is pathognomonic for pseudogout; it is the calcification of cartilage (patches of white in joint space where it should be clear)
A 70 year old female presents with acute knee pain. According to this x-ray what is the likely diagnosis?
Essential tremor patho: autosomal dominant inherited dx risk factors: increase age, family hx, stress presentation: -intentional tremor worsens with voluntary movement, extreme heat (hot tub) and anxiety. Affects head, arms, hands which is why manual skills like handwriting are effected. Alcohol ingestion can alleviate sx -speech impairment since it effects the laryngeal muscles -NO other neuro symptoms tx: not usually needed -1st line: Propranolol helps control tremor (careful w/ bradycardia & asthmatics) -2nd line: Primidone (barbiturate)
A 70 year old male is presenting with a two month onset of impairment in performing ADLs, handwriting and speech. He attributed this to tremors of the arms and head that worsen with voluntary movement and alleviated with rest. He also noticed that the tremor improves after he has a glass of wine. On PE you notice his voice is very soft, when performing finger to nose the tremor increases as his finger approaches the nose. Denies any confusion, sensory changes or gait issues. What is the likely diagnosis? -patho -risk factors -presentation -tx
Dementia presentation: -short term memory loss -language difficulties like finding words, comprehension -visuospatial dysfunction: gets lost driving home -apathy -distractibility -apraxia: loss of learned behaviors like walking diagnostics: r/o delirium (glucose check, hx of drug use, stroke sx) MRI: cortex atrophy
A 70 year old male presents to the ED with police after walking in the middle of the road. He cannot remember where he was before, his name and his memories are scattered. He gets confused while talking and forgets words. On PE you notice he is frustrated by the difficulty he has initiating getting up from the chair and walking. What is the likely diagnosis? -presentation -diagnostics
pre-renal AKI (due to hypovolemia ) patho: decreased renal perfusion causes: -hypovolemia: diuretics, vomiting, diarrhea, blood loss, hyponatremia, <3 failure -NSAIDS, ACE -IV contrast diagnostics: ALL AKI → elevated SCr, BUN, decreased GFR -FeNa <1% -RUS: no hydronephrosis treatment: fluid resuscitation → then fix underlying cause
A 70 year old patient is in the hospital due to anemia caused by a chronic GI bleed. He has a history of hypertension and congestive <3 failure. Medications include Lasix and Lisinopril. In the hospital, he has developed polydipsia, decreased urine output and concentrated urine. PE is positive for poor skin turgor, tachycardia and peripheral edema. CMP shows SCr 2 and FeNa <1%. What is the likely diagnosis? -causes -diagnostics -treatment
acute diverticulitis patho: small herniations through the muscle layer of the colon → inflammation leading to microperforation + abscess formation presentation: altered bowel motility (constipation usually) diarrhea w/ N/V anorexia -LLQ pain -fever -Pain intermittent or constant -TTP -distension* w/ rebound + guarding if peritoneal irritation diagnostics: CT w/ contrast (fat stranding, bowel wall thickening) tx: most cases can be tx w/ just ABx: Cipro + Flagyl
A 70 year old patient presents to the ED with fever, left lower quadrant pain, bloody stool, and was struggling with constipation. On PE there is abdominal distension and tenderness to palpation on the left lower quadrant. CT shows soft tissue inflammation of the pericolic fat. What is the likely diagnosis? -patho -presentation -diagnostics -treatment
osteoarthritis x-ray changes associated with OA is "LOSS" L - Loss of joint space O - Osteophytes S - Subchondral sclerosis (increased density of the bone along the joint line) S - Subchondral cysts (fluid-filled holes in the bone)
A 70 year old patient presents with pain in the knees that worsens with activity. Based on the x-ray what is the likely diagnosis?
pericardial tamponade patho: elevated intrapericardial pressure (greater than 15 mm Hg), which restricts venous return and ventricular filling → hypotension + tachycardia causes: pericarditis, trauma, free wall post-MI, aortic dissection, cancer presentation: becks triad- diminished <3 sounds, hypotension + JVD -SOB, dyspnea diagnostics: -echo = diagnostic -EKG: electrical alternans -pulsus paradoxus on BP (<10 mmHg drop in SBP during inspiration) -CXR: water bottle shaped <3 tx: 500-1000 mL fluid bolus followed by pericardiocentesis
A 70-year-old man is in the ICU post-coronary artery bypass grafts. He is post-op day 9 and becomes both hypotensive and tachycardic. His blood pressure improves with 1 bolus of IV fluid. 30 minutes later, however, he becomes hypotensive again. His heart sounds are muffled and the chest tubes draining the pericardium have had decreased output over the past 3 hours. The patient is afebrile. What is the likely diagnosis? -patho -causes -presentation -diagnostics -treatment
pleural effusion patho: abnormal accumulation of fluid in the pleural space (not a disease itself but a sign of a disease) presentation: -dyspnea -pleuritic chest pain -cough PE: -dullness to percussion -decreased breath sounds -decreased tactile fremitus xray: blunting of costophrenic angle, white fluid in the lung tx: tx underlying dx; thoracentesis to drain the fluid if it is causing symptoms (i.e. in this patient). -chronic: pleurodesis (obliteration of pleural space)
A 72-year-old man presents with longstanding but increasing dyspnea, and a 3-day history of shortness of breath, coughing, unilateral sharp chest pain and mild temperature elevation. Pain is worse when he takes a deep inspiration and when he coughs. PMH is significant for a smoking history of 2 packs/day for 40 years (80 pack-years). The patient has other co-morbidities, including CAD, COPD, CHF, hypertension, and dyslipidemia. He denies recent travel, sick contacts, occupational exposure, and drug or alcohol use. Vital signs are BP 190/96, HR 140, RR 48, 02 68% on room air. Chest examination of the left posterior chest reveals a dull percussion note, inaudible bronchovesicular breath sounds, decreased tactile fremitus, a pleural friction rub on inspiration and diminished voice sounds below the sixth intercostal space. Significant bilateral lower extremity edema is also noted. A chest X-ray reveals blunting of the costophrenic angle. What is the likely diagnosis? -patho -presentation -diagnostics -treatment
aortic dissection risk factors: HTN, smoking, family hx, CT dx presentation: abrupt onset of tearing chest pain radiates to back -asymmetrical differences in BP and asymmetrical diminished distal pulses diagnostics: CT angiogram = gold standard treatment: -Type AI/II (proximal): SURGICAL EMERGENCY! -Type B/III (descending): beta blockers reduce wall stress
A 72-year-old man with known hypertension and high cholesterol presents with acute chest pain. He describes the pain as an excruciating tearing pain radiating to his back. His blood pressure in his right arm is 105/65 mm Hg and in his left arm 140/90 mm Hg. He also has a decreased pulse in his right arm. On auscultation of the chest, a III/VI early diastolic murmur is best heard at the left sternal border, third intercostal space. His EKG shows a sinus tachycardia with no acute changes. What is the most likely diagnosis -risk factors -presentation -diagnostics -treatment
cardiogenic shock (low CO, high PCWP + SVR) causes: acute MI, <3 failure, cardiac tamponade presentation: -hypotension <90 mmHg -tachycardia -weak peripheral pulses -cool extremities, cyanosis (increased SVR trying to get blood flow to <3) labs: elevated pulmonary capillary wedge pressure (failure LV) treatment: -FLUID resuscitation and pressors (dopamine)
A 75 y/o patient presents with altered mental status, cool extremities, and crackles in the lungs. He recently went off of his heart failure medication due to cost. Peripheral pulses are weak and he is tachycardic. Blood pressure is 84/50. What type of shock is the patient likely experiencing? -causes -presentation -labs -treatment
C. endotracheal intubation and mechanical ventilation This patient is in severe respiratory arrest with markedly impaired mental status; conventional mechanical ventilation is required
A 75 year-old man with a long history of COPD presents with acute onset of worsening dyspnea, increased productive cough, and marked agitation. While in the emergency department he becomes lethargic and obtunded. His ABG's reveal a PaO2 40 mmHg, PaCO2 65 mmHg, and arterial pH 7.25. Which of the following is the most appropriate management at this point? A. oxygen supplementation with a 100% non-rebreather mask B. noninvasive positive pressure ventilation (NIPPV) C. endotracheal intubation and mechanical ventilation D. emergency tracheostomy
intracerebral bleed patho: bleeding that occurs in the brain parenchyma risk factors: hypertension, alcohol consumption, AV malformation presentation: neuro symptoms increase within minutes to hours -headache -N/V -syncope -hemiparesis, hemiplegia (focal neurology sx) -AMS diagnostics: CT w/out contrast treatment: -reduced BP: IV labetalol or nicardapine -prevent increased ICP: raise head of bed, limit IV fluids, sedation
A 75-year-old African-American man has a past medical history that is significant for severe and uncontrolled hypertension; he is brought into the emergency room by his family due to a 30-minute history of change of mental status. The patient had just climbed stairs when he first developed a headache that has become progressive; it is associated with nausea, non-bilious vomiting, and unilateral upper and lower extremity numbness. His physical exam reveals an alert and oriented times zero patient. He is afebrile, with a blood pressure of 185/108 mm Hg, and there is nuchal rigidity. His neurological exam notes contralateral sensory loss, contralateral hemiparesis, gaze paresis, homonymous hemianopia, and miosis. A CT scan without contrast was performed and shows the following. What is the likely diagnosis? -patho -presentation -risk factors -diagnostics -treatment
Central Cord syndrome presentation: -hyperextension injuries, MVA, gunshot, cervical stenosis -UE>LE deficits: weakness, deficit of pain + temp "shawl" distribution -Retained proprioception + vibration
A 75-year-old man is involved in a motor vehicle accident and strikes his forehead on the windshield. He complains of neck pain and severe burning in his shoulders and arms. His physical examination reveals weakness of his upper extremities. What type of spinal cord injury does this patient have? -presentation
orbital (septal) cellulitis patho: infection of the orbit (fat, ocular muscles and nerve) usually arises secondarily from a sinus infection presentation: -pain especially with movement of EOM (leading to restriction of eye movement) -decreased visual acuity -proptosis (bulging of eye) -eye appears red, swollen diagnostics: CT with contrast as it helps establish the extent of infection within the orbit and sinuses and evaluate for subperiosteal abscess treatment: systemic broad spectrum antibiotics (PPP says Vanco + Ceftriaxone and admit patient)
A 8 year old presents to the ED with complaint of blurred vision and eye pain in the right eye.. The pain is so severe patient does not want to move eyes during examination of EOM. On PE you notice proptosis of the right eye as well as swelling and erythema surrounding it. Visual acuity is 20/20 in the left eye and 20/40 in the right. Patient had a rhinosinusitis infection 3 weeks prior to these symptoms. What is the likely diagnosis? -patho -presentation -diagnostics -treatment
Dupuytren's contracture diagnostic: tabletop test (pt cannot lay hand flat on table top) treatment: injected collagenease or steroid -fasciotomy if pt refractory to injections
A patient with alcoholic cirrhosis presents with painless nodules on the distal palmer crease. There is a fixed flexion deformity at the 4th MCP joint. What is the likely diagnosis & treatment?
non-selective beta blockers such as propranolol; these can exacerbate vasospasms (block B2 vasodilation of smooth muscle)
What medication class is contraindicated in patients with prinzmetal variant angina?
subdural hematoma causes: sequelae of head injury usually in the elderly or alcoholics. patho: usually arises from venous blood that leaks from torn/sheared cortical veins that bridge the subdural space. Bleeding occurs between the dura and arachnoid mater. In populations like the elderly or alcoholics the brain atrophies putting tension/stretch on the bridging veins making it easier for them to break. presentation: -No LOC (diff than epidural) -gradual onset of N/V HA -focal defects such as weakness, aphasia, facial droop diagnostics: -CT scan shows "crescent" shaped hemorrhage that appears concave and can cross suture lines/more diffuse bleeding treatment: -non operative if midline shift < 5mm and no sign of increased ICP -if midline shift > 5mm or increased ICP may do craniotomy or decompressive craniectomy
A 80 year old female comes into the ER with a three day history of headache following a fall she had when walking her dog. She reports no loss of consciousness after the event but has felt nauseated the past three days. On PE you see bruising on the L temporal region of scalp and she progressive weakness in her right upper extremity. What is the likely diagnosis/finding on CT scan? -causes -patho -presentation -diagnostics -treatment
normal pressure hydrocephalus patho: impaired CSF absorption after CNS injury like hemorrhage, TBI, meningitis, inflammation presentation: "Wet, wobbly, and wacky" -dementia/cognitive dx -urinary incontinence -ataxia: wide based shuffling gait, instability when turning and won't lift foot off floor when walking diagnosis: MRI: enlarged ventricles in absence of sulcal dilation -LP: normal CSF opening pressure (5-20 cmH2O) treatment: -LP can alleivate sx -ventriculoperitoneal shunt definitive tx
A 80 year old man is presenting with three month history of gradual worsening behavior changes, difficulty performing activities of daily living. He has also developed urinary incontinence and on PE you notice a wide based shuffling gait where he does not lift foot off of floor. Denies significant memory loss or aphasia. Lumbar puncture shows normal opening pressure (<20 cmH2O). What is the likely diagnosis? -patho -presentation -diagnosis -treatment
myxedema coma presentation: -hypothermia -bradycardia -altered mental status -swollen tongue + pharynx -sx of hypothyroidism treatment: -IV levothyroxine (high dose) -IV hydrocortisone (usually this dx state coexists with adrenal suppression; tx w/ HC until it is ruled out) -as stated in vignette; tx hyponatremia or other lab abnormalities + heat pt up symptoms and TSH improve within 24 hours
A 80 year old patient comes into the ER with altered mental status and dizziness. Her vitals show temp of 94.4 F, HR of 52 bpm, SaO2 92%. On PE you see a swollen tongue, pallor, puffy face and non-pitting pretibial edema. CMP shows hyponatremia and elevated SCr. Her daughter describes that her mother was out in the snow walking their dog before this all happened. You warm her up and treat with IV 0.9% saline but her symptoms are persisting. TSH is elevated and T3/T4 are low. What is the likely diagnosis and treatment?
abruptio placentae patho: premature seperation of implanted placenta after 20 weeks gestational age but before infant is delivered. presentation: MCC 3rd trimester bleeding* -sudden onset of abdominal pan + excessive vaginal bleeding -uterine tenderness, back pain or cramping -non-reassuring fetal status (fetal distress, bradycardia) -firm uterus imaging: U/S may see hemorrhage -check blood work: CBC, ABO/Rh, coagulation studies treatment: 1. stabilize mom hemodynamically w/ fluids, packed RBCs. 2. deliver the baby either vaginally or emergent C section 3. corticosteroids enhance fetal lung maturity
A G1P1 30 week pregnant 29 year old female presents to the ED with a sudden onset of abdominal pain and excessive vaginal bleeding. She is very worried. In the ED CBC shows normocytic anemia, hematocrit is low and PT/INR is elevated. Vitals show hypotension, tachycardia. She has a history of hypertension and smoking. What is the likely diagnosis? -patho -presentation -diagnostics -treatment
medial collateral ligament patho: valgus force to the lateral aspect of knee "Mucho Gusto" medial = valgus PE: positive valgus stress test (++ lax) *note in this scenario he is also at risk for an ACL injury tx: rarely needs surgery, RICE brace
A football player plants his foot and is hit by an opponent on the lateral aspect of his knee. What injury did he likely sustain? -patho -PE -tx
(c) This person has increasing respiratory failure as indicated by the raising PaCO2 levels. Intubation is required at this time
A patient with severe COPD presents to the Emergency Department with a 3 day history of increasing shortness of50breath with exertion and cough productive of purulent sputum. An arterial blood gas reveals a pH of 7.25, PaCO2 of 70 mmHg and PaO2 of 50 mmHg. He is started on albuterol nebulizer, nasal oxygen at 2 liters per minute, and an IV is started. After one hour of treatment, his arterial blood gas now reveals a pH of 7.15, PaCO2 100 mmHg and PaO2 of 70 mmHg. Which of the following is the most appropriate next step in his treatment? A. Decrease the oxygen flow rate. B. Administer oral corticosteroids. C. Intubate the patient. D. Administer salmeterol (Serevent)
acute bronchiolitis presentation: common in winter months -acute URI: wheezing, stridor, cough -upper respiratory infection prodome: fever, coryza, cough, hoarseness -irritability; poor feeding -myalgia, malaise, headache work up: -rapid viral diagnostic swab (if tx will change, like in RSV) -CXR non specific treatment: -support (suction, hydration) -supplemental O2 -ribivarin if RSV hospitalize: tachypnea with feeding difficulties, visible retractions, O2 sat <95%
A mom comes in very concerned about her 18 month old who recently recovered from a viral sinus infection. She noticed her daughter started wheezing and grunting, it seems like she cannot get air in. She is not feeding well and coughs. CXR rules out pneumonia. What is the likely diagnosis? -patho -work up -treatment -when to hospitalize
barotrauma patho: Damage to the tympanic membrane can occur with sudden pressure changes (eg, flying, diving, decompression, hyperbaric oxygen). tx: supportive with anti-inflammatories -pseudoephedrine prophylaxis (do NOT use >3 days) -autoinsufflation (chewing gum, yawning)
A patient comes to the ear with right sided ear pain, fullness and hearing loss after going cliff diving. He noticed some blood coming out of his R ear on his way to the ER. On physical exam, the tympanic membrane has a perforation on the margin. What is the likely cause of his injury?
Bankart lesion injury of the anterior (inferior) glenoid labrum following a dislocated shoulder
A patient has sustained a anterior shoulder dislocation. Which associated injury is this xray presenting?
Hill sachs lesion (dent in humeral head) compression chondral injury of the posterior superior humeral head following impaction from glenoid
A patient has sustained a anterior shoulder dislocation. Which associated injury is this xray presenting?
work up: CT or MRI of brain to r/o central pathology DDX: -diabetes (elevated glucose destroys brain) -nerve root (spinal cord impingement, infection) -central: multiple sclerosis, CVA
A patient is presenting to the ER with new onset of numbness and paresthesias. What should the work up be? What are possible ddx?
Acute cholecystitis 90% of cases due to gallstones presentation: -RUQ pain after high fat meal -RUQ pain that radiates to R shoulder (if d/t gallstones, known as Boas sign) -N/V loss of appetite -Murphy's sign + labs: -CBC, left shift -LFTS, Amylase, Lipase should be normal -ultrasound ' -HIDA gold standard if u/s non-conclusive treatment: -NPO, IV fluids, Antibiotics: Metro + ceftriaxone followed by cholecystectomy (24-48 hours)
A patient is presenting with an acute onset of pain for the past two days. It is localized to the right upper quadrant and radiates to the right shoulder. Patient also feels nauseated and has a loss of appetite. Their abdomen is extremely rigid and tender. Positive Murphy's sign. What is the likely diagnosis? -presentation -diagnostics -treatment
coccidioides (valley fever) treatment: fluconazole/itraconazole
A patient on chronic steroid treatment presents with a month long cough that has not responded to steroids, inhalers, or ABx. They are not presenting with any other symptoms. They recently moved to Arizona and it has been windier than usual. What is the likely diagnosis & treatment?
A) The black widow spider (Latrodectus) is found in many areas of the United States. Its bite produces immediate pain and pinprick sensations that soon encompass the entire extremity. Erythema of the bitten area develops usually within 1 hour and in about half of the cases quickly evolves into a target pattern. Patients frequently complain of cramp-like spasms in the large muscle groups. The physical examination rarely exhibits muscle rigidity, and serum creatine kinase concentrations usually are not elevated significantly
A patient presents to the ED after being bitten by an unknown "insect" while camping. The pain began as a pinprick sensation at the bite site and spread quickly to include the entire bitten extremity. The bite wound became erythematous 45 minutes after the bite. The bite evolved into a target lesion and the patient complains of muscle cramp-like spasms in the large muscle groups. Which of the following is the most likely cause? (A) Black widow spider (B) Brown recluse spider (C) Hobo spider (D) Scorpion (E) Tarantula
(B) The answer is 45% burn. The rule of nines to estimate percentage of burns is as follows: head 9%, anterior trunk 18%, posterior trunk 18%, each leg 18%, each arm 9%, groin 1%, perineum 1%.
A patient presents to the ED after being trapped in a house fire. The patient suffered partial thickness burns over the entire anterior chest and abdomen, entire right arm, and the entire right leg. Using the rule of nines, what is the estimated percentage of burn? (A) 36% (B) 45% (C) 48% (D) 54% (E) 72%
work up: -CT of head or orbit (depending on type of fracture) -check intraocular pressure (BB or carbonic anhydrase inhibitors if elevated) tx: blood is reabsorbed over days-weeks -elevate bed at night to 30 degrees -NO NSAIDS (increase bleeding) can use acetaminophen
A patient presents with blood in the anterior chamber of the eye following blunt trauma (punched in the face). What should his work up and treatment consist of?
can disseminate and cause meningitis treatment: amphotericin B
A patient presents with cryptococcus lung infection. What is the concern if this is left untreated? Treatment?
post-renal AKI causes: obstruction of outflow -BPH -prostate cancer -urolithiasis -underactive bladder patho: urine back flow into the nephron diagnostics: -FeNa <1% -U/S: hydronephrosis -UA: if any RBC present they will be isomorphic treatment: treat/remove the obstruction
A patient was diagnosed with a 6 mm kidney stone a month ago. He has still not passed it and came in because he has noticed a decrease in urine output. Renal ultrasound shows hydronephrosis. What is the likely diagnosis? -causes -patho -diagnostics -treatment
nephrolithiasis presentation: -flank pain, constant cannot get comfortable -hematuria (microscopic or gross) -voiding changes: decreased output or increased urgency (UTI sx) -N/V/guarding diagnostics: assess kidney fx w/ CMP -U/A rule out infection -CT scan if its a first time stone (no contrast) -U/S tx: -NSAIDS, opioids -tamsulosin: help w/ urinary retention - <5 mm can pass on own - >5 mm surgical intervention
A previously healthy 27-year-old man has had left flank pain radiating to his left groin for the past 2 hours. His urine output is also decreased. He denies hematuria, dysuria, penile discharge, and pyuria. He has had no trauma. He takes no medications. His temperature is 98.6 Fahrenheit, his pulse is 95 beats per minute, and his blood pressure is 150/95 mm Hg. He has left flank tenderness. The remainder of his exam is normal.His urinalysis shows: pH 6 No white cells 10 red blood cells/high powered field No nitrate, leukocyte esterase, or bacteria Multiple needle shaped crystals. What is the likely diagnosis? -presentation -diagnostics -treatment
lateral collateral ligament tear patho: varus force to the medial aspect of the knee "Leaky pipes Rust" lateral = varus PE: positive varus stress test (++ lax) tx: rarely needs surgery, RICE brace
A soccer player comes in for left knee pain that followed an injury where her left leg was planted and a player hit the medial aspect of her left knee. What injury did she likely sustain? -patho -PE
anterior shoulder dislocation patho: FOOSH typically MOI presentation: arm is abducted and externally rotated -"squaring" of shoulder: loss of rounded appearance of humeral head and acromion is sharp -axillary nerve injury can occur, check for numbness and tingling of lateral shoulder treatment: reduce → sling + swath → PT
An 80-year-old woman arrives at the emergency room with severe right shoulder pain and immobility. She fell down the steps outside her house and landed on her right side two hours prior to presentation. On exam, her right arm is abducted and externally rotated. She has decreased sensation to touch over the lateral aspect of her right shoulder. X-ray shows the following. What is the likely diagnosis? -patho -presentation -treatment
felon
An abscess in the tip of the finger is known as a
thumb print sign neck x-ray sign of epiglottitis
An unvaccinated child presents to the ED with drooling, stridor, and hypoxemia. CXR shows the following. What is the likely diagnosis?
Erythromycin promotes gastric emptying based upon its ability to be an agonist of motilin receptors
What medication is administered prior to endoscopy to promote gastric emptying and improve gastric visualization?
ACE inhibitors
What medication is renoprotective in chronic glomerulonephritis/CKD
malignant otitis externa → spread of the infection to the skull base and resulting in osteomyelitis this is a LIFE THREATENING condition and needs to be evaluated EMERGENTLY! suspect if patient is not getting better, worsening sx like fever, lethargy, CN palsies
If an immunocompromised or uncontrolled diabetic patient develops otitis externa, what condition are we worried about?
urinalysis to r/o a UTI
If someone is presenting with any type of urinary incontinence what initial test should be completed?
PO challenge with GI cocktail -maalox -viscous lidocaine -droperidol
If someone presents with vomiting in ER, what should be done prior to them being discharged?
-ABx: fluroquinolones, aminoglycosides, macrolides -Cardiac: BB, CCB, statins -muscle relaxants
What medications should be AVOIDED in patients with myasthenia gravis as they exacerbate MG
once a month for 5 months starting in November pts: IMC, premature infants, neuromuscular dx
What populations receive palivizumab prophylaxis for RSV?
asystole and pulseless EA protocol: -Compressions -1 mg Epi ASAP, then Q 3-5 minutes with compressions -Check rhythm Q 3-5 mins- shockable now?
ACLS: -which rhythms are not shockable rhythms -non-shockable rhythm protocol
shockable: pulseless V-tach + V-fib start compressions → Shock → Shock → 1 mg Epi → Shock → 300 mg Amio → Shock → 1 mg Epi → Shock → 150 mg Amio → (Shock → Shock → 1 mg Epi → repeat this portion)
ACLS: -which rhythms are shockable -shockable rhythm protocol
AML: blasts + auer rods (PBS) + >20% blasts in bone marrow CML: WBC > 100,000, asymptomatic until patient has a blastic crisis (acute leukemia) -diagnostic: Philadelphia chromosome (translocation X 9 and 22)
AML vs. CML
check for blood clots using echo
What should you always do before you cardiovert a patient?
rotate laterally against resistence
What test assesses infraspinatus/teres minor muscle of rotator cuff?
life off test (rotate medially against resistance)
What test assesses subscapularis muscle of rotator cuff?
Boutonniere deformity PIP flexion + DIP hyperextension d/t jammed finger splint in PIP extension 4-6 weeks
What type of x-ray finding is this indicative of?
tetanus shot
What vaccine should every burn patient receive prophylactically?
if it persists for > 2 years of life
When should a patient be referred to surgery for an umbilical hernia?
a fracture until proven otherwise
ANY extremity injury presenting with ecchymosis is...
low cardiac output: -cool extremities -weak pulse -low SBP -AMS -reduced urine output tx→ inotropes (dobutamine, milrinone, epinephrine) congestion: -S3 -pulmonary crackles -Cheyne-Strokes: deep/fast breathing cycles w/ apnea -JVD (> 8cm) -edema tx→ diuretics- loop (furosemide, bumetanide)
Acute decompensated <3 failure -signs of low cardiac output -signs of congestion -tx for each
MOA: agonists adenosine 1 receptors in the AV node → slows conduction time and interrupts re-entry pathway in SVT dose: 6 mg rapid push → 12 mg if ineffective contrainidcations: wolf parkinson white b/c if the AV node is blocked there is no slowing down the conduction in the accessory pathway -also: bronchospastic dx, afib/flutter
Adenosine -MOA -dose -contraindications
patients with renal insufficiency (cannot use factor Xa or direct thrombin inhibitors)
When should a patient be treated with warfarin for a pulmonary embolism?
1. ABC 2. signs of ICP? (i.e. fixed dilated pupil) contact neurosurgery ASAP 3. MISTO- potential causes -Metabolic: electrolytes, glucose (always check blood sugar), vitamin deficiencies -Infection: meningitis, UTI, pneumonia, sepsis -Structural/Seizure: stroke, hemorrhage, tumor -Toxins: medication, drug withdrawal or overdose -Oxygenation: low O2/high CO2- pneumothorax, COPD
Altered mental status: approach (steps 1-3)
diabetic ketoacidosis patho: increased insulin requirements → leads to shortage → body burns excess fat + fatty acids → ketone accumulation presentation: younger pts with type I DM -polyuria, polydipsia -weakness -N/V -fatigue -abdominal pain PE: tachycardia, Kussmaul's respiration, decreased skin turgor, hypotension, fruit breath labs: -plasma glucose: >250 -pH: <7.3 -bicarb: <18 -plasma + urine ketones treatment: -IV FLUIDS! -check potassium before give insulin -insulin (switches body from catabolic to anabolic state) -Bicarb if severely acidotic
An 18-year-old woman with Type I DM presents with nausea, vomiting, drowsiness, and abdominal pain. She has Kussmaul respirations, ketotic breath, dry tongue, and loss of skin turgor. What is the likely diagnosis? -patho -presentation -labs -treatment
incomplete -cervical os open -passage of some POC but not all (some remains in uterus)
An 8 week G1P0 26 year old female presents to the clinic after she noticed small amounts of vaginal bleeding and cramping for the past day. Transvaginal ultrasound shows dilated cervical os and that there are still some products of conception in the uterus. What type of abortion classification is this?
middle cerebral artery (MC site)
Where is the location of a stroke if the patient is presenting with: -aphasia -visual neglect + gaze preference (eyes deviate to side of lesion) -hemiplegia + hemisensory loss (> face/arm)
basilar or posterior circulation
Where is the location of a stroke if the patient is presenting with: -coma -apnea -drop attack -vertigo -visual changes -cranial nerve palsies
1. apply direct pressure 10-15 minutes 2. short acting topical decongestants (phenylephrine, Afrin) 3. anterior nasal packing → MUST tx with ABx to prevent toxic shock syndrome 4. if able to visualize bleeding source: cauterize
Anterior epistaxis treatment (initial → step up)
RLQ pain with internal rotation of hip
Appendicitis: Obturator sign
RLQ pain with hip extension
Appendicitis: Psoas sign
RLQ pain w/ palpation LLQ
Appendicitis: Rovsing's sign
mild intermittent: <2 x a week or <2 nighttime sx a month -SABA prn mild persistent: >2x a week or 3-4 nighttime sx a month -low dose ICS moderate persistent: daily sx or > 1x a week nighttime sx -low-medium dose ICS +/- LABA severe: multiple sx a day and nightly sx -high dose ICS + LABA
Asthma classifications -mild intermittent -mild persistent -moderate persistent -severe persistent
c4 or higher C3-C5 = phrenic nerve cervical cord injuries = quadriplegia MC site of trauma injuries due to it's flexibility
At what level of the spinal cord can a spinal cord injury result in respiratory paralysis?
anterior cerebral artery
Where is the location of a stroke if the patient is presenting with: -leg paresis -urinary incontinence
The (left) anterior cerebral artery supplies motor and sensory innervation to the legs, and occlusion may result in paralysis of the contralateral foot and leg with a lesser degree of involvement in the contralateral arm.
Where is the location of stroke if there is paralysis of the right foot and leg?
L5-S1 b/c junction between mobile and non-mobile spine
Where is the most common location of lumbar disc herniation?
3rd metatarsal
Where is the most common stress fracture in the foot/ankle in athletes or military members?
-antiplatelets -anticoagulation (LMWH or UFH) -beta blockers -statins <3 failure sx too: ACE-I, aldosterone antagonists
Which anti-ischemic medications prove to have a mortality benefit in patients with acute coronary syndrome (after reperfusion therapy)
Chvostek sign is a more sensitive test (few false negatives) Trousseau: more specific (few false positives)
Which is more sensitive for hypocalcemia- Chovetseks or Trousseau?
1. Primary spontaneous: atraumatic, no underlying lung dx. Usually due to bleb rupture, tall thin men 20-40 y/o + smokers tx → small, <3cm from chest wall + lung, observation + supplemental O2. Large: needle decompression followed by chest tube 2. secondary spontaneous: underlying lung dx such as COPD or asthma tx→ pt is stable: Chest tube or catheter thoracostomy + admit 3. trauma: iatrogenic (CPR, thoracentesis), car accidents. tx→ if hemothorax chest tube required 4. tension: we know her tx → needle aspiration followed by chest tube thoracostomy.
Describe the different types of pneumothorax + their treatment
affects epidermis and dermis, characterized by blistering Superficial partial: epidermis + papillary dermis Red, blistered, PAINFUL skin, blanches, hair singed Deep partial: epidermis + deeper portion of dermis (reticular) -Pale, blistered, decreased sensitivity to pain but it is still present (nerve endings damaged); Swelling of skin tx: Usually requires dressings and topical antibiotics.
Burns: 2nd degree -layers -presentation -treatment
layers: full thickness (burns through epidermis + dermis) presentation: Waxy, white, leathery, black skin Painless, NO blanching, lack of blistering and lack of sensation tx: May need skin graft
Burns: 3rd degree -layers -presentation -treatment
layer: ALL the way through skin into underlying fat, muscle, or bone presentation: Skin is black, charred, dry. Painless or loss of capillary refill tx: Requires tissue reconstruction, debridement & skin graft
Burns: 4th degree -layers -presentation -treatment
layer: epidermis only; superficial thickness presentation: -erythema , painful, tender, hypersensitive (sunburn), dry appearing with NO blisters -Capillary refill intact (blanches w/ pressure) treatment: heals w/in 7 days, no scarring
Burns: 1st degree -layers -presentation -treatment
+ straight leg raise ALP (L4, L5, S1) L4: Anterior thigh pain + sensory changes to medial ankle -weakness of ankle dorsiflexion -loss of knee jerk reflex L5: Lateral thigh/leg, hip, groin, dorsum of foot -weakness: big toe extension + dorsiflexion (walking on heels harder than toes) -reflexes usually normal S1: Posterior leg/calf/gluteus, plantar surface of foot -weakness of plantar flexion (walking on toes harder than heels) -LOSS of ankle reflex
Describe the findings associated with the following disc herniation locations -L4 -L5 -S1
>5 mm: high risk factors: IMC patients (HIV, immunosuppressant drugs, close contact w/ people w/ infectious TB) >10 mm: <4 y/o, some risk factors: hospitals and other healthcare workers, IVDU, homeless shelter, prison, diabetes, cxr >15 mm: no risk factors
Describe the following + TB PPD tests - >5 mm - >10 mm - >15 mm
-Gallstones (MCC acute) -Ethanol (MCC chronic) -Trauma -Steroids -Mumps (kids) -Autoimmune dx -Scorpion Stings -HyperCa and hyperLipid -ERCP -Drugs
Causes of acute pancreatitis: GET SMASHED -MCC acute vs. chronic
A. Two hours after the onset of an acute myocardial infarction, the cardiac enzymes would most commonly demonstrate a normal troponin and CK-MB levels. -myoglobin: 1-4 hours, peak at 12 and return to baseline within 24 hours -Cardiac troponin I: appears 2-4 hours, peaks at 12-24 and lasts for 7-10 days -CKMB: appears 4-6 hours peaks 12-24 and returns to normal at 48-72 hours
Cardiac enzymes in a patient with a 2-hour history of chest pain secondary to an acute myocardial infarction would commonly demonstrate which of the following findings? (A) normal creatine kinase (CK-MB) and troponin I levels (B) elevated troponin I and normal CK-MB levels (C) elevated CK-MB and normal troponin I levels (D) normal myoglobin with elevated CKMB levels (E) elevated myoglobin, troponin I, and CK-MB levels
age: 1 point for 3-14, 0 points 15-44, -1 >45 tonsilar exudates: +1 cervical LAD swelling +1 temp >100.4/38 +1 cough absent: +1 (present 0) 0-1: do nothing 2 points: optional tx 3-5: definitely tx for strep w/ penicillin
Centor Criteria for Strep -criteria -0-1 points -2 points -3-5 points
Primary active TB may appear on chest X-ray as hilar lymphadenopathy, pleural effusions, and perihilar or right-sided lower and upper lobe pulmonary consolidations. Patchy areas of consolidation of the apical and posterior segments of the upper lobes indicate reactivation TB
Chest x-ray for primary vs. reactivation TB
Non-selective BB: decrease splanchnic blood flow and reduce portal pressure. Decrease cardiac output due to B1 block, decrease splanchinic blood flow via B2 block agents → propranolol, carvedilol, nadalol EBL: controls bleeding, as soon as hemorrhage suspected start octreotide and tx with antibiotics after EBL for 7 days (cipro, ceftriaxone) TIPS: transjugular intrahepatic portosystem shunt -indicated if patient has recurrent variceal hemorrhages despite tx. Shunt connects portal and hepatic veins to reduce pressure and bleeding risk. -HOWEVER not filtering blood effectively so increases risk for hepatic encephalopathy; major risk factors are: age, existing HE (pre-TIPS), and high MELD score -bridge to transplant
Cirrhosis: Treatment for Varices -Non-selective BB -Endoscopic band ligation -TIPS
MC complication of cirrhosis tx: -salt restriction -diuretics (furosemide, spironolactone) -paracentesis if tense ascites, early satiety, SOB
Cirrhosis: ascites -treatment
A. 1 random plasma glucose >200 B. x2 fasting glucose >126 C. A1C >6.5%
Criteria for diabetes mellitus (3 different options)
cervical sprain: last 18+ months presentation: neck pain, paraspinral muscle tenderness, Spurling + tx: soft cervical collar 2-3 days, RICE thoracic/lumbar: MC cause of back pain due to lifting, twisting or strenuous activity presentation: stiffness, difficultiy bending NO pain below knees or neuro changes tx: bed rest <2 days, NSAIDS + muscle relaxants (cyclobenzaprine)
Describe the presentation + treatment for cervical and lumbar back strain
decreased libido/sexual dysfunction (BPH tx)
Common ADR for 5-alpha reductase inhibitors such as finasteride
flushing* headaches, dyspepsia, nasal congestion, back pain, myalgias, sinusitis
Common ADR for PDE-5 inhibitors such as tadalifil
-scopolamine patch -dexamethasone -ondasetron rescue: prochlorperazine, droperidol
Commonly prescribes antiemetics
1. increase fiber 20-25 mg per day 2. bulk forming laxatives (first line): psyllium, methylcellulos 3. osmotic laxatives: PEG, milk of Mg 4. stimulant laxatives
Constipation treatment
rifampin: orange body fluids isoniazid: peripheral neuropathy; give with pyridozine (B6) pyrazinamide: hyperuricemia (gout) ethambutol: optic neuritis, red-green blindness ALL are hepatotoxic must get baseline labs
Describe the side effects of the following TB drugs -rifampin -isoniazid -pyrazinamide -ethambutol
PTU has a BBW of acute liver injury, therefore have to be choosy when we are using it -1st trimester of pregnancy (methimazole is teratogenic in 1st trimester) -thyroid storm -refractory to methimazole ADRS of both: agranulocytosis, reduced neutrophils putting pt at risk of infection breastfeeding: methimazole
Describe thioamides, when would we use methimazole versus PTU? ADRS?
tonic: extreme rigidity then LOC clonic: loss of control of bodily function, pt jerks
Describe tonic vs clonic seizures
Flumazenil competitively blocks the effects of benzodiazepines on GABAnergic pathway-mediated inhibitors in the central nervous system (CNS). benzos: CNS depression -disinhibition -depression: slurred speech, somnolence, ataxia -labile mood: erratic behavior
Which of the following drugs represents the most appropriate antidotal agent for benzodiazepine overdose? (A) Activated charcoal (B) Flumazenil (C) Flutamide (D) Ketamine (E) Naloxone
-anxiety -electrolyte abnormalities (hypoK, hypoMg) -hyperthyroidism -ischemic <3 dx -stimulant drugs (rx or recreational) -pheochromocytoma -hypoglycemia -MVP -atrial fibrillation -WPW, SVT -sick sinus syndrome
DDX for patient experiencing palpitations
drop >20 mmHg systolic, 10 mmHg diastolic or 15 BPM increase in pulses 2-5 minutes after changing from supine to standing
Define orthostatic hypotension
Nonstress testing - Records movement, heartbeat, and contractions. It notes changes in heart rhythm when the baby goes from resting to moving, or during contractions if the mother is in labor (non-invasive) GOOD- Reactive NST - > 2 accelerations in 20 minutes defined by increased fetal heart rate of at least 15 bpm from baseline lasting > 15 seconds, indicates fetal well being BAD - Nonreactive NST - no fetal heart rate accelerations or < 15 bpm increase lasting < 15 seconds, if this is the case then get a contraction stress test Contraction stress test - measures fetal response to stress at times of uterus contraction GOOD - Negative CST - No late decelerations in the presence of 2 contractions in 10 minutes, indicates fetal wellbeing, repeat CST as needed BAD - Positive CST - Repetitive late decelerations in the presence of 2 contractions in 10 minutes, worrisome especially if nonreactive NST, prompt delivery
Describe Non-stress testing vs. Contraction stress testing for monitoring fetal distress
SPRAIN 1st: fibers of ligament are stretched but intact 2nd: moderate: tear of a ligament 3rd: complete rupture of ligament, sometimes avulses a piece of the bone STRAIN 1st: little tissue tearing, pain with full ROM 2nd: torn muscle or tendon. Limited ROM with pain 3rd: limited or NO ROM, severe acute pain
Describe a 1st, 2nd, and 3rd degree sprain vs. strain
do NOT adminster antihypertensives until systolic BP >220 or diastolic >120 -lower BP by 15% in first 24 hours BP has to be <185/110 for thrombolytics (give Labetalol)
Describe blood pressure control in the event of a ischemic stroke
partial: affects portion of brain, can either be -simple: NO consciousness impairment -complex: consciousness impaired (amnesia follows) associated sx like automatisms (lip smacking), N/V, auras, AMS tx: phenytoin or carbamezapine
Describe partial seizures -simple vs. complex -treatment
cardiac: <3 failing as a pump (CHF, MI, arrhythmias) pulm: lungs cannot move blood so fluid is leaking out (pulmonary HTN, COPD) obesity: belly fat goes up into lungs making space smaller but when they stand do not experience SOB
Describe patho/causes of orthopnea
premature atrial contraction (beat 4)
Determine the EKG
Wolff-Parkinson-White patho: accessory pathway connecting atria to ventricles bypassing AV node EKG triad: -short pr interval -can have wide QRS complex -slurred upstroke of QRS (delta wave) treatment: -procainamide -unstable: cardioversion -definitive tx: radiofreqeuncy ablation of accessory pathway
Determine the EKG -patho -EKG -treatment
ventricular fibrillation patho: ventricles quiver and do not contract in a coordinated way →no blood pumped from heart EKG: erratic rhythm no discernable p, QRS, or T waves treatment: defibrillation
Determine the EKG -patho -treatment
atrial flutter patho: single foci having multiple p waves whereas a-fib has multiple foci EKG: -regular rhythm with sawtooth pattern -atrial rate: 250-350 bom -narrow QRS complex risk factors: COPD, <3 failure, CAD, ASD
Determine the EKG -risk factors -path
ventricular tachycardia 3 or more consecutive PVCs → structural <3 disease associated with sudden death risk treatment: -pulseless: defibrillate -pulse: cardioversion
Determine the EKG -treatment (pulse vs. pulseless)
SVT (rate from atria) EKG: -regular rhythm -rate: 160-220 bpm treatment: -vagal maneuvers -adenosine (ACLS: 6 mg then 12 mg) -if unstable (hypotensive) then cardioversion
Determine the EKG and treatment
SGLT-2 inhibitors MOA: increased urinary glucose excretion ADRs: UTIs, vulvovaginal candidiasis, bone fractures, DKA, AKI benefits: weight loss, reduced CV mortality, reduce systolic BP
Diabetes medication: Canagliflozin, dapagliflozin, empagliflozin -MOA -ADRs -pros
"tides" GLP-1 agonists MOA: mimicks incretin: insulin secretion, decreases glucagon, and delays gastric emptying ADRS: GI side effects, caution if pt has gastroparesis. Contrainidcated if patient has hx of pancreatitis pros: weight loss, reduced CV mortality
Diabetes medication: Dulaglutide, semaglutide -MOA -ADRs -pros
Sulfonylureas MOA: stimulates beta islet insulin release ADR: hypoglycemia benefits: cheap, rapidly effective
Diabetes medication: Glyburide, glipizide -MOA -ADRs -pros
MOA: decreases hepatic glucose production ADRS: -lactic acidosis -GFR < 30 mL: do not initiate D/C 24 hours BEFORE contrast and resume 48 hours after (if SCR >1.5 stop) pros: weight loss, inexpensive
Diabetes medication: Metformin -MOA -ADRs -Contrast -pros
Meglitinides MOA: stimulates beta islet insulin release ADR: hypoglycemia
Diabetes medication: Repaglinide, Nateglinide -MOA -ADRs
DPP-4 inhibitors MOA: inhibitors degradation of GLP 1 so more of it is circulating ADRs: expensive, increased risk of <3 failure
Diabetes medication: Sitagliptin, saxagliptin -MOA -ADRs
thiazolidinediones MOA: increases insulin sensitivity in adipose tissue (no effect beta islet cells) ADRS: -weight gain + fluid retention -bladder cxr -MI increase DO NOT USE IN CHF OR LIVER DX PTS
Diabetes medication: pioglitazone, rosiglitzaone -MOA -ADRs -contrainidcations -pros
arthrocentesis: diagnostic test of choice as this is definitive -shows negatively birefringent, needle shaped crystals -joint fluid also shows increases WBC > 20,000 and looks hazy radiograph: chronic gout shows "mouse" bite lytic lesions that are punched out erosions with sclerotic margins blood work: serum + UA elevated uric acid levels (but this is not always the case
Diagnostics for gout -arthrocentesis -radiograph -blood work
NO use oral fluconazole
Do clotrimazole troches treat esophageal candidiasis?
Normal CT findings do not rule out subarachnoid hemorrhage (although in first 6 hours almost 100% sensitive) . As demonstrated in this scenario, these patients must undergo a lumbar puncture to evaluate for the presence of blood (usually RBC is > 2,000 × 106/L) or xanthochromia
Do normal CT findings rule out subarachnoid hemorrhages?
cardiac: <3 not pumping blood out to lungs during exertion causing shortness of breath -ACS (angina, STEMI) -<3 failure -myocarditis -pericarditits pulmonary: lungs not functioning properly and lungs cannot exchange oxygen -asthma -copd -pneumonia -pulm hypertension -panic attack *anemia also cause of SOB check with CBC
Dyspnea upon exertion should make you think of what 2 systems and possible differentials
- anterior drawer test (AFTL) - talar tilt test (calcaneofibular ligament) I: stretching / small tears II: larger but incomplete tear III: complete tear
What are special tests to assess ankle sprain? Describe the grading of ankle sprains
infections usually large and require surgical intervention
What are struvite stones associated with?
-fever -tachycardia -leukocytosis -abdominal tenderness
What are the 4 main signs of strangulated bowel
1. pain along lateral or medial malleolus 2. midfoot pain, 5th metatarsal or navicular pain 3. unable to walk more than 4 steps
What are the Ottowa rules to image an ankle injury?
TPAIN IS 45 -T: trauma to head within last 3 months -P: platelet <100 K -A: active internal bleeding -I: intracranial hemorrhage -N: neurosurgery (last 3 months) -I: intestinal malignancy or bleed -S: stroke last 3 months -45: >4.5 hours of onset
What are the major contraindications of administering alteplase in treatment of an ischemic stroke patient
1. > 55 y/o 2. tenderness to the head of fibula 3. isolated tenderness to patella 4. cannot flex knee to 90 degrees 5. inability to weight bear (4 steps)
What are the ottawa rules to x-ray the knee?
(C) Delirium is the most likely diagnosis. presentation: rapid onset that waxes/wanes cognitive/perceptual problems such as: -memory loss -disorientation -difficulty w/ language and speech -hallucinations, delusions causes: -infection (leukocytosis = key in vignette along w/ acute onset) -hypoglycemia -Wernicke -stroke -poisoning, drug withdrawal workup/tx based on underlying cause
For the past 4 days, a 77-year-old female has been crying easily, confused, and rambling incoherently. Her medical history is remarkable for mild dementia and well-controlled hypertension. She has never had anything like this in the past and she has not had any recent changes to her medications. When questioned, she has no difficulty articulating a sentence but difficulty remembering what she was asked. Laboratory testing is significant for leukocytosis. What is the most likely diagnosis? -presentation -causes -workup/tx
GRUESOME MURDER Galeazzi: -radial fracture -ulnar dislocation (disal radio-ulnar joint) Monteggia: -ulnar fracture (proximal) -radial-head dislocation both are forearm fractures, both result in surgery as these are unstable (ORIF)
Galeazzi vs. Monteggia fractures -differences -treatment
Galeazzi: -radial fracture -ulnar dislocation (disal radio-ulnar joint) Monteggia: -ulnar fracture (proximal) -radial-head dislocation both are forearm fractures, both result in surgery as these are unstable (ORIF)
Galeazzi vs. Monteggia fractures -differences -treatment
A. fibular fracture below mortisse, tibiofibular syndemosis intact (unstable) B. fibiular fracture at level of mortisse, tibiofibular syndemosis intact or mildly torn (stable or unstable) C. fibular fracture above mortisse, tibiofibular syndemosis torn, deltoid ligament damage or medial malleolar fracture (unstable)
What are the weber ankle classification
Chvostek sign: facial twitching after facial stimulation Trousseau's sign: brachial compression causes anterior forearm contraction
What are two PE findings that are highly associated with hypocalcemia
-immobilization: splint, cast -reduction of fracture/dislocation is required in ANY pulseless limbs -antibiotics + tetanus prophylaxis if fracture is open -ortho referral
General treatment of extremity fractures
G5P1132
Give the patients gravidity + parity: 34 year old with 4 year old born at 35 weeks, 9 month old born at 38 weeks, 2 sponatenous miscarriages and 1 chemical pregnancy?
G2P1103
Give the patients gravidity + parity: 40 year old with 4 year old born at 40 weeks and 1 year old twins born at 32 weeks.
acute HBV infection
HBV serology: interpret the following -anti-HBC IgM: + -Anti-HBC IgG: - -HBSAg: + -Anti-HBS: -
chronic HBV
HBV serology: interpret the following -anti-HBC IgM: - -Anti-HBC IgG: + -HBSAg: + -Anti-HBS: -
"resolved" acute HBV infection
HBV serology: interpret the following -anti-HBC IgM: - -Anti-HBC IgG: + -HBSAg: - -Anti-HBS: +
HBV vaccine or immunity
HBV serology: interpret the following -anti-HBC IgM: - -Anti-HBC IgG: - -HBSAg: - -Anti-HBS: +
-elevate BNP -CXR: kerley B lines -echo: can differentiate HrEF vs. HpEF classifications: -I/A: NO limitations of physical activity / at risk of HF -II/B: slight limitation physical activity, comfortable at rest / structural <3 dx but no symptoms -III/C: marked limitation of physical activity, comfortable at rest / structural <3 dx with <3 failure -IV/D: unable to carry on any physical activity without discomfort and have sx of <3 failure or angina EVEN at rest / refractory <3 dx
Heart failure -lab/imaging -classifications
3 mo-5 y/o: amoxicillin +/- macrolide 5 y/o-18y/o: "walking" pneumonia caused by mycoplasma. CXR shows scattered infiltrates + atelectasis, rarely shows lobar consolidation. Tx w/ macrolide
How does the pneumonia treatment change from ages 3 mon-5y/o to 5-18 y/o
Any injury thought to have the potential to impair the patient's ability to appreciate other injuries.
How is a distracting injury defined in the National Emergency X-Radiography Utilization Study criteria?
annually if >3 cm 6 months if > 4 cm
How often should patients with AAA be monitored if the diameter of the aneurysm is >3 cm? What about >4 cm?
cancer
Hypercalcemia in the elderly is ______ until proven otherwise
excision of the thrombosed hemorrhoid for clot evacuation.
If a patient is presenting with severe pain from external thrombosed hemorrhoid within 72 hours, what treatment should be initiated?
male patients w/ gout
What are uric acid stones associated with?
indications: -cannot access veins due to: vein collapse, burns, cardiopulmonary arrest, or status epilepticus sites of insertion: -humeral head -distal femur (peds) -proximal tibia -distal tibia -sternum (trauma) contraindications: -absolute: fractured or previously punctured bone -relative: cellulitis, burn, osteopetrosis, osteogenesis imperfecta
IO Insertion -indications -sites of insertion -contraindications
need pseudomonas coverage levofloxacin + cefepime or pipercillin-tazo
If a patient develops hospital acquired pneumonia, how should they be empirically treated?
urgent referral to ENT want to drain the septal hematoma ASAP to avoid serious complications such as -A nasal septal abscess, can easily spread to adjacent structures including the sinuses and the brain. -The expanding hematoma can cause avascular necrosis of the cartilage and collapse, leading to a saddle nose deformity.
If a patient is presenting with a nasal septal hematoma, why do we want to treat this ASAP?
The goal for obtaining an initial electrocardiogram and blood sample is within 10 minutes of first medical contact. The electrocardiogram should be repeated at 20 to 30 minute intervals if suspicion remains high for acute coronary syndrome.
If a patient is presenting with acute chest pain, what is the time goal for getting an initial EKG and blood sample? How often should the EKG be repeated if suspicion for ACS is high?
swelling of the face, lips, tongue, neck can lead to airway compromise call HEENT ASAP if pt has dyspnea → plan for intubation epinephrine/Benadryl/steroids (if due to ACE-I these may not work but will still try!)
If a patient is presenting with angioedema, what are some potential interventions you can do?
causes: Excess of extravascular fluid → due to high salt, venous insufficiency, renal failure, HTN, CHF, recent travel diagnostics -CMP (LFTs, renal function), UA proteinuria -BNP <3 failure tx: low salt diet, compression stockings, diuretics
If a patient is presenting with bilateral peripheral lower extremity edema: -what are some potential causes -work up -treatment
airway maintenance b/c primary mechanism of death is asphyxiation (not exsanguination)
If a patient is presenting with hemopytsis what is the most important aspect of treatment?
median nerve !
In any distal radial fracture (colles or smith) what nerve is most at risk?
-continuously hypoxic -unable to eat or drink -severe dehydration (pernicious vomiting/diarrhea)
In general, when should a patient be admitted?
urinary retention; inability to pass urine w/ retention can be due to BPH, kidney stones send pt home w/ Foley
In older male patients with abdominal pain, what diagnosis should you always consider?
Upper endoscopy should be performed within 24 hours for evaluation and management of upper GI bleeding. Endoscopy is used to identify and locate the source of bleeding, achieve acute hemostasis, and prevent recurrent bleeding.
In patients with upper GI bleeding what will achieve acute hemostasis and prevent recurrent bleeding?
OK sign: anterior interosseus nerve (branch of median nerve) and radial nerve finger spread against resistance: ulnar nerve thumbs up sign: radial nerve
In the presence of a supracondylar fracture, describe the following tests to evaluate major nerve function - OK sign - Finger spread against resistance - thumbs up sign
PECARN (>2 y/o) -normal mental status -no LOC -no vomiting -no signs of basilar skull fracture -no HA -non-severe injury
What criteria is used to determine if a pediatric patient needs imaging after a fall or injury to head?
indirect: goes through inguinal ring + same course as the spermatic cord which is why it can pass into the scrotum or labia majora -lateral to the inferior epigastric vessels direct: bulge through weakened fascia of abdominal wall + rarely enter the scrotum -medial to the inferior epigastric vessels -old men w/ weak abdominal muscles MC pt type
Indirect vs. Direct hernia -location -relation to epigastric vessels -where does it pass
internal: BRBPR, pruritus, rectal discomfort -if protrudes with defecation, enlargement or intermittent bleeding tx w/ rubber band ligation external: PAIN but no bleeding thrombosed: palpable perianal mass with purplish hue tx: excision for thrombosed external hemorrhoids
Internal vs. External vs. Thrombosed hemorrhoids
indications -Patient is not breathing on their own / Respiratory Arrest -cannot protect airway (trauma) -Vomit (risk of aspiration) -Cardiac Arrest process: -Sniff position -Insert Laryngoscope to Vallecula -Push tongue forward to bring epiglottis forward (do not rock back on the teeth!) -Visualize the cords -Watch the tube pass through the cords -Remove the stylet, inflate balloon to stabilize, attach BVM, ventilate.
Intubation -indications -process
hemolytic: increased indirect/unconjugated bilirubin, dark urine (hemoglobinuria) obstructive: cholestasis due to bile duct blockage → increased conjugated/ direct bilirubin. dark urine d/t increased direct -GGT and ALP also elevated hepatocellular: increased indirect and direct bilirubin, AST + ALT elevated increased bilirubin without increased LFTs: gilbert's, hemolysis
Labs associated with the following causes of jaundice -hemolytic (prehepatic) -obstructive (post hepatic) -hepatocellular (intrahepatic)
unstable angina: -chest pain -EKG normal or with ST depression or T wave inversion -NO cardiac biomarkers NSTEMI: coronary artery not completed blocked, subendocardial infarct -chest pain -EKG with ST depression or T wave inversion - positive cardiac biomarkers STEMI: coronary artery completely blocked, full thickness of myocardial wall involved -chest pain -EKG ST elevation -positive cardiac biomarkers
What different lab findings would you see to differentiate between unstable angina, NSTEMI and STEMI
gravida = # of pregnancies parity: # of completed pregnancies beyond 20 weeks (viable + non-viable) TPAL: -Term pregnancies: >37 weeks -Premature Deliveries: <37 weeks -Abortions (<20 weeks) -living children twins count as one pregnancy two living children
What does gravida vs. parity mean? What about TPAL?
-age >30 y/o -smoker -prior malignancy
What factors makes a solitary pulmonary nodule more at risk of malignancy?
-NTG + aspirin -plavix -ACE-I -heparin -statins -BB IF STEMI: plavix + aspirin given immediately together
Medications initiated with ACS
MC location is Kisselbach's plexus (anterior) treatment: pinch anterior cartilaginous nose onto the septum x 30 minutes (should resolve in first 9-10 mins) secondary: Afrin, lidocaine, merocel pack. If bleeding cannot be controlled then need to consult w/ ENT
Nosebleeds -MC site -treatment (1st line) -secondary tx
-cat/dog bite: pasturella -sickle cell pt: salmonella -vertebral involvement: myobacterium TB (Potts dx) -prosthetic joints: staph epidermidis
Osteomyelitis + different organisms -cat/dog bite -sickle cell pt -vertebral involvement -prosthetic joints
increased trigeminal nerve and parasympathetic activity, leading to vasodilation in the intracranial vasculature
Patho of a cluster headache
toxic megacolon presentation: common in pts w/ UC or Crohns -fever -distended abdomen with peritonitis -hypotensive (shock) KUB: dilated colon > 6 cm treatment: decompression of colon
Patient brought into the emergency room appearing quite ill. He has a fever of 103.2°F, dry skin and oral mucosal membranes, and abdominal distention and tenderness. His medical history is significant for ulcerative colitis. What is the likely diagnosis? -presentation -x-ray -treatment
anterior/septal STEMI (V1-V4) left anterior descending artery is occluded
Patient presents with crushing chest pain and diaphoresis. Their EKG is as follows. What is the likely diagnosis & affected artery?
inferior STEMI (II, III, aVF) right coronary artery
Patient presents with crushing chest pain and diaphoresis. Their EKG is as follows. What is the likely diagnosis & affected artery?
lateral STEMI (I, AVL, V5-V6) left circumflex artery
Patient presents with crushing chest pain and diaphoresis. Their EKG is as follows. What is the likely diagnosis & affected artery?
Stable patients with a positive FAST should undergo a computed tomographic (CT) scan of the abdomen to reveal the source of the fluid.
Patient with stable patients with a positive FAST exam should undergo which imaging modality?
PJP treated prophylactically with bactrim
Patients with a CD4 count <200 should be prophylactically treated for what type of pneumonia infection?
causes: -H.pylori -NSAIDS/aspirin -Zollinger-Ellison Syndrome -smoking/alcohol/stress presentation: dyspepsia, aching, gnawing pain -gastric: pain greater w/ meals -duodenal: pain relieved w/ meals diagnostics: upper endoscopy w/ biopsy treatment: H2RA, PPI H. pylrori: bismuth quadruple therapy → bismuth subsalicylate + tetracycline + metronidazole + PPI x 14 days
Peptic Ulcer Disease -causes -presentation (gastric vs. duodenal) -diagnostics -treatment
A cervical spine MRI scan is the definitive way to diagnose and clear a cervical spine injury on patients of all ages. This diagnostic study allows for clear visualization of the ligaments, intervertebral disc spaces, spinal cord, and bones.
What imaging modality can definitively clear the C spine
<3, liver and kidneys edema MCC → salt retention ddx: -CHF -kidney dx -liver dx (not enough albumin) -pregnancy -drugs (DHP-CCB, alpha blockers vasodilate vessels)
Peripheral edema should make you think of dx states regarding what systems + differentials
-pinpoint pupils/miosis -respiratory depression -bradycardia -hypotension -decreased bowel sounds/constipation (long term) -AMS -sedation
Physical exam findings consistent with opioid overdose/intoxication
-temp >38 degrees -HR >90 -RR >20 -WBC >12,000 or less than 4,000 +suspected source of infection need at least 2 to qualify for SIRS
What is SIRS criteria for sepsis?
Chance fractures (unstable involves all 3 columns of the vertebra) MOA: flexion-distraction injury of the spine. Typically occur in the thoracolumbar junction, which spans from T11-L2. -Flexion-distraction injuries result from sudden deceleration and most commonly occur in restrained passengers of motor vehicle collisions and people who have fallen from a height. -These injuries are most commonly seen in people who wear only lap belts that are improperly positioned above the pelvic bones. This is especially common in pregnant or obese patients
What is a chance fracture?
tibial plateau fracture (check peroneal nerve)
What is a common fracture in kids during a motor vehicle accident?
fracture: - tender - pain with passive and active ROM - shortened, abducted, externally rotated
Presentation of a hip fracture
primary: no menses by age 13 with an absence of secondary sexual characteristics or by age 15 with normal secondary sex characteristics secondary: absence of menses for 3 months in women with previously normal menstruation or 6 month in a woman with a history of irregular cycle -MCC is pregnancy
Primary vs. secondary amenorrhea
immunoglobulin RhoGam
Rh negative women experiencing spontaneous abortion should be given
The brief moment of apnea that occurs in preparation for intubation can cause acute and significant worsening of the patient's respiratory acidosis. Intubation of patients exhibiting tachypnea in aspirin overdose has resulted in death and should be reserved for patients with hypoventilation and severe respiratory distress.
Should patients with aspirin toxicity be intubated?
Salmonella (tx w/ 3rd gen cephalosporin)
Sickle cell osteomyelitis pathogen
-coma -brainstem herniation -seizures high risk pts are alcoholics / malnourished
What is a major concern with severe hyponatremia?
-GCS 13 to 15, 30 minutes after injury -consciousness lost <30 minutes -amnesia (memory loss) lasting <24 hours
What is a mild TBI (aka concussion) defined as
mild: score of >13 moderate: score 9-12 severe: score <8 (comatose) best score: 15 lowest score: 3
What is a mild, moderate, or severe GCS score?
cerebral edema :( patho: osmotic changes + fluid shifts during DKA → accumulation of intracellular osmolytes inside brain cells. Then during treatment decline in intravascular osmolality → water follows the party + moves inside brain cells causing cerebral edema presentation: -AMS -HA -seizure -bradycardia tx: -IV mannitol -IV hypertonic saline to maintain BP + cerebral perfusion
What is a rare complication that can occur if electrolyte abnormalities are overcorrected when treating diabetic ketoacidosis in children?
monitor for hepatocellular carcinoma -ultrasound every 6-12 months -monitor AFP
Screening for patients with cirrhosis includes
sprain: ligaments -MC site: ankle strain: muscles + tendons -hamstring + lower back most common sites
Sprain vs. Strain
Sprain: involves ligaments strain: involves muscles + tendons
Sprain vs. strain
Allen's test
What is an important test to ensure radial and ulnar artery flow when differentiating ganglion cyst vs. vascular aneurysm?
-Facial sutures are placed for 3-4 days -scalp sutures for 5-7 days -trunk sutures are placed for 6-8 days, -sutures on the extremity are placed for 7-14 days.
Suture time frame -face -scalp -trunk -extremities
**dexamethasone** 18-50 y/o -ceftriaxone and vanco >50 y/o -ceftriaxone + vanco + ampicillin (covers Listeria)
What is empiric treatment for bacteria meningitis for: -18-50 y/o - >50 y/o
pts who have normal voiding systems but difficulty reaching toilet due to mental or physical disability treatment: schedule voiding times
What is functional incontinence
ingrown toenail treatment
What is onychocryptosis also known as
primary: parathyroid adenoma secondary: CKD (increased PTH d/t hypocalcemia or vitamin D deficiency)
What is the MCC of primary vs. second hyperparathyroidism?
The USPSTF recommends one-time screening for abdominal aortic aneurysm by ultrasonography in men ages 65 to 75 years who have ever smoked
The USPSTF recommends screening for AAA in what population and via what imaging method
pelvic inflammatory disease patho: ascending sexually transmitted infection; cervix → endometrium → fallopian tubes → pelvic cavity presentation: abdominal or pelvic pain (burning, cramping, stabbing) can have guarding/rigidity -systemic sx: fever, chills -increased vaginal discharge, abnormal bleeding, dyspareunia -UTI + GI sx can also present PE: -Chandelier's sign: cervical motion tenderness -lower abdominal tenderness -purulent cervical discharge -friable cervix diagnostics: clinical: cervical motion tenderness -definitive via laparoscopy -r/o ectopic pregnancy -STD testing -transvaginal u/s: thicken fluid filled tubes treatment: ceftriaxone 500 mg IM + doxycycline 100 mg BID x 14 days -Inpatient care: HIV, fail outpt therapy, cannot r/o ectopic or appendicitis
The patient is a 19-year-old woman who presents due to lower abdominal pain, chills, and vaginal discharge. She admits to being sexually active and has a history of irregular menstruation. Her pelvic examination reveals bilateral adnexal and cervical motion tenderness, as well as purulent discharge seen at cervical os. B-hCG levels are within normal limits. What is the likely diagnosis? -patho -presentation -PE -diagnostics -treatment (when to admit)
cocaine intoxication stimulant intoxication: -psychomotor agitation: pressure speech, tremors, pacing -elevated or euphoric mood -SNS hyperactivity: pupillary dilation, flushing -SEVERE: hyperthermia, seizures, arrhythmias -psychoses in severe cases: flight of ideas, echolalia, delusions treatment: benzodiazepines inhibits SNS activity -psychosis: haloperidol (antipsychotic)
The sister of a 20-year-old man is concerned because her brother has not been himself lately; his mood has been alternating from happy and euphoric to irritable and depressed. The man states that he is adjusting to his new life at college, which has so far been stressful. He states that he does not smoke, and only drinks socially at parties. He admits to using recreational drugs. Physical examination reveals marked nasal congestion, dilated pupils, heart rate of 120 beats/min, and a blood pressure of 155/92 mmHg. What is the likely diagnosis -treatment
Calculate fluid needed in burns → 4 mL (LR) x Weight (Kg) x BSA burned -BSA differences: head in kids: 18% front + back whereas adults = 9% front + back -Give ½ first 8 hours → rest of next 16 hours
What is the Parkland Formula utilized to fluid resuscitate a burn patient?
"Pop eye" deformity is associated with bicep tendon rupture
This type of deformity is associated with what type of injury?
Hashimotos: MCC hypothyroidism Post-partum: 2-12 months after giving birth -during pregnancy immune system diminished → after childbirth immune system more active and might attack Quervain's: MCC of thyroid pain -painful, inflamed thyroid -post-infection usually viral infection -increased ESR medications: lithium, amiodarone -check TSH every 6-12 months
Thyroiditis DDX -Hashimoto's -Post-partum -Quervain's -medication
TB is a four-drug regimen for 2 months consisting of isoniazid, rifampin, pyrazinamide, and ethambutol, followed by 4 months of isoniazid and rifampin, for a total of 6 months of treatment.
Timeline for TB treatment
N-acetylcysteine
Toxicology: what is the antidote for acetaminophen
sodium bicarbonate (helps alkalize urine + promote excretion)
Toxicology: what is the antidote for aspirin overdose
flumazenil
Toxicology: what is the antidote for benzodiazipine overdose
fomepizole
Toxicology: what is the antidote for ethylene glycol
naloxone
Toxicology: what is the antidote for opioid overdose
sodium bicarbonate
Toxicology: what is the antidote for tricyclic antidepressants
Transudate: decreased oncotic pressure or increased hydrostatic pressure → CHF(MCC), nephrotic syndrome, cirrhosis thoracentesis: low LDH, low protein tx: diuretics, sodium restriction Exudate: any condition associated w/ infection or inflammation leading to increased cap permeability → pneumonia, cancer, TB, PE thoracentesis: high LDH, high protein
Transudative vs. exudative pleural effusions
narrow: impulse coming from above AV node -CCB, BB -adenosine -procainamide -cardioversion if unstable wide: impulse coming from ventricles -cardioversion or defibrillation -amiodarone
Treatment in narrow vs. wide complex tachycardias
reversible: 3 months unprovoked: 6 months 2 episodes unprovoked: long term/indefinite
Treatment length for pulmonary embolism with reversible causes vs. unprovoked
Stable patients may be treated with intravenous or intraosseous adenosine, and if adenosine is ineffective, procainamide or amiodarone should be used. Unstable patients with monomorphic VT should be immediately treated with synchronized direct current cardioversion, unstable polymorphic VT is treated with immediate defibrillation.
Treatment of v-tach -stable -unstable (monomorphic vs. polymorphic)
-Nitrofurantoin -Keflex
Treatment options for pregnant patients with lower UTI
end-tidal CO2
What is the best way to confirm that the intubation tube is in the correct location?
eye opening: -4: spontaneous -3: response w/ verbal commands -2: response to pain -1: no eye opening best verbal response: -5: oriented -4: confused -3: inappropriate words -2: incomprehensible sounds -1: no verbal responses best motor response: -6: obeys commands (can. you wiggle toes) -5: localizing response to pain -4: withdrawal response to pain -3: flexion to pain (decorticate) -2; extension to pain (decerebate) -1: no motor response
What is the criteria for GCS -eyes -verbal -motor
stable angina: predictable pattern of chest pain where an activity always elicits chest pain. Caused by a fixed plaque obstruction on 1 or more coronary arteries and some endothelial dysfunction (unable to vasodilate) EX: Patient gets out of breath and has chest pain when they walk up three flights of stairs, but after resting and taking nitroglycerin their symptoms go away ACS conditions → unstable plaque! unstable angina: (precursor to MI) change in chronic stable angina with increased with increased frequency, intensity, and duration of chest pain with longer recovery time. EX: patient gets out of breath now at 2 flights of stairs and it takes much longer for the pain to go away after resting
What is the difference between stable angina vs. unstable angina) -give example scenario
-endoscopy with biopsy (underlying cause) -manometry -24 pH probe -barium esophagram
What is the general work up for heartburn
12 weeks: symphysis pubis 20 weeks: umbilicus after 20 weeks: 1 cm for every week of gestation
Uterine growth -12 weeks -20 weeks -after 20 weeks
gold standard: angiography to assess the severity of coronary artery lesions when considering PCI or CABG can also do nuclear stress testing (but takes longer)
What is the gold standard when evaluating unstable angina? What are other options?
seizures** pt presents with arm that is adducted and internally rotated
What is the main etiology of a posterior shoulder dislocation? How will the patient present?
dizziness (orthostatic hypotension)
What is the most common ADR of alpha blockers such as tamsulosin
sinus tachycardia
What is the most common EKG finding associated w/ pulmonary embolism
renal failure
What is the most common cause of hyperphosphatemia?
bronchitis
What is the most common presentation of mild or acute hemopytsis?
posterior dislocation (talo-tibial dislocation) talus moves in a posterior direction to the tibia
What is the most common type of ankle dislocation
AV nodal reentrant tachycardia (AVNRT) accessory pathway (reentry circuit) in or near AV node leads to premature heart beats (p waves can be hidden)
What is the most common type of supraventricular tachycardia?
mitral MC valve involved IVDU: tricuspid valve involved
What is the most common valve involved in acute bacterial endocarditis? What about in IVDU?
hypertension
What is the most significant predisposing factor in regards to aortic dissection?
diabetes** other causes: -HTN -smoking
What is the number 1 cause of CKD/renal failure?
1. Neer test: arm pronatated with pain during forward flexion 2. Hawkin's: elbow flex, pain with internal rotation 3. drop arm: inability to lift arm above shoulder level subacromial impingement: inflammation of rotator cuff tendons as they pass through subacromial space
What 3 tests assess for subacromial impingement
-empty can test (rotate externally) -full can test (rotate internally) -arm drop: Arm passively abducted and actively adducted slowly. Positive if arm drops quickly @ 30 degrees
What 3 tests assess supraspinatus muscle of rotator cuff?
1. bell clapper formation: seen in intravaginal torsion (puberty) → horizontal/transverse testicle 2. NO cremasteric reflex 3. negative Prehn's sign (no pain relief with elevation of the scrotum) 4. Blue dot sign: tender nodule 2-3 mm on the upper pole of testicle
What 4 unique PE findings are associated with testicular torsion?
Bactrim
What ABx can reduce the frequency of UTIs in sexually active women if taken post-coitally?
1. colorectal cancer (consider all pts >50 w/ new onset constipation) 2. bowel obstruction: x-ray with air-fluid levels + dilated loops of bowel 3. volvulus: obstruction d/t twisting of GI tract. Presentation: belly pain, bloating, nausea, bloody stools. X-ray: colonic distension 4. ileus: hypomobility of GI tract in absence of mechanical obstruction, absent bowel sounds 5. gastroparesis: diabetic pt; vomiting, fullness after eating small amounts
What are 5 DDx to consider in a patient with severe constipation
- >25% TBSA burn in adults - >20%TBSA burn in young/old - >10% full thickness burn -Burns involving the face, hands, perineum, feet -Burns crossing major joints, circumferential burns admit to burn unit/ICU for major burns
What are 5 classifications that qualify a burn as a major burn?
ophthalmologist (retinopathy, leading cause of blindness) urine microalbumin measurement (nephropathy) foot checks! (neuropathy)
What are annual follow ups for diabetics?
-vertical nystagmus*** -ataxia -autonomic dysfunction: reduced respiratory drive, problems chewing, heart rate low
What are concerning findings that could be indicative that a patient's vertigo is due to cerebellar/brainstem stroke?
1. water deprivation test: continued production of diluted urine despite water deprivation (low urine osmolality, high serum osmolality) 2. desmopressin stimulation test: differentiate between central and nephrogenic -central: reduction in urine output (b/c responding to ADH) -nephrogenic: continued production of diluted urine (no response to ADH)
What are diagnostic tests for diabetes insipidus
-purine rich foods like alcohol, liver, seafood that increase UA concentrations -medications such as thiazide, loop diuretics, ACE, aspirin -surgery, trauma
What are factors that precipitate gout flares
-Heinz bodies (damaged HgB precipitates) -Bite cells causes: fava beans, antimalarials, sulfonamides X linked dx (African American Males)
What are findings on PBS associated with G6PD deficiency? What are common causes?
-age >65 -hx of ulcers -high dose NSAID therapy -concurrent use of corticosteroid/anticoagulant/aspirin
What are general risk factors for NSAID GI toxicity
1. cholesterol will increase 2. BUN + Cr will decrease 3. cardiac output increases
What are lab changes that are associated with pregnancy
-vasovagal (MCC, trigger/warning sx) -arrhythmias, structural <3 dx -cerebral hypoperfusion( CVA, hypovolemia) -POTS -seizures
What are potential causes of syncope
-rifampin -cipro -azithro -ceftriaxone
What are prophylactic treatment options for bacterial meningitis
-previous ectopic* -previous PID -hx abdominal or tubal surgery -IUD -smoking
What are risk factors for ectopic pregnancy?
-maternal HTN** -smoking -elevated age -drug use (cocaine, meth) -increasing parity -trauma (MVA, domestic violence, blunt abdominal trauma)
What are risk factors for placentio abruptae?
Secondary causes of polycythemia are related to chronic hypoxia and tobacco use (e.g., patients with COPD) or erythropoietin-secreting tumors. It commonly presents in men, with a median age of onset of 60 years. phlebotomy or hydroxyurea
What are secondary causes of polycythemia vera? Treatment options?
-severe abdominal or shoulder pain -peritonitis -tachycardia, hypotension -syncope
What are signs/symptoms associated with ruptured ectopic pregnancy?
-Infection → Osteomyelitis -Fat embolism -Fluid extravasation → compartment syndrome -Bone fracture -Avoid these using proper insertion technique, asepsis, monitoring, and prompt removal
What are some complications to IO insertion?
Bleeding stops spontaneously in the majority of cases Explanation Mallory-Weiss syndrome involves a tear of the mucosa and submucosa of the lower esophagus or gastric cardia and accounts for up to 9% of cases of significant upper gastrointestinal bleeding. This syndrome results from forceful events that increase intra-abdominal pressure, including trauma, seizures, coughing, and, most commonly, vomiting, with retching preceding hematemesis in approximately 50% of patients. Mallory-Weiss syndrome is associated with alcoholism, hiatal hernias, esophagitis, or gastritis. The majority of episodes involve only mild to moderate bleeding that stops spontaneously, and surgery is rarely required.
Which of the following is true regarding Mallory-Weiss syndrome? 1 Operative repair usually is necessary 2 Is the most common cause of gastrointestinal bleeding during pregnancy 3 Endoscopy is contraindicated 4 Bleeding stops spontaneously in the majority of cases 5 Accounts for 1/3 of deaths from upper gastrointestinal bleeding
soft tissue inflammation of the pericolic fat
Which one of the following is a characteristic finding on computed tomography (CT) of the abdomen in a patient with acute diverticulitis? A toxic megacolon B air-fluid levels C soft tissue inflammation of the pericolic fat D thinning of the colon wall E paucity of bowel gas in the colon
-acute bacterial: infection of normal valves with virulent organisms (S. aureus) -subacute: indolent infection of abnormal valves with less virulent organisms (S. viridans) **IVDU: s. aureus involved
Which pathogens are involved in acute bacterial endocarditis vs. subacute bacterial endocarditis
early decelerations these are normal. begin near the onset of a uterine contraction, and their lowest point occurs at the same time as the peak of the contraction
Which type of fetal electric <3 strip is associated with compression of baby's head as it passes through the uterus + birth canal
variable decelerations
Which type of fetal electric <3 strip is associated with compression of the umbilical cord?
late decelerations uteroplacental insufficiency→ decrease in the blood flow to the placenta → reduces the amount of oxygen and nutrients transferred to the fetus.
Which type of fetal electric <3 strip is associated with uteroplacental insufficiency + hypoxemia
The correct answer is bilateral ankle-brachial indices. This patient's history and exam are concerning for a knee dislocation. Knee dislocations are classified according to the direction that the tibia is displaced in relation to the femur. Anterior knee dislocation is the most common type, and often due to a high-velocity injury such as an MVA. While rare, dislocations of the knee have significant complications involving both structural damage to the knee (disruption of tendons, ligaments, and cartilage), as well as neurovascular injuries (injury to the popliteal and/or tibial arteries, and damage to the peroneal nerve most commonly). this is a surgical emergency + limb threatening!
You are evaluating a 28-year-old man who is brought in by ambulance for a right knee injury sustained during an MVA. The patient is in obvious distress, holding his right knee and screaming. You are told by EMS that the patient was the restrained driver of a car that was rear-ended at high speed. You are unable to obtain any additional history. Examination already shows the development of a right knee effusion. The skin is intact, and there does not appear to be any deformity. Examination of laxity is limited, secondary to the patient's pain. It appears that distal sensation is intact but you cannot palpate a dorsalis pedis or posterior tibial pulse. You order an emergent MRI for further evaluation. Question In the meantime, what test would be best to help confirm the patient's diagnosis?
peritonsillar abscess presentation: -severe sore throat -unilateral tonsillar swelling -deviation of uvula -trismus (limited mouth opening) -hot potato voice -drooling tx: EMERGENCY; can lead to airway compromise -I&D w/ amoxicillin -consult w/ ENT next day
You are seeing a 19-year-old male for follow-up from urgent care where he was seen 2 days earlier with a sore throat. The patient is febrile (102°F), has a muffled (hot potato) voice, and extreme difficulty opening his mouth (trismus). He opens it just far enough for you to note uvular deviation. What is the likely diagnosis? -presentation -treatment
Ludwig's angina patho: Severe floor of mouth cellulitis d/t dental infection → life threatening airway compromise presentation: -tongue + floor of mouth is raised + edematous -dysphagia -difficulty talking -difficulty breathing → asphyxiation diagnostics: CT Neck w/ contrast (see cellulitis) tx: ENT Emergency life threatening airway compromise → ENT + Anesthesia STAT (pt will get worse + worse) need to secure airway
Your patient is a 35-year-old male who recently underwent restoration of teeth #28, 29, and 30. He has a 24-hour history of fever, dysphagia, odynophagia, and drooling secondary to inability to swallow his own secretions. Examination of the patient shows a toxic-appearing male with trismus, edema of the neck, and submandibular area along with poor dentition, foul smelling breath, and gingivitis. The tongue, which is painful for the patient to move is displaced posteriorly, and a temperature of 103°F is noted. What is the likely diagnosis? -patho -presentation -diagnostics -treatment
mitral valve prolapse often d/t Marfan Dx murmur caused by an abnormal systolic ballooning of part of the mitral valve into the left atrium; midsystolic click heard best the the apex -decreases/comes later with increased preload: squatting, leg raise, expiration -increases/comes earlier with decreased preload: standing, valsalva, hand grip, inspiration
a 22-year-old female who complains of generalized, sub-sternal chest pain that is worsened with exertion. She appears anxious; she denies ETOH, tobacco, and illicit drug use. You auscultate her heart and hear a midsystolic click. -what increases or decreases the muurmur
Biceps tendonitis presentation: -pain @ biceps groove -anterior shoulder pain -pain with resisted supination - popeye deformity = rupture diagnostics: -U/S: thickened tendon within bicipital groove -MRI: increased T2 signal treatment: NSAIDS, PT, steroid injection, surgical release
a 33-year-old man who complains of left anterior shoulder pain for 4 weeks. The pain is made worse with overhead activities. On examination, you note maximal pain in the shoulder with palpation between the greater and lesser tubercle. Pain in the shoulder is exacerbated when the arm is held at the side, elbow flexed to 90 degrees, and the patient is asked to supinate and flex the forearm against your resistance. What is the likely diagnosis? -presentation -diagnostics -treatment
esophagitis reflux: mechanical or functional abnormality of the LES (GERD, acidic stomach contents come into the esophagus) medication: NSAIDS or bisphosphonates -prevent bisphosphonate esophagitis by drinking with 4 oz water, avoid laying down 30-60 mins after ingestion eosinophilic: pts w/ asthma sx + GERD not responsive to antacids -barium swallow: ribbed esophagus with multiple corrugated rings -tx: remove foods causing allergy, inhaled steroids
a 54-year-old female with dysphagia and retrosternal chest pain. Vitals and physical exam are normal. What is the likely diagnosis? describe the following types -reflux -medication induced -eosinophilic
infectious esophagitis odynophagia (pain while swallowing food or liquids) = hallmark sign MC found in immunocompromised patients Candida: yellow-white plaques on EGD. Tx w/ fluconzaole HSV: shallow punched out lesions on EGD, tx with acyclovir CMV: solitary ulcers or erosions on EGD, tx w ganiciclovir
a 54-year-old male with a history of HIV presents with odynphagia, dysphagia, retrosternal chest pain. What is the likely diagnosis? -describe the most common etiologies, findings on EGD, and tx
aortic regurgitation (insufficiency) murmur: diastolic blowing decrescendo murmur, increases when patient is sitting and leaning forward
a 61-year-old male presents with a recent history of increased fatigue with mildly increased exertional dyspnea. Patient denies any significant past medical history but states that he had some heart problems as a child, though he was never clear as to what was the problem. On cardiac examination, you hear an early diastolic, soft blowing decrescendo murmur with a high pitch quality, especially when the patient is sitting and leaning forward. No thrill is felt. What is the likely diagnosis? -describe murmur
Prinzmetal variant angina patho: coronary artery vasospasms causing transient ST-segment elevations NOT associated with a clot. Can have ST elevation, T wave abnormalities or inverted T waves. Cardiac markers can also be positive. **exercise capacity is preserved risk factors: #1 RF is smoking history, cocaine abuse treatment: -acute: nitrates -long term/preventative: CCB and long acting nitrates. *long acting nitrates should include a daily 8-10 hour treatment free interval to prevent drug tolerance
a 65-year-old man who presents to the ED at 1 am with 90 minutes of central chest pressure that awoke him from sleep. He says he thinks he has 'indigestion.' The pain is non-radiating, with mild shortness of breath but no nausea, vomiting or diaphoresis. He is an ex-smoker with a 20 pack-year history. There is no previous history of CAD, diabetes, hypertension, or high cholesterol. On arrival, he looks well, with a normal heart rate (64 bpm), blood pressure (127/86), and oxygen saturation of 98% on RA. His pain has improved slightly with sublingual nitrates in the ambulance, although he still has some ongoing chest discomfort. His ECG is shown below. Troponins are positive. He is admitted to the hospital and undergoes emergent cardiac catheterization, where he is without obstructive coronary disease. What is the likely diagnosis? -patho -risk factors -treatment (acute vs. long term)
Atrial fibrillation patho: multiple atria foci conducting chaotically. presentation: pts that are elderly, excessive alcohol use -syncope -dyspnea -palpitations EKG: -irregularly irregular rhythm -absence of p -waves -narrow QRS treatment: -rate control <110 bpm: non-DHP CCB or BB -rhythm control: depends on amount of time → less than 48 hours: cardioversion, amiodarone → greater than 48 hours: anticoagulate for 21 days before cardioversion -anticoagulate with DOACs (warfarin if mechanical <3 valve or mitral stenosis) using CHADSVASC score
a 65-year-old woman with palpitations. Her past medical history is notable for chronic obstructive pulmonary disease (COPD) for which she has been hospitalized once in the last year. On exam her T 98.4F, HR 86, BP 105/70, RR 18, SpO2 94% on room air consistent with her baseline. EKG shows the following. What is the likely diagnosis? -patho -presentation -EKG -treatment
arm/shoulder pain at rest whereas rotator cuff injury pain elicited with movement only
cervical disc herniation pain vs. rotator cuff pain
long chain omega 3 fatty acids (seafood)
consumption of what can decrease women's risk of endometriosis?
-severe egg allergy -GBS within 6 weeks of previous vaccination -<6 months -flumist pt with asthma
flu vaccine contraindications
Treatment of children with heat stroke consists of immediate whole body cooling, usually performed by cold water immersion. Rapid cooling should be continued until the temperature decrease to 101 or 102°F.
how is heat stroke in children treated
chest x-ray (free air under the diaphragm)
if a perforated gastric or duodenal ulcer is suspected what imaging should be done?
Injuries to and around the elbow joint have a high risk of joint stiffness, and prolonged immobilization of the elbow is contraindicated as this can result in reduced ROM
most common complication of supracondylar humeral fractures
-Normal pH: 7.35-7.45 -Normal CO2: 35-45 -Normal HCO3: 22-26 -O2 sat: 94%+ -PaO2: 75-100
normal ABG values -pH -pCO2 -HCO3 -O2 sat -pAO2
A dilation and curettage (A) is used for many reasons, including endometrial sampling, persistent abnormal uterine bleeding, and evacuation for miscarriage, molar pregnancy, or suspicion of retained products. This procedure is avoided and not indicated in patients with an active pelvic infection due to risk of worsening infection.
should you do dilation and curretage in female patients with a pelvic infection?
treatment of hemodynamically stable patients with threatened abortion is expectant management. Patients who have mild vaginal bleeding with an ultrasound confirming an intrauterine pregnancy with fetal cardiac activity can be reassured that in most cases the bleeding does not lead to pregnancy loss. Patients can be discharged and can follow up for a repeat ultrasound with their obstetrician. Patients should be educated to return to the emergency department if they develop heavy bleeding (saturating a pad an hour for 2 hours in a row).
treatment for threatened abortion in a hemodynamically stable patient
-confusion -urea >7 -RR >30 -Systolic BP <90 -age >65 0-1: low risk 2: probable admission or close outpatient monitoring 3: admission (severe)
what is the CURB-65 score to determine pneumonia severity?
Adenosine, beta-blockers, calcium channel blockers, and digoxin.
what medications should be avoided in WPW